Pediatrics Final Exam, Patho PrepU: 5, EPID 512 Final, PATHO - Cellular Response to Stress, Injury & Aging, Patho Final, NUR01: Patho → Quiz 1, Pathophisiology - test 2 w/ feedback, Patho Unit 1 Exam, Pathophysiology Review, CH. 2 Pathophysiology, Pa...

Ace your homework & exams now with Quizwiz!

The Krebs cycle provides a common pathway for the metabolism of nutrients by the body. The Krebs cycle forms two pyruvate molecules. Each of the two pyruvate molecules formed in the cytoplasm from one molecule of glucose yields another molecule of:

ATP

Which of the following processes will be most immediately disrupted by this side effect?

ATP synthesis

A client with a diagnosis of liver cirrhosis secondary to alcohol abuse has a distended abdomen as a result of fluid accumulation in his peritoneal cavity (ascites). Which of the following pathophysiologic processes contributes to this third spacing?

Abnormal increase in transcellular fluid volume

A patient with a diagnosis of liver cirrhosis secondary to alcohol use has a distended abdomen as a result of fluid accumulation in his peritoneal cavity (ascites). Which of the following pathophysiologic processes contributes to this third spacing?

Abnormal increase in transcellular fluid volume

Which of the following factors is most strongly associated with the pathogenesis of gallstones?

Abnormalities or stasis of bile

A distinguishing feature of viral influenza is:

Abrupt-onset, profound malaise

Impairment in the function of peroxisomes would result in:

Accumulation of free radicals in the cytoplasm

AIDS

Acquired immune deficiency syndrome. Something a person gets from someone else. Your defense system for fighting off diseases. Below normal level. Group of signs and symptoms that occur together.

In contrast to acute inflammation, chronic inflammation is characterized by which of the following phenomena?

Activated macrophages

When explaining acute pancreatitis to a newly diagnosed patient, the nurse will emphasize the pathogenesis begins with an inflammatory process whereby:

Activated pancreatic enzymes escape into surrounding tissues, causing autodigestion of pancreatic tissue

A client with a diagnosis of chronic renal failure secondary to diabetes has seen a gradual increase in her blood pressure over the past several months, culminating in a diagnosis of secondary hypertension. Which of the following has most likely resulted in the client's increased blood pressure?

Activation of the renin-angiotensin-aldosterone mechanism

Naturally acquired immunity

Active: Infection; contract with pathogen. Passive: Antibodies pass from mother to fetus via placenta; or to infant in her milk.

Artifically acquired immunity

Active: Vaccine; dead or attenuated pathogens. Passive; injection of immune serum (gamma globulin)

The most common causes of left-sided heart failure include:

Acute myocardial infarction

A child is recovering from a bout with group A β-hemolytic Streptococcus infection. They return to the clinic a week later complaining of decrease in urine output with puffiness and edema noted in the face and hands. The health care provider suspects the child has developed:

Acute postinfectious glomerulonephritis

Lymphocytes

Adaptive immune responses, also called acquired or specific immunity, are composed primarily of which type of cells?

the signs and symptoms of abrupt cessation of pharmacologic glucocorticoids resemble those of:

Addison Disease

A decrease in the serum level of which of the following substances is suggestive of liver injury?

Albumin

Acute gastritis refers to a transient inflammation of the gastric mucosa that is most commonly associated with:

Alcohol intake

A client presents to the emergency department following a major traffic accident. Though outwardly there are no apparent physical injuries found, the client is experiencing chest pain and heightened alertness, which the health care worker attributes to the first stage of general adaptation syndrome (GAS). The health care worker concludes the client is experiencing manifestations related to the release of:

Aldosterone, which interferes with sodium absorption

Which of the following statements about cell structures best supports the common ancestry of all living things?

All cells contain ribosomes.

A client has been given the diagnosis of diffuse glomerulonephritis. They ask the nurse what diffuse means. The nurse responds:

All glomeruli and all parts of the glomeruli are involved.

An otherwise healthy client has been referred to a pain clinic because she claims to experience exquisite pain from the friction of her clothes on her torso. The client is likely to be diagnosed with which of the following health problems

Allodynia

A nurse working at nights is constantly complaining of being tired and sick. It seems like she catches every illness that is on the unit. One possible reason for this may relate to:

Alterations in sleep-wake cycle have decreased her immune function.

An elderly client has experienced some hypoxia as a result of chronic respiratory problems. Knowing that oxygen -deprived cells result in an accumulation of lactic acid in the cells, physiologically, the client may experience:

Altered cell membrane permeability

Injured cells become very swollen as a result of:

Altered cell regulation

Injured cells become very swollen as a result of:

Altered cell volume regulation

Which digestive enzyme breaks down carbohydrates during acute pancreatitis?

Amylase

When assessing the client with acute pancreatitis, which of these diagnose tests- consistent with the disease- dose the nurse anticipate will be altered?

Amylase and lipase

A review of the client's medications Placing the client on an alternating pressure mattress Nutritional supplements

An 80-year-old client has a stage 3 decubitus ulcer on the left ischial tuberosity which has not shown much improvement despite optimal local wound treatment. What other interventions should the nurse recommend to promote wound healing? Select all that apply.

Which of the following clients should most likely be assessed for orthostatic hypotension?

An 80-year-old elderly client who has experienced two falls since admission while attempting to ambulate to the bathroom

Which of the following residents of a long-term facility is exhibiting signs and symptoms that are indicative of hypothyroidism?

An 80-year-old woman who has uncharacteristically lost her appetite of late and often complains of feeling cold

Which of the following clients would be considered high risk for falling and fracturing a hip?

An 81-year-old female taking medication for chronic osteoporosis

Which of the following individuals should be prioritized for receiving a seasonal influenza vaccination?

An 81-year-old resident of a long-term care facility

Immunodeficiency

An absent or depressed immune response, thus increasing susceptibility to infection.

24-year old women presents with fever and painful, swollen cervical lymph nodes and sore throat that she has had for 5 days. Her CBC with different indicates an increase in neutrophils with a shift to the left. She most likely has:

An acute bacterial infection

Which of the following emergency department clients is most likely demonstrating clinical manifestations of acute stress? A client with:

An acute heightened sense of alertness to surroundings and personnel

Which of the following individuals displays the precursors to acromegaly?

An adult with an excess of growth hormone due to an adenoma

Which of the following clients would be considered a good candidate for hyperbaric oxygen therapy?

An older adult with history of diabetes and intermittent claudication (poor circulation in the legs) who received a laceration while on the lawnmower

88 year old resident of a long term care home has been suffering from a three day onset of increasing shortness of breath and decreased oxygen saturation. at the hospital, an anterior-posterior chest X-Ray and sputum culture and sensitivity has confirmed a diagnosis of bacterial pneumonia, yet the client's tympanic temperature has not exceeded 37.3 degrees C. The health care team would recognize that which of the following phenomena likely underlies this situation.

An older adult's hypothalamus has diminished thermoregulatory ability.

A client experiences burning muscle pain at the peak of high intensity physical training for an athletic competition. What is the best explanation for the cause of this occurrence?

Anaerobic glycolysis creates pyruvate, which converts to lactic acid.

Leukemia is associated with immature, non-functioning white blood cells to start rapidly proliferating and accumulating in the bone marrow. What does this lead to?

Anemia, thrombocytopenia

A 12 yr old boy's severe wound that he received from a dog bite has begun to heal and currently shows no signs of infection. Which process occurred first during the repair of connective tissue deposition?

Angiogenesis

A 12-year-old boy's severe wound that he received from a dog bite has begun to heal and currently shows no signs of infection. Which of the following processes occurred first during this process of repair by connective tissue deposition?

Angiogenesis

A client presents for a scheduled Papanicolaou (Pap) smear. The clinician who will interpret the smear will examine cell samples for evidence of: A) Changes in cell shape, size, and organization B) The presence of unexpected cell types C) Ischemic changes in cell samples D) Abnormally high numbers of cells in a specified field

Ans: A Feedback: A Pap smear is an example of a diagnostic procedure that tests for the presence of cell dysplasia, that is, deranged cell growth of a specific tissue that results in cells that vary in size, shape, and organization. Unexpected cell types are evidence of metaplasia, whereas ischemic changes are associated with cell hypertrophy. Increases in the number of cells are characterized as hyperplasia.

Which body tissue exhibits the highest rate of turnover and renewal? A) The squamous epithelial cells of the skin B) The connective tissue supporting blood vessels C) The skeletal muscle that facilitates movement D) The nervous tissue that constitutes the central nervous system

Ans: A Feedback: Cells making up the epithelial tissues generally exhibit a high rate of turnover, which is related to their location and function. Renewal of connective and muscle tissue takes place at a much slower pace, whereas nervous tissue is incapable of postnatal regeneration.

An elderly client asks her health care provider if the reason she has developed aortic stenosis is because she drank so much milk as a child growing up on a farm. Which of the following responses is most accurate?

"Atherosclerosis is a long process that eventually results in calcification of heart valves."

A 78-year-old female client has been scheduled for outpatient cataract surgery. While taking a presurgery history, which statement by the client correlates to the surgical procedure?

"I have blurred vision in both my eyes and my visual is distorted."

A neighbor is complaining to a friend (who happens to be a nurse) about several changes in their body. Which of the following complaints raises a "red flag" because it could be a sign of epithelial cell bladder cancer?

"I noticed my urine is pinkish red, but I'm not having any pain when I pee."

An adult client has been diagnosed with polycystic kidney disease. Which of the client's following statements demonstrates an accurate understanding of this diagnosis?

"I suppose I should be tested to see if my children might inherit this."

A 23-year-old female client has been diagnosed with von Willebrand disease following a long history of "heavy periods" and occasional nosebleeds. Which of the client's following statements demonstrates a sound understanding of her new diagnosis?

"I'll make sure to take Tylenol instead of aspirin when I get aches and pains."

A 51-year-old man has been diagnosed with chronic bronchitis after a long history of recurrent coughing. Which of the man's following statements demonstrates a sound understanding of his new diagnosis?

"If I had quit smoking earlier than I did, I think I could have avoided getting bronchitis."

A student asks the instructor what it means when the book states, skeletal muscles are syncytial or multinucleated? The instructor responds:

"If a skeletal muscle is injured and a portion dies, the adjacent sections of that same skeletal muscle fiber do not die because they have their own nuclear material."

When working with a client who has end-stage renal disease (ESRD) and is receiving peritoneal dialysis, the concept of diffusion can be explained by which of the following statements?

"If your potassium level is high, then K+ particles will move from your peritoneal cavity into the dialysis solution, where the concentration of K+ is lower."

When working with a client who has end-stage renal disease (ESRD) and is receiving peritoneal dialysis, the concept of diffusion can be explained by which statement?

"If your potassium level is high, then K+ particles will move from your peritoneal cavity into the dialysis solution, where the concentration of K+ is lower."

A client who had a pulmonary embolism is receiving IV heparin and has just begun taking his first dose of warfarin (Coumadin). The client asks the nurse, "How long will this pill take in order to prevent me from developing more clots. I would like to go home soon." The nurse responds:

"It usually takes 2 to 3 days for warfarin to become therapeutic, meaning your blood will be thin enough to prevent further clot formation."

A woman who has just learned that she is pregnant for the first time has sought advice from her healthcare provider about the safe use of alcohol during pregnancy. What advice should the clinician provide to the woman?

"It's likely best to eliminate alcohol from your diet while you're pregnant"

Which of the following patient complaints should prompt a clinician to order a diagnostic work-up for multiple myeloma?

"Lately my bones just seem to ache so bad, and nothing seems to help."

A homeless client asks, "Why can't I get this wound on my foot to heal?" Knowing that the client is not receiving good nutrition on a regular basis, the nurse will reply:

"Right now your immune system is decreased because you are not eating a balanced diet."

A client with heart failure asks, "Why am I taking a 'water pill' when it's my heart that is having a problem?" While educating the client about the Frank-Starling mechanism, which of the following explanations is most appropriate to share?

"Since your heart is not pumping efficiently, the kidneys are getting less blood flow; therefore, the kidneys are holding on to sodium and water."

A child has been experiencing hypoglycemic episodes. "How does the body know when to secrete insulin and when to stop secreting it?" The best response by the nurse, explaining the physiologic background, would be:

"The body knows that if the blood glucose level falls, it will inhibit insulin secretion and release glycogen to release glucose from the liver."

In anatomy class, the instructor asks, "Explain how urine is expelled from the bladder during voiding." The student with the most accurate response would be:

"The detrusor muscle contract down on the urine and the ureteral orifices are forced shut. The external sphincter relaxes as urine moves out of the bladder."

A client asks, "Why do I keep getting these cold sores on my lips? Am I kissing people too much?" The nurse will base her response by saying:

"These sores usually develop when you are not sleeping well or are emotionally upset."

A couple has just learned that their newborn infant has been diagnosed with osteogenesis imperfecta, and they have responded by seeking out as much information as possible about their child's diagnosis. What should the clinician teach the couple about their child's health problem?

"This is something that your child may have inherited from one or both of you."

A heart failure client has an echocardiogram performed revealing an ejection fraction (EF) of 40%. The nurse knows this EF is below normal and explains to the client:

"This means your heart is not pumping as much blood out of the heart with each beat."

Which of the following dietary guidelines should a nurse provide to a group of older adults to possibly decrease their risks of developing colon cancer?

"Try to minimize fat and maximize fiber when you're planning your meals"

Which of the following dietary guidelines should a nurse provide to a group of older adults to possibly decrease their risks of developing colon cancer?

"Try to minimize fat and maximize fiber when you're planning your meals."

An elderly client newly diagnosed with systolic hypertension asks her health care provider why this happens. The most accurate response would be:

"With age, your arteries lose their elasticity and are replaced with collagen, which makes your arteries stiffer."

Upon admission assessment, the nurse hears a murmur located at the fifth intercostal space, midclavicular line. The client asks, "What does that mean?" The nurse will base her answer on which of the following physiologic principles?

"You have a heart valve that is diseased."

A client recently diagnosed with leukocyte adhesion deficiency (LAD) ask, "Why am I always sick with an infection?" Which response by the nurse explains this rare autosomal recessive disorder?

"Your white blood cells are not able to leave the blood vessels and move into the area of infection."

Which wound will most likely heal from secondary intention?

"road rash"

Which of the following clients have a pathophysiologic process capable of causing fever by inducing the production of pyrogens? Select all that apply.

-A farmer who cut his arm while sharpening his tools coming to clinic because of acute inflammation signs like fever and redness -An older adult recuperating following a myocardial infarction -A newly diagnosed Hodgkin lymphoma client

Which of the following scenarios place the client at a high risk for developing hypoparathyroidism and require close supervision for assessing for development of muscle cramps, carpopedal spasm, convulsions, and paresthesia in the hands and feet? Select all that apply.

-A neck cancer client returning from OR after having a radical neck dissection -A hyperthyroid client experiencing a "thyroid storm" requiring urgent thyroidectomy

Which of the following clients coming to a small free clinic are at high risk for malnutrition? Select all that apply.

-An 88-year-old senior citizen on a fixed budget -A 60-year-old homeless Vietnam veteran complaining of pain -A 4-year-old child who lives with a single mom in a rooming house

Which of the following clients will be more able to adapt to a stressor based on an individual's ability to adapt? Select all that apply.

-An infant with decreased water intake due to diarrhea -A male client producing increased corticotropin-releasing factor (CRF) -A client with known CAD, expressing problems affording all his prescribed medication

A child has been diagnosed with marasmus due to the fact that the parents have both lost their jobs and have very limited funds for food. Which of the following clinical manifestations would the school nurse assess that would confirm this diagnosis? Select all that apply.

-Bradycardia -Stunted growth pattern

A person eating peanuts starts choking and collapses. His airway obstruction is partially cleared, but he remains hypoxic until he reaches the hospital. The health care providers will be assessing this client for which of the following physiological events? Select all that apply.

-Cerebral infarction -Coagulation necrosis

Although stress exposure initiates integrated responses by multiple systems, the functional results are first manifested as: Select all that apply.

-Enhanced respiratory rate/depth -Increased alertness and focus -Increased glucose utilization

Which of the following lab results confirm the client has developed an acute-phase inflammatory response? Select all that apply.

-Erythrocyte sedimentation rate (ESR) 175 mm/h (high). -Leukocytes (WBC) 18.7 cells/L (high). -C-reactive protein (CRP) 10.0 mg/L (high).

Which assessment findings of a client with an elevated temperature would be considered a "normal" finding? Select all that apply.

-Flushed skin -Pain when moving joints to sit up in bed -Unusual fatigue and drowsiness

A client with poor arterial circulation in the lower limbs has developed areas of inflammation and "weeping" clear serous exudate. Since chronic inflammation lasts for a long time, it has been associated with which of the following changes in physiological response? Select all that apply.

-Formation and development of new blood vessels -The death of one or more cells in the body within a localized area

An elderly client is admitted with elevated magnesium level related to a history of renal insufficiency and excess use of antacids and laxatives containing magnesium. On admission assessment, the nurse notes which clinical manifestations that correlate to hypermagnesemia? Select all that apply.

-Hyporeflexia -Muscle weakness causing shallow breathing

A client has received too much morphine (narcotic) in the postsurgical recovery room. Blood gas results reveal the patient has developed respiratory acidosis. Which of the following assessment findings correlate with acute primary respiratory acidosis? Select all that apply.

-Irritability -Muscle twitching -Respiratory depression

While assessing a teenage girl suspected of having bulimia nervosa, the health care provider may find which of the following clinical manifestations that would confirm the diagnosis? Select all that apply.

-Large number of teeth with dental caries -Eroded tooth enamel leading to sensitive teeth -Painless parotid gland enlargement

A client with a history of heart and kidney failure is brought to the emergency department. Upon assessment/diagnosis, it is determined the client is in decompensated heart failure. Of the following assessment findings, which are associated with excess intracellular water? Select all that apply.

-Lethargy -Confusion -Seizures

A heart failure client has gotten confused and took too many of his "water pills" (diuretics). On admission, his serum potassium level was 2.6 mEq/L. Of the following assessments, which correlate to this hypokalemia finding? Select all that apply

-Polyuria -Constipation -Paresthesia with numbness of the lips/mouth

A client has been admitted for deterioration of her renal function due to chronic renal failure. Her admission K+ level is 7.8 mEq/L. The nurse would expect to see which of the following abnormalities on her telemetry (ECG) strip? Select all that apply.

-Prolonged PR interval with widening of the QRS complex -Ventricular fibrillation

The physiology instructor asks the students, "How does growth factor affect cell proliferation?" Which students have the most accurate responses? Growth factors: Select all that apply.

-Regulate the inflammatory process -Act as a chemoattractant for many cells like neutrophils and macrophages -Stimulate new blood vessel growth

A client has been brought to the emergency department with a large, gaping wound from a farming accident. The client is critically ill and has required blood products and surgery to clean and close the wound. Sharing with a student nurse the steps in wound healing, the nurse discusses the inflammatory phase, stating macrophages: Select all that apply.

-Release growth factors that stimulate epithelial cell growth -Facilitate the body's ability to grow new vessels in the injured area -Remove debris from the wound

The nurse notes the client has developed a systemic response of inflammation based on assessment findings. Which of the following clinical manifestations support this diagnosis? Select all that apply.

-Temperature of 100.9°F, lethargy -Pulse rate 130 beats/minute (high) -Generalized achiness

................................

...................................

Which of the following processes associated with cellular injury is most likely to be reversible?

.Cell damage resulting from accumulation of fat in the cytoplasm

When educating a client with possible glucocorticoid dysfunction, the nurse will explain that the CRH controls the release of ACTH. The best time to perform the blood test to measure peak ACTH levels would be:

06:00 to 08:00 AM

A client has experienced an acute inflammatory response with an elevation of white blood cells. The nurse is reviewing the client's most recent lab results to determine if the counts have returned to a normal range. Select the result that suggests the client is now within normal range.

4000 to 10,000 cells/μL

A client states, "I heard that my healed wound tissue is stronger than my normal tissue. Is that true?" The nurse responds that roughly 3 months after a wound; the wound tensile strength is approximately what percentage from normal?

70% of normal

How strong is a healed wound. The wound tensile strength is what percentage from normal?

70% of normal

A client had a positive Pap smear. The surgeon diagnosed "cancer in situ of the cervix." The client asks, "What does this mean?" From the following statements, which is most appropriate in response to this question? The tumor has: A. Not crossed the basement membrane, so it can be surgically removed with little chance of growing back B. Been walled off within a strong fibrous capsule C. Developed a distant infiltration D. Grown undifferentiated cells that no longer look like the tissue from which it arose

A

A diabetic client has injured his foot while walking barefoot on the lawn. On admission, which of the following assessment findings would be considered a localized cardinal sign of acute inflammation? A. Redness and edema at the injured site B. Urine output of less than 500 mL/24 hours (low) C. Temperature of 101°F D. Fatigue with listlessness

A

During a lecture on inflammation, the physiology instructor discusses the major cellular components involved in the inflammation response. The instructor asks, "Which of the following cells arrives early in great numbers?" The student with the correct response is: A. Neutrophils B. Monocytes C. Lymphocytes D. Basophils

A

Which of the following individuals likely faces the highest risk of megaloblastic anemis?

A 21 year old college student who lives a vegan lifestyle

"Inflammation helps your body to produce the right antibodies to fight the infection."

A 23-year-old man has received a recent diagnosis of appendicitis. The nurse providing care for the man is explaining that the inflammation of his appendix is playing a role in his body's fight against the underlying infectious process. Which teaching points should the nurse eliminate from client education?

Which of the following clients would be at high risk for developing primary varicose veins? Select all that apply.

A 47-year-old waitress who works 12-hour shifts three or four times/week; A morbidly obese (>100 pounds overweight) male who works behind the counter of a convenience store 10 hours/day, 5 days/week

"This is what we call a type I hypersensitivity reaction and usually occurs a few minutes after exposure to his allergen. It is primarily caused by mast cells in his body."

A 5-year-old child is experiencing itchy, watery eyes and an increased respiratory rate with some inspiratory wheezes. He has been outside playing in the yard and trees. The mother asks, "Why does he get like this?" The health care worker's best response is:

Which of the following individuals most likely faces the greatest risk of developing Clostridium difficile colitis?

A 79-year-old hospitalized client who is being treated with broad-spectrum antibiotics

Which of the following wounds is most likely to heal by secondary intention?

A boy's "road rash" that he got by falling off his bicycle

which symptom indicates the next stage of a fever after the prodrome stage

A chill

Fibrous tissue

A client asks the nurse why a scar developed after an injury. Which response by the nurse is most accurate?

Innate immunity is the body's first line of defense. Redness, heat, pain and swelling are part of the normal inflammatory process. The epithelial layers of the skin provide innate immunity. (3)

A client comes to the emergency room concerned about infection after being cut on the hand by a piece of glass; the skin surrounding the wound is red, warm and painful. After assisting with suture insertion, the nurse develops discharge instructions to include which information on the body's natural defenses? Select all that apply.

"You have developed a secondary immunodeficiency disorder as a result of your chemotherapy."

A client develops an immunodeficiency disorder after receiving chemotherapy for the treatment of lung cancer. The client asks the nurse if he was born with this deficiency. What is the nurse's best response?

What client is most likely to experience during impaired wound healing?

A client diagnosed with type 1 diabetes and a history of poor blood sugar regulation

Your immune system is attacking your own tissues as if they were harmful organisms.

A client has been diagnosed with rheumatoid arthritis and asks the nurse what causes this condition. What is the nurse's best response?

Phagocytosis

A client has been exposed to bacteria. Which term best describes the body's initiate ability to consume the foreign substances?

Specificity and memory of the immune response

A client has been identified as having an excess of macrophage inhibitory factor, causing the client to have inhibited movement and activity of macrophages. Which process should the health care team expect to remain unaffected?

Redness

A client in the acute stage of inflammation will experience vasodilation of the arterioles and congestion in the capillary beds. The nurse would assess the client's skin for:

Vasodilation of the capillaries from the release of histamine

A client in the primary or initial phase of a type I hypersensitivity reaction would most likely experience which physiologic response?

Antibody titer

A client is applying to nursing school and has come to the clinic with a request to be tested for immunity to hepatitis B. Which type of testing would be best to determine immunity?

Active natural immunity

A client is brought to the physician's office with a raised, red macular rash on the trunk and arms accompanied by a fever. A diagnosis of measles is made. Which type of immunity does this disease process provide?

Detection of antigens Culture Serology

A client is suspected of having a bacterial disease. Which basic laboratory techniques can be used to make a definitive diagnosis of the infectious agent? Select all that apply. (3)

"Not all interactions between microorganisms and humans are detrimental."

A client is very concerned about the harmful effect that all microorganisms may have. The best response by the nurse would be:

Which of the following clients is most likely to experience impaired wound healing?

A client with a diagnosis of type 1 diabetes and a history of poor blood sugar control

Which of the following individuals likely faces the greatest risk for the development of chronic kidney disease?

A client with a recent diagnosis of type 2 diabetes who does not monitor his blood sugars or control his diet

Histamine

A client with environmental allergies is experiencing respiratory inflammation. Which mediator causes vasodilation during the vascular stage of the inflammatory response?

Which of the following clients are at risk for developing blood clots and should be assessed frequently? Select all that apply.

A college-aged female taking oral contraceptives for irregular menstrual periods; A middle-aged male who smokes three packs of cigarettes/day; A diabetic client who is also greater than 100 pounds over the ideal body weight

Recovery of probable pathogen

A cytotechnologist is examining a blood sample. Which must be identified for a diagnosis of infectious disease?

While explaining to a post-surgical knee client about the various forms and function of connective tissue, the nurse gives the example of the client's surgical repair of a torn anterior cruciate ligament (ACL), which is due to:

A dense regular connective tissue tear that is usually rich in collagen fibers that allows ligaments to join bone to bone

Apply a topical corticosteroid and avoid further contact with latex-containing products

A dental assistant comes to the clinic with reports of a rash on her hands that feel like they are coming from the gloves being worn. The nurse observes both hands with crusted and thickened areas. What type of education does the nurse anticipate providing to the client?

Redness and edema at the injured site

A diabetic client has injured his foot while walking barefoot on the lawn. On admission, which assessment finding would be considered a localized cardinal sign of acute inflammation?

Which of the following scenarios place the client at a high risk for developing hypoparathyroidism and require close supervision for assessing for development of muscle cramps, carpopedal spasm, convulsions, and paresthesia in the hands and feet? Select all that apply.

A hyperthyroid client experiencing a "thyroid storm" requiring urgent thyroidectomy; A neck cancer client returning from OR after having a radical neck dissection

Of the following clients, which would be at highest risk for developing hyperkalemia?

A male admitted for acute renal failure following a drug overdose

Which of the following individuals is experiencing a health problem that is the result of a parasite?

A man who acquired malaria while on a tropical vacation

Which of the following individuals is most clearly in need of diagnostic testing for lung cancer?

A man who demonstrates wasting of the pelvic and shoulder muscles combined with signs of hypercalcemia

Which of the following clients may be experiencing the effects of neuropathic pain?

A man with pain secondary to his poorly controlled diabetes

A secondary response causes a sharp rise in antibody levels

A middle school student is scheduled to receive booster immunizations and the father asks the nurse why the booster is necessary. What characteristic of the adaptive immune system listed below would provide the rationale for the nurse's response?

Which client is an example of cellular atrophy?

A middle- aged female experiencing menopause due to loss of estrogen stimulation.

Which of the following clients would be an example of cellular atrophy?

A middle-aged female experiencing menopause due to loss of estrogen stimulation

"An infant gets immunity from the mother at birth and from breast milk; this provides protection from infection but wears off in approximately 3 to 6 months. An infant still needs time for his own immune system to mature."

A mother brings her 5-month-old infant to the pediatrician for recurrent colds. The mother has never breast-fed and the infant is fed iron-fortified formula. The mother asks, "My baby has been perfectly healthy up until last month—now it seems like she has been sick constantly. Why?" How should the physician respond?

Rubor, swelling, and pain

A nurse is assessing a client for the classic signs of acute inflammation. The nurse would assess the client for:

"Bacterial resistance to antibiotics can occur due to genetic mutations."

A nurse is instructing a client on the long-term use of antibiotics and antibiotic resistance. Which statement by the client indicates that the teaching has been successful?

Active immunity is long-lasting and can improve on repeated exposures to an antigen. Active immunity can be acquired through the process of vaccination or from environmental exposure to an antigen. Active immunity allows the host's own immune system to develop an immunologic response.

A nurse is planning to teach a parent group about the spread of infection, the importance of vaccines, and active immunity. Which statements should be included in the teaching plan? Select all that apply. (3)

Which of the following patients likely faces the highest risk of an acquired hypocoagulation disorder vitamin K deficiency?

A patient who has a diagnosis of liver failure secondary to alcohol abuse

Which of the following individuals is experiencing the effects of a primary endocrine disorder?

A patient who has low calcium levels because of the loss of his parathyroid gland

Which of the following is most likely to experience impaired wound healing?

A patient with a diagnosis of type 1 diabetes and a history of poor blood sugar control

Memory T cells

A person has been exposed to a particular antigen and a now experiences a repeat exposure. What stimulates a quicker immune response?

An occupational therapist is preparing to begin a relaxation program on an oncology unit of a hospital. Which of the following variables will assist in the success of the program?

A quiet, calm, and therapeutic environment

A client is extremely anxious about his impending surgery. Which of the following measures should the nurse implement to create an atmosphere for effective use of relaxation techniques?

A quiet, dim environment

Two people experience the same stressor, yet only one is able to cope and adapt adequately. An example of the person with an increased capacity to adapt is the one with:

A sense of purpose in life

"These little hairs move germs trapped in mucus toward the throat so the body can cough them out."

A sixth-grade science teacher asks the students to explain the role of cilia in the lower respiratory tract. Which student response is best?

Which of the following processes characterizes an epigenetic contribution to oncogenesis?

A tumor suppressor gene is present, but it is not expressed.

Destroying one type of resident flora (bacteria) can allow overproliferation of another competing type (yeast).

A woman reports to the nurse that she has developed a yeast infection. The woman does not understand how she could get a yeast infection since she has been on antibiotics for a urinary tract infection. What is the rationale for this client's complaint?

Which of the following individuals most likely faces the highest risk of developing chronic pancreatitis?

A woman who has 6-8 alcoholic beverages an evening

Which of the following patients is most likely to be in positive nitrogen balance?

A woman who has been admitted to the hospital in early labor

A 40-year-old female has been diagnosed with hepatocellular carcinoma. When asked what causes this cancer, the physician looks at the history and responds, "I see in your history, you have had: A. Hepatitis B related to IV drug use as a teenager." B. Your uterus and ovaries removed for endometriosis." C. Numerous admissions for COPD exacerbations requiring steroids." D. Three miscarriages with no live births."

A)

During lecture on wound care, the instructor mentions the final stage of the cellular response of acute inflammation. Of the following statements, which describes what physiologically occurs in the final stage? A. Neutrophils, monocytes, and macrophages engulf and degrade the bacteria/cellular debris. B. Chemokines direct the trafficking of leukocytes. C. Leukocytes accumulate and begin migration to the site of injury. D. Mediators are transformed into inactive metabolites.

A)

Which of the following clients would be an example of cellular atrophy? A) A middle-aged female experiencing menopause due to loss of estrogen stimulation B) A postnephrectomy client whose remaining kidney enlarges to compensate for the loss C) A hypertensive, noncompliant client who has developed a progressive increase in left ventricular mass D) A female client with the change in uterine size as a result of pregnancy

A) A middle-aged female experiencing menopause due to loss of estrogen stimulation Feedback: In women, the loss of estrogen stimulation during menopause results in atrophic changes in the reproductive organs. Compensatory hypertrophy is the enlargement of a remaining organ or tissue after a portion has been surgically removed or rendered inactive. For instance, if one kidney is removed, the remaining kidney enlarges to compensate for the loss. In hypertension, for example, the increased workload required to pump blood against an elevated arterial pressure results in a progressive increase in left ventricular muscle mass and need for coronary blood flow. The pregnant uterus undergoes both hypertrophy and hyperplasia as a result of estrogen stimulation.

17. Which of the following processes associated with cellular injury is most likely to be reversible? A) Cell damage resulting from accumulation of fat in the cytoplasm B) Cellular changes as a result of ionizing radiation C) Cell damage from accumulation of free radicals D) Apoptosis

A) Cell damage resulting from accumulation of fat in cytoplasm.

19. Which of the following processes associated with cellular injury is most likely to be reversible? A) Cell damage resulting from accumulation of fat in the cytoplasm B) Cellular changes as a result of ionizing radiation C) Cell damage resulting from accumulation of free radicals D) Apoptosis

A) Cell damage resulting from accumulation of fat in the cytoplasm

Which of the following processes associated with cellular injury is most likely to be reversible? A) Cell damage resulting from accumulation of fat in the cytoplasm B) Cellular changes as a result of ionizing radiation C) Cell damage resulting from accumulation of free radicals D) Apoptosis

A) Cell damage resulting from accumulation of fat in the cytoplasm Feedback: Intracellular accumulation of fat leads to serious cell damage, but this is a potentially reversible effect. Ionizing radiation and damage from free radicals are more likely to be permanent, whereas apoptosis is defined as the permanent removal of injured and aged cells.

A person eating peanuts starts choking and collapses. His airway obstruction is partially cleared, but he remains hypoxic until he reaches the hospital. The health care providers will be assessing this client for which of the following physiological events? Select all that apply. A) Cerebral infarction B) Coagulation necrosis C) Rapid phagocytosis D) Protein p53 deficiency

A) Cerebral infarction B) Coagulation necrosis

A client presents for a scheduled Papanicolaou (Pap) smear. The clinician who will interpret the smear will examine cell samples for evidence of: A) Changes in cell shape, size, and organization B) The presence of unexpected cell types C) Ischemic changes in cell samples D) Abnormally high numbers of cells in a specified field

A) Changes in cell shape, size, and organization Feedback: A Pap smear is an example of a diagnostic procedure that tests for the presence of cell dysplasia, that is, deranged cell growth of a specific tissue that results in cells that vary in size, shape, and organization. Unexpected cell types are evidence of metaplasia, whereas ischemic changes are associated with cell hypertrophy. Increases in the number of cells are characterized as hyperplasia.

14. Free radicals damage cells by: A) Destroying phospholipids in the cell membrane B) Altering the immune response of the cell C) Disrupting calcium storage in the cell D) Inactivation of enzymes and mitochondria

A) Destroying phospholipids in the cell membrane

Free radicals damage cells by: A) Destroying phospholipids in the cell membrane B) Altering the immune response of the cell C) Disrupting calcium storage in the cell D) Inactivation of enzymes and mitochondria

A) Destroying phospholipids in the cell membrane Feedback: Free radicals are highly reactive and can damage cells in several ways. One way is by destroying lipids, which results in a loss of cell membrane integrity. Free radicals modify proteins but do not affect the immune function, calcium storage, or intracellular enzymes of cells.

18. Ischemia and other toxic injuries increase the accumulation of intracellular calcium as a result of: A) Release of stored calcium from the mitochondria B) Improved intracellular volume regulation C) Decreased influx across the cell membrane D) Attraction of calcium to fatty infiltrates

A) Release of stored calcium from the mitochondria

Ischemia and other toxic injuries increase the accumulation of intracellular calcium as a result of: A) Release of stored calcium from the mitochondria B) Improved intracellular volume regulation C) Decreased influx across the cell membrane D) Attraction of calcium to fatty infiltrates

A) Release of stored calcium from the mitochondria Feedback: Disruption of the normal intracellular functions causes the impaired mitochondria to release stored calcium. Cell injury disrupts intracellular volume regulation, allowing excessive influx of substances across the impaired cell membrane. Fatty infiltrate is an ominous sign of cell damage and does not interact with calcium.

A pregnant client is attending a nutrition class for first-time moms. During the class, the instructor stressed that they should avoid consumption of which food that may cause brain damage from methyl mercury exposure? A) Tuna B) Raw hamburger C) Fresh milk D) Beets

A) Tuna Feedback: The main source of methyl mercury exposure is from consumption of long-lived fish, such as tuna and swordfish. Fish concentrate mercury from sediment in the water. Because the developing brain is more susceptible to mercury-induced damage, it is recommended that young children and pregnant and nursing women should avoid consumption of fish known to contain high mercury content. None of the other foods listed pose a threat of mercury toxicity.

12. A 20-year-old college student has presented to her campus medical clinic for a scheduled Papanicolaou (Pap) smear. The clinician who will interpret the smear will examine cell samples for evidence of: A) changes in cell shape, size, and organization. B) the presence of unexpected cell types. C) ischemic changes in cell samples. D) abnormally high numbers of cells in a specified field.

A) changes in cell shape, size, and organization.

10. A 99-year-old woman has experienced the decline of cell function associated with age. A group of theories of cellular aging focus on programmed: A) changes with genetic influences. B) elimination of cell receptor sites. C) insufficient telomerase enzyme. D) DNA mutation or faulty repair.

A) changes with genetic influences

5. The patient has a prolonged interruption in arterial blood flow to his left kidney, causing hypoxic cell injury and the release of free radicals. Free radicals damage cells by: A) destroying phospholipids in the cell membrane. B) altering the immune response of the cell. C) disrupting calcium storage in the cell. D) inactivation of enzymes and mitochondria.

A) destroying phospholipids in the cell membrane

1. Ischemia and other toxic injuries increase the accumulation of intracellular calcium as a result of: A) release of stored calcium from the mitochondria. B) improved intracellular volume regulation. C) decreased influx across the cell membrane. D) attraction of calcium to fatty infiltrates.

A) release of stored calcium from the mithochondria

A person eating peanuts starts choking and collapses. His airway obstruction is partially cleared, but he remains hypoxic until he reaches the hospital. The health care providers will be assessing this client for which of the following physiological events? Select all that apply. A) Cerebral infarction B) Coagulation necrosis C) Rapid phagocytosis D) Protein p53 deficiency

A)Cerebral infarction B)Coagulation necrosis Feedback: Tissue infarction is caused by prolonged oxygen deprivation, and the resulting large group of dead cells coagulates in the damaged area. During coagulation necrosis, acidosis develops and denatures the enzymatic and structural proteins of the cell. Phagocytosis occurs rapidly during apoptosis, so it does not elicit an inflammatory response. Protein p53 activation initiates apoptosis.

A client with a history of chronic obstructive pulmonary disease (COPD) is undergoing pulmonary function testing. Which of the following instructions should the technician provide in order to determine the client's forced vital capacity (FVC)? A. "I'll ask you to breathe in as deep as you can, and then blow out as much of that air as possible. B. "I'd like you to take a deep breath, and then blow out as much air as you can during 1 second. C. "I want you to breathe as normally as possible, and I'm going to measure how much air goes in and out with each breath. D. "Breathe normally, and then exhale as much as you possibly can when I tell you.

A. "I'll ask you to breathe in as deep as you can, and then blow out as much of that air as possible.

The nurse would anticipate that which of the following clients would be considered a good candidate for coronary artery bypass grafting (CABG)? A. A 56-year-old with a history of MI experiencing new-onset chest pain and ST elevation B. A 24-year-old auto accident client diagnosed with pericardial effusion and cardiac tamponade heart cannot beat due to too much fluid in pericardial sac) C. A 87-year-old client admitted with uncontrolled dilated cardiomyopathy- weak heart muscle D. A 78-year-old client admitted with increasing fatigue related to aortic valve failure

A. A 56-year-old with a history of MI experiencing new-onset chest pain and ST elevation

Which of the following clients would be an example of cellular atrophy? A. A middle-aged female experiencing menopause due to loss of estrogen stimulation B. A postnephrectomy client whose remaining kidney enlarges to compensate for the loss C. A hypertensive, noncompliant client who has developed a progressive increase in left ventricular mass D. A female client with the change in uterine size as a result of pregnancy

A. A middle-aged female experiencing menopause due to loss of estrogen stimulation

Which of the following clients would be an example of cellular atrophy? A. A middle-aged female experiencing menopause due to loss of estrogen stimulation B. A postnephrectomy client whose remaining kidney enlarges to compensate for the loss C. A hypertensive, noncompliant client who has developed a progressive increase in left ventricular mass D. A female client with the change in uterine size as a result of pregnancy

A. A middle-aged female experiencing menopause due to loss of estrogen stimulation

Which of the following clients would be an example of cellular atrophy? Select one: A. A middle-aged female experiencing menopause due to loss of estrogen stimulation B. A hypertensive, noncompliant client who has developed a progressive increase in left ventricular mass C. A postnephrectomy client whose remaining kidney enlarges to compensate for the loss D. A female client with the change in uterine size as a result of pregnancy

A. A middle-aged female experiencing menopause due to loss of estrogen stimulation

Which is a description of the characteristics of apoptosis? A. A programmed cell death of scattered, single cells B. Characterized by swelling of the nucleus and cytoplasm C. Has unpredictable patterns of cell death D. Results in benign malignancies

A. A programmed cell death of scattered, single cells

An IV drug abuser has been diagnosed with infective endocarditis. He is in the emergency department reporting increasing shortness of breath, rapid breathing, chest pain that worsens with breathing, and coughing up blood. The health care provider recognizes this may be caused by: A. Blood clots (emboli) traveling in the blood stream to the lungs B. Infected blood clots in the left ventricle traveling through the aorta C. Microemboli being developed in the carotids by Staphylococcus epidermidis D. Infarction of the tissue surrounding the endocardium of the heart

A. Blood clots (emboli) traveling in the blood stream to the lungs

A client has entered hypovolemic shock after massive blood loss in a car accident. Many of the client's peripheral blood vessels have consequently collapsed. How does the Laplace law account for this pathophysiologic phenomenon? A. Blood pressure is no longer able to overcome vessel wall tension. B. Decreasing vessel radii has caused a decrease in blood pressure. C. Wall thickness of small vessels has decreased due to hypotension. D. Decreases in wall tension and blood pressure have caused a sudden increase in vessel radii.

A. Blood pressure is no longer able to overcome vessel wall tension.

Although the basic structure of the cell plasma membrane is formed by a lipid bilayer, most of the specific membrane functions are carried out by: A. Bound and transmembrane proteins B. Complex, long carbohydrate chains C. Surface antigens and hormone receptors D. A gating system of selective ion channels

A. Bound and transmembrane proteins

Although the basic structure of the cell plasma membrane is formed by a lipid bilayer, most of the specific membrane functions are carried out by: A. Bound and transmembrane proteins B. Complex, long carbohydrate chains C. Surface antigens and hormone receptors D. A gating system of selective ion channels

A. Bound and transmembrane proteins

Genetic screening may be indicated for individuals who have a family history of which of the following neoplasms? Select one: A. Breast cancer B. Leukemia C. Liver cancer D. Multiple myeloma

A. Breast cancer

Free radicals play a major role in the initiation and progression of which diseases? A. Cardiovascular diseases such as hypertension and ischemic heart disease B. Renal diseases such as acute tubular necrosis and glomerulonephritis C. Gastrointestinal diseases such as peptic ulcer disease and Crohn disease D. Muscular disease such as muscular dystrophy and fibromyalgia

A. Cardiovascular diseases such as hypertension and ischemic heart disease

Which of the following processes associated with cellular injury is most likely to be reversible? A. Cell damage resulting from accumulation of fat in the cytoplasm B. Cellular changes as a result of ionizing radiation C. Cell damage resulting from accumulation of free radicals D. Apoptosis

A. Cell damage resulting from accumulation of fat in the cytoplasm

A client presents for a scheduled Papanicolaou (Pap) smear. The clinician who will interpret the smear will examine cell samples for evidence of: A. Changes in cell shape, size, and organization B. The presence of unexpected cell types C. Ischemic changes in cell samples D. Abnormally high numbers of cells in a specified field

A. Changes in cell shape, size, and organization

The process responsible for generating and conducting membrane potentials is: A. Diffusion of current-carrying ions B. Milliviltage of electric potential C. Polarization of charged particles D. Ion channel neurotransmission

A. Diffusion of current-carrying ions

Which of the following variables determine the extent of teratogenic drug effects? Select all that apply.

A. Duration of drug exposure B. Stage of pregnancy when the drug was taken C. Molecular weight of the drug

Clients with ischemic coronary vessel disease and acute coronary syndrome (ACS) are classified as low or high risk for acute myocardial infarction based on characteristics that include significant: A. ECG changes B. Heart murmurs C. Pulmonary disease D. Pericardial effusion

A. ECG changes

*Match the structure with its function. (Answers A through E may be used more than once.) A. Endoplasmic reticulum B. Ribosome C. Golgi complex D. Secretory vesicle E. Lysosomes 1. Packages and transports proteins 2. Fuses with the plasma membrane to release contents from the cell 3. Synthesizes and transports lipids 4. Provides energy to digest proteins into amino acids

A. Endoplasmic reticulum D. Secretory vesicle A. Endoplasmic reticulum E. Lysosomes

What is a consequence of leakage of lysosomal enzymes during chemical injury? A. Enzymatic digestion of the nucleus and nucleolus occurs halting DNA synthesis. B. Influx of potassium ions into the mitochondria occurs halting the ATP production. C. Edema of the Golgi body occurs preventing the transport of proteins out of the cell. D. Shift of calcium out of the plasma membrane occurs destroying the cytoskeleton.

A. Enzymatic digestion of the nucleus and nucleolus occurs halting DNA synthesis

A male client with a diagnosis of type 1 diabetes mellitus is experiencing hyperglycemia because he lacks sufficient insulin to increase the availability of glucose transporters in his cell membranes. Consequently, his cells lack intracellular glucose, and it accumulates in his blood. Which of the following processes would best allow glucose to cross his cell membranes? A. Facilitated diffusion B. Simple diffusion C. Secondary active transport D. Endocytosis

A. Facilitated diffusion

A male patient with diagnosis of type 1 diabetes mellitus is experiencing hyperglycemia because he lacks sufficient insulin to increase the availability of glucose transporters in his cell membranes. Consequently, his cells lack intracellular glucose and it accumulates in his blood. Which of the following processes would best allow glucose to cross his cell membranes? A. Facilitated diffusion B. Simple diffusion C. Secondary active transport D. Endocytosis

A. Facilitated diffusion

Although the muscle tissue cells have some similarities, smooth muscle (also known as involuntary muscle) differs by: A. Having dense bodies attached to actin filaments B. Containing sarcomeres between Z lines and M bands C. Having rapid contractions and abundant cross-striations D. Contracting in response to increased intracellular calcium

A. Having dense bodies attached to actin filaments

Which of the following would be considered a major cause of secondary hyperlipoproteinemia since it increases the production of VLDL and conversion to LDL? A. High-calorie diet B. Diabetes mellitus C. Bile-binding resin D. Cholesterol ingestion

A. High-calorie diet

The movement of fluid across the arterial end of capillary membranes into the interstitial fluid surrounding the capillary is an example of which process of fluid movement? A. Hydrostatic pressure B. Osmosis C. Diffusion D. Active transport

A. Hydrostatic pressure

A client has just been diagnosed with Graves disease. The primary care provider tells the client that this is due to the presence of autoantibodies to the TSH receptor. The client asks the nurse to explain this in simpler terms. Which response would be the best for this client? A. Hyperthyroidism is basically an autoimmune disorder. B. "Basically your lymph node system has gone haywire. C. "Your body has loss its ability to delete autoreactive T cells in the thyroid. D. The filtration system in your body is no longer working.

A. Hyperthyroidism is basically an autoimmune disorder.

What is the single most common cause of cellular injury? A. Hypoxic injury B. Chemical injury C. Infectious injury D. Genetic injury

A. Hypoxic Injury

Paraneoplastic syndromes are manifestations of cancer that often result from: Select one: A. Inappropriate hormone release B. Radiation and chemotherapy C. Compression of area vessels D. Tumor-related tissue necrosis

A. Inappropriate hormone release

A farmer's long-term exposure to pesticides has made the cells in his alveoli and bronchial tree susceptible to malignancy. Which of the following processes has taken place in the farmer's lungs? Select one: A. Initiation B. Promotion C. Differentiation D. Progression

A. Initiation

How is passive transport described? A. It is driven by osmosis, filtration, and diffusion. B. It involves receptors that can bind with substances being transported. C. It is capable of transporting macromolecules. D. It requires energy generated by the cell.

A. It is driven by osmosis, filtration, and diffusion

What are the major chemical components of the cell membranes? A. Lipids and proteins B. Sodium and potassium ions C. Carbohydrates and water D. DNA and RNA

A. Lipids and proteins

The cancer with the highest mortality in the US (regardless of race or gender) is: A. Lung B. Breast C. Colon D. Prostate

A. Lung

Breakdown and removal of foreign substances and worn-out cell parts are performed by which of the following organelles? A. Lysosomes B. Golgi apparatus C. Ribosomes D. Endoplasmic reticulum (ER)

A. Lysosomes

What occurs during exocytosis? A. Macromolecules can be secreted across eukaryotic cell membranes. B. All substances are secreted into the cellular matrix. C. No repairs in the plasma membrane can take place. D. Solute molecules flow freely into and out of the cell.

A. Macromolecules can be secreted across eukaryotic cell membranes

The US Preventive Services Task Force: A. Makes recommendations based on rigorous review of peer-reviewed evidence B. Conducts research C. Evaluates value of services regardless of benefits or harms of each service D. Is a non-independent panel of experts

A. Makes recommendations based on rigorous review of peer-reviewed evidence

Innate immunity, also called natural or native immunity, consists of mechanisms that respond specifically to: A. Microbes B. Self cells C. Antibodies D. Inflammation

A. Microbes

Crossing-over of chromatid segments during meiosis division 1 results in: Select one: A. New gene combinations B. Initial DNA synthesis C. Spontaneous gene mutations D. Bivalent X and Y genes

A. New gene combinations

Aneuploidy of the X chromosome can result in a monosomy or polysomy disorder. The clinical manifestations of a female with monosomy X include: Select all that apply.

A. Nonpitting lymphedema of the feet C. Difficulty with fine motor skills E. A short-stature female individual

A public health nurse has cited a reduction in cancer risk among the many benefits of maintaining a healthy body mass index. Which of the following facts underlies the relationship between obesity and cancer? A. Obesity can cause inflammation and hormonal changes that are associated with cancer. B. Adipose tissue is more susceptible to malignancy than other types of connective tissue. C. Increased cardiac workload and tissue hypoxia can interfere with normal cell differentiation. D. Increased numbers of body cells increase the statistical chances of neoplastic cell changes.

A. Obesity can cause inflammation and hormonal changes that are associated with cancer.

A drug abuser was found unconscious after shooting up heroin 2 days prior. Because of the pressure placed on the hip and arm, the client has developed rhabdomyolysis. The nurse knows this can: A. Obstruct the renal tubules with myoglobin and damage tubular cells B. Be cured by administering an anticoagulant immediately C. Cause the kidney to develop renal stones due to stasis D. Cause compartment syndrome in the lower extremities

A. Obstruct the renal tubules with myoglobin and damage tubular cells

An older adult client has just sheared the skin on her elbow while attempting to boost herself up in bed, an event that has precipitated acute inflammation in the region surrounding the wound. Which of the following events will occur during the vascular stage of the client's inflammation? A. Outpouring of exudate into interstitial spaces B. Chemotaxis C. Accumulation of leukocytes along the epithelium D. Phagocytosis of cellular debris

A. Outpouring of exudate into interstitial spaces

Which mode of chemical signaling uses local chemical mediators that are quickly taken up, destroyed or immobilized? A. Paracrine signaling B. Autocrine signaling C. Neurotransmitter signaling D. Hormone signaling

A. Paracrine signaling

While lecturing on blood pressure, the nurse will emphasize that the body maintains its blood pressure by adjusting the cardiac output to compensate for changes in which of the following physiologic processes? A. Peripheral vascular resistance B. Electrical impulses in the heart C. Release of stress hormones D. Rigidity of the ventricular walls

A. Peripheral vascular resistance

Which of the following is most likely to precipitate an asthmatic attack in a child with a diagnosis of extrinsic, or atopic, asthma? A. Pet dander B. Cold weather C. Stress D. Respiratory tract infections

A. Pet dander

____________ failure, the most common form of acute renal failure, is characterized by a marked decrease in renal blood flow. A. Pre-renal B. Post-renal (obstructive) C. Glomerular D. Tubular

A. Pre-renal

Ischemia and other toxic injuries increase the accumulation of intracellular calcium as a result of: A. Release of stored calcium from the mitochondria B. Improved intracellular volume regulation C. Decreased influx across the cell membrane D. Attraction of calcium to fatty infiltrates

A. Release of stored calcium from the mitochondria

A client with an 80-pack-year history of tobacco smoking has presented to the clinic complaining of bronchitis cough for the past 5 months, weight loss, and shortness of breath. Today, this client got scared when he coughed up blood in his sputum. The health care provider is concerned this client may have which of the following possible diagnoses? A. Small cell lung cancer due to smoking history B. Tuberculosis due to long period of coughing C. Pulmonary embolism due to blood in sputum D. Pneumothorax related to chronic lung infection weakening the alveoli

A. Small cell lung cancer due to smoking history

A client who lives with angina pectoris has taken a sublingual dose of nitroglycerin to treat the chest pain he experiences while mowing his lawn. This drug facilitates release of nitric oxide, which will have what physiologic effect? A. Smooth muscle relaxation of vessels B. Decreased heart rate and increased stroke volume C. Increased right heart filling D. Reduction of cardiac cycle refractory periods

A. Smooth muscle relaxation of vessels

A student asks, What does cell-mediated immunity mean to the client? The instructor responds. This means: A. The body is trying to defend itself against intracellular microbe invasion by engulfing and destroying the microbe. B. The person's immune system is trying to eliminating all responses to certain antigens. C. The person's immune system is trying to rapidly mature more killer cells to protect against any microbial invasion. D. Our body is trying to systematically control the immune response.

A. The body is trying to defend itself against intracellular microbe invasion by engulfing and destroying the microbe.

What is the sequence of steps in the development of a What is the sequence of steps in the development of a digestive enzyme by the pancreas cells from the initial transcription to the release from the cell? A. The enzyme is transcribed from DNA by RNA in the nucleus, proceeds to the ribosome for synthesis, and is transported in a secretory vesicle to the cell membrane. B. The enzyme is transcribed from RNA by DNA in the nucleus, proceeds to the lysosome for synthesis, and is transported in a encapsulated membrane to the cell membrane. C. The enzyme is transcribed by the mitochondria in the nucleus, proceeds to the ribosome for synthesis, and is transported in a cytoskeleton to the cell membrane. D. The enzyme is transcribed from DNA by RNA in the nucleus, proceeds to the Golgi complex for synthesis, and is transported in a cytosol to the cell membrane.

A. The enzyme is transcribed from DNA by RNA in the nucleus, proceeds to the ribosome for synthesis, and is transported in a secretory vesicle to the cell membrane

Prenatal genetic testing that counts the number of Barr bodies in a chromosome is able to determine: Select one: A. The genetic sex of a child B. Fetal viability C. The presence of fragile X syndrome D. Susceptibility to hemophilia B

A. The genetic sex of a child

What cells turn over (reproduce and replace themselves) most frequently? A. The squamous epithelial cells of the skin B. The connective tissue supporting blood vessels C. The connective tissue supporting blood vessels D. The nervous tissue that constitutes the central nervous system

A. The squamous epithelial cells of the skin

Which of the following body tissues exhibits the highest rate of turnover and renewal? A. The squamous epithelial cells of the skin B. The connective tissue supporting blood vessels C. The skeletal muscle that facilitates movement D. The nervous tissue that constitutes the central nervous system

A. The squamous epithelial cells of the skin

A lung biopsy and magnetic resonance imaging have confirmed the presence of a benign lung tumor in a client. Which of the following characteristics is associated with this client's neoplasm? A. The tumor will grow by expansion and is likely encapsulated. B. The cells that constitute the tumor are undifferentiated, with atypical structure. C. If left untreated, the client's tumor is likely to metastasize. D. The tumor is likely to infiltrate the lung tissue that presently surrounds it.

A. The tumor will grow by expansion and is likely encapsulated.

A 5-year-old child is experiencing itchy, watery eyes and an increased respiratory rate with some inspiratory wheezes. He has been outside playing in the yard and trees. The mother asks, Why does he get like this? The health care worker's best response is: A. This is what we call a type I hypersensitivity reaction and usually occurs a few minutes after exposure to his allergen. It is primarily caused by mast cells in his body. B. Because his allergy is related to something in his environment, the best thing you can do is try to keep him indoors as much as possible. C. This sounds like he is on his way to having an anaphylactic reaction and you need to get a prescription for an EpiPen to decrease his response to monocytes. D. This is pretty common in children. He is just getting used to all the allergens in the air. I suggest you just give him a shower after every time he plays outside.

A. This is what we call a type I hypersensitivity reaction and usually occurs a few minutes after exposure to his allergen. It is primarily caused by mast cells in his body.

A breast cancer client has just learned that her tumor clinical stage is T3, N2, M0. After the physician leaves, the client asks the nurse to explain this to her again. The nurse will use which of the following statements in his or her answer? Your: Select one: A. Tumor is large and at least two lymph nodes are positive for cancer cells. B. Extent of disease is unknown, but it looks like your cancer has stayed intact and not spread to the bloodstream. C. Tumor has metastasized to at least three distal sites and you have cancer in your lymph nodes. D. Tumor is very small and has zero number of mitoses.

A. Tumor is large and at least two lymph nodes are positive for cancer cells.

A pregnant client is attending a nutrition class for first-time moms. During the class, the instructor stressed that they should avoid consumption of which food that may cause brain damage from methyl mercury exposure? A. Tuna B. Raw hamburger C. Fresh milk C. Beets

A. Tuna

A newborn is admitted to the hospital with a high bilirubin level of 13 mg/dL. The assessment data related to this lab value includes: A. Yellowish discoloration of the skin B. Colicky, intermittent pain associated with formula feeding C. Xanthomas D. Clear urine

A. Yellowish discoloration of the skin

A 16-year-old girl has been admitted to the emergency department after ingesting 20 g of acetaminophen (Tylenol) in a suicide attempt. The care team would recognize that this patient faces a severe risk of: A. acute fulminant hepatitis. B. hepatitis D virus infection. C. secondary biliary cirrhosis. D. portal hypertension.

A. acute fulminant hepatitis.

Pre-hepatic causes of jaundice are primarily due to A. breakdown of red blood cells B. blockage of the common bile duct C. pancreatic cancer D. cirrhosis

A. breakdown of red blood cells

After ovulation, the uterine endometrial cells divide under the influence of estrogen; this is an example of: A. hormonal hyperplasia. B. hormonal dysplasia. C. hormonal hypertrophy. D. hormonal anaplasia

A. hormonal hyperplasia

Alcoholic liver disease manifests in three stages. The intermediate stage, alcoholic hepatitis, is characterized by liver cell: A. necrosis B. nodules C. atrophy D. hypertrophy

A. necrosis

When working with a client who has end-stage renal disease (ESRD) and is receiving peritoneal dialysis, the concept of diffusion can be explained by which of the following statements? A) ìIf your potassium level is high, then K+ particles will move from your peritoneal cavity into the dialysis solution, where the concentration of K+ is lower.î B) ìYou will need to give yourself a potent diuretic so that you can pull the potassium into your blood stream and filter the potassium out in your kidneys.î C) ìYour potassium molecules are lipid soluble and will dissolve in the lipid matrix of your cell membranes.î D) ìIf you can get very warm in a sauna, you will heat up your K+ particles, and the kinetic movement of the particles will increase and pass through the cell membranes faster.î

Ans: A Feedback: Diffusion refers to the passive process by which molecules and other particles in a solution become widely dispersed and reach a uniform concentration because of energy created by their spontaneous kinetic movements. In the process of reaching a uniform concentration, these molecules and particles move ìdownhillî from an area of higher to an area of lower concentration. Lipid-soluble molecules, such as oxygen, carbon dioxide, alcohol, and fatty acids (not K+), become dissolved in the lipid matrix of the cell membrane and diffuse through the membrane in the same manner that diffusion occurs in water. Diuretics are not very effective if a person has ESRD. The rate of diffusion depends on how many particles are available for diffusion, the kinetic movement of the particles, and the number and size of the openings in the membrane through which the molecules or ions can move. The environmental temperature does not play a role in this.

Ischemia and other toxic injuries increase the accumulation of intracellular calcium as a result of: A) Release of stored calcium from the mitochondria B) Improved intracellular volume regulation C) Decreased influx across the cell membrane D) Attraction of calcium to fatty infiltrates

Ans: A Feedback: Disruption of the normal intracellular functions causes the impaired mitochondria to release stored calcium. Cell injury disrupts intracellular volume regulation, allowing excessive influx of substances across the impaired cell membrane. Fatty infiltrate is an ominous sign of cell damage and does not interact with calcium.

Free radicals damage cells by: A) Destroying phospholipids in the cell membrane B) Altering the immune response of the cell C) Disrupting calcium storage in the cell D) Inactivation of enzymes and mitochondria

Ans: A Feedback: Free radicals are highly reactive and can damage cells in several ways. One way is by destroying lipids, which results in a loss of cell membrane integrity. Free radicals modify proteins but do not affect the immune function, calcium storage, or intracellular enzymes of cells.

Which of the following clients would be an example of cellular atrophy? A) A middle-aged female experiencing menopause due to loss of estrogen stimulation B) A postnephrectomy client whose remaining kidney enlarges to compensate for the loss C) A hypertensive, noncompliant client who has developed a progressive increase in left ventricular mass D) A female client with the change in uterine size as a result of pregnancy

Ans: A Feedback: In women, the loss of estrogen stimulation during menopause results in atrophic changes in the reproductive organs. Compensatory hypertrophy is the enlargement of a remaining organ or tissue after a portion has been surgically removed or rendered inactive. For instance, if one kidney is removed, the remaining kidney enlarges to compensate for the loss. In hypertension, for example, the increased workload required to pump blood against an elevated arterial pressure results in a progressive increase in left ventricular muscle mass and need for coronary blood flow. The pregnant uterus undergoes both hypertrophy and hyperplasia as a result of estrogen stimulation.

Which of the following processes associated with cellular injury is most likely to be reversible? A) Cell damage resulting from accumulation of fat in the cytoplasm B) Cellular changes as a result of ionizing radiation C) Cell damage resulting from accumulation of free radicals D) Apoptosis

Ans: A Feedback: Intracellular accumulation of fat leads to serious cell damage, but this is a potentially reversible effect. Ionizing radiation and damage from free radicals are more likely to be permanent, whereas apoptosis is defined as the permanent removal of injured and aged cells.

Which accurately explains the functions of the organelles lysosomes? They: A) Are sacs filled with enzymes that breakdown and remove foreign substances and worn-out cell parts B) Function in association with the endoplasmic reticulum to modify protein end products and package them into secretory granules or vesicles C) Are small particles of nucleoproteins that are involved in the synthesis of proteins that remain in the cell as cytoplasmic structural or functional elements D) Are a dynamic system of interconnected membranous tubes that functions as a tubular communication system for transporting various substances from one part of the cell to another

Ans: A Feedback: Lysosomes are sacs that are filled with hydrolytic enzymes that aid in the processing and removal of unwanted substances within the cytoplasm. The Golgi apparatus functions in association with the endoplasmic reticulum to modify and package substances in preparation for secretion, whereas ribosomes are small particles of nucleoproteins that are involved in the synthesis of proteins. The ER is a tubular communication system for transporting various substances from one part of the cell to another.

The process responsible for generating and conducting membrane potentials is: A) Diffusion of current-carrying ions B) Millivoltage of electrical potential C) Polarization of charged particles D) Ion channel neurotransmission

Ans: A Feedback: Membrane potentials rely on the permeability of the cell membrane and the diffusion of electrically charged ions. Charged particles are polarized (positive charge on one side of the membrane and negative charge on the opposite side of the membrane), but membrane potential exists when the charges are unbalanced on the two sides. Ion channel neurotransmitters are involved with opening protein channels for purposes of cell-to-cell communication.

Although the basic structure of the cell plasma membrane is formed by a lipid bilayer, most of the specific membrane functions are carried out by: A) Bound and transmembrane proteins B) Complex, long carbohydrate chains C) Surface antigens and hormone receptors D) A gating system of selective ion channels

Ans: A Feedback: The functions of plasma membrane depend on the presence of proteins that are bound in the lipid bilayer and some that have the ability to pass freely into and out of the cell. Carbohydrate chains form a cell coat that surrounds the membrane and that contain surface antigens and surface hormone receptors. Some ion channels are gated and open only when the membrane potential changes significantly.

A pregnant client is attending a nutrition class for first-time moms. During the class, the instructor stressed that they should avoid consumption of which food that may cause brain damage from methyl mercury exposure? A) Tuna B) Raw hamburger C) Fresh milk D) Beets

Ans: A Feedback: The main source of methyl mercury exposure is from consumption of long-lived fish, such as tuna and swordfish. Fish concentrate mercury from sediment in the water. Because the developing brain is more susceptible to mercury-induced damage, it is recommended that young children and pregnant and nursing women should avoid consumption of fish known to contain high mercury content. None of the other foods listed pose a threat of mercury toxicity.

A male client with a diagnosis of type 1 diabetes mellitus is experiencing hyperglycemia because he lacks sufficient insulin to increase the availability of glucose transporters in his cell membranes. Consequently, his cells lack intracellular glucose, and it accumulates in his blood. Which of the following processes would best allow glucose to cross his cell membranes? A) Facilitated diffusion B) Simple diffusion C) Secondary active transport D) Endocytosis

Ans: A Feedback: Transport molecules perform facilitated diffusion, in which one substance carries another substance across a cell membrane. Simple diffusion does not require a transport molecule. Glucose does not cross the cell membrane by secondary active transport or endocytosis.

With skeletal muscle contraction, what prevent the development of cross-bridges between the actin and myosin? Select all that apply. A) Tropomyosin attaches globular heads to the myosin filament. B) Troponin covers the tropomyosin-binding sites and prevents the formation of cross-bridges. C) ATP actually plays a role in the positioning of the myosin filaments and actin. D) The concentration of calcium around the myofibrils will prevent the cross-bridges from being formed.

Ans: A, B Feedback: Associated with each actin filament are the two regulatory proteins: tropomyosin and troponin. Tropomyosin, which lies in grooves of the actin strand, provides the site for attachment of the globular heads of the myosin filament. In the noncontracted state, troponin covers the tropomyosin-binding sites and prevents formation of cross-bridges between the actin and myosin. Energy from ATP is used to break the actin and myosin cross-bridges, stopping the muscle contraction. The binding of calcium to troponin uncovers the tropomyosin-binding sites such that the myosin heads can attach and form cross-bridges.

A person eating peanuts starts choking and collapses. His airway obstruction is partially cleared, but he remains hypoxic until he reaches the hospital. The health care providers will be assessing this client for which of the following physiological events? Select all that apply. A) Cerebral infarction B) Coagulation necrosis C) Rapid phagocytosis D) Protein p53 deficiency

Ans: A, B Feedback: Tissue infarction is caused by prolonged oxygen deprivation, and the resulting large group of dead cells coagulates in the damaged area. During coagulation necrosis, acidosis develops and denatures the enzymatic and structural proteins of the cell. Phagocytosis occurs rapidly during apoptosis, so it does not elicit an inflammatory response. Protein p53 activation initiates apoptosis.

Which of the following statements about how smooth muscle differs from skeletal or cardiac muscle are accurate? Smooth muscle: Select all that apply. A) Has dense bodies attached to actin filaments B) Contains sarcomeres between Z lines and M bands C) Contracts more rapidly than skeletal muscles D) Has one centrally located nucleus

Ans: A, D Feedback: Smooth muscle cells are spindle shaped and smaller than skeletal muscle fibers. Each smooth muscle cell has one centrally positioned nucleus. Smooth muscle contains dense bodies that are attached to the membrane, dispersed in the cell, and attached to actin filaments. Smooth muscle has slow contractions and no cross-striations. Sarcomeres are the functional units of cardiac and skeletal muscle only. The sarcomeres extend between Z lines and M bands.

Semen analysis indicates that the client's sperm have decreased motility. Which of the following cellular components may be defective within the client's sperm? A) Ribosomes B) Microtubules C) Mitochondria D) Microfilaments

Ans: B Feedback: Abnormalities in the structure and function of microtubules and consequent dysfunction of the flagella may contribute to impaired sperm motility. Ribosomes, microfilaments, and mitochondria do not directly contribute to movement in cells such as cilia and flagella.

Injured cells become very swollen as a result of: A) Increased cell protein synthesis B) Altered cell volume regulation C) Passive entry of potassium into the cell D) Bleb formation in the plasma membrane

Ans: B Feedback: Altered cell volume regulation, caused by the impaired permeability of the cell membrane, leads to sodium retention and substance accumulation within the cell. Sodium attracts water, and the cell swells even more. Protein synthesis is decreased in injured cells. Potassium has difficulty entering the cell and accumulates in the serum, due to loss of the ATPase sodium/potassium pump. Bleb formation occurs in the cell that is already swollen and ready to burst.

An elderly client has experienced some hypoxia as a result of chronic respiratory problems. Knowing that oxygen -deprived cells result in an accumulation of lactic acid in the cells, physiologically, the client may experience: A) An increase in fat load B) Altered cell membrane permeability C) Retention of lysosomal enzymes D) Cellular shrinkage and dehydration

Ans: B Feedback: Altered membrane permeability impairs the balance, allowing too much of some and not enough of other substances to flow in and out of the cell. The altered permeability, lack of ATP, and loss of functional surface receptors make it difficult for glucose to enter the cell. Excess fat accumulates because it is unable to move through the damaged membrane. Injury to the lysosomal membranes results in the leakage (not retention) of destructive lysosomal enzymes into the cytoplasm and enzymatic digestion of cell components.

The parents of a 4-year-old girl have sought care because their daughter has admitted to chewing and swallowing imported toy figurines that have been determined to be made of lead. Which of the following blood tests should the care team prioritize? A) White blood cell levels with differential B) Red blood cell levels and morphology C) Urea and creatinine levels D) Liver function panel

Ans: B Feedback: Anemia is a cardinal sign of lead toxicity. Consequently, assessment of the quantity and morphology of RBCs is paramount in cases of suspected lead toxicity. White blood cell and liver studies are not central to the care of this client. Lead is indeed nephrotoxic, and urea and creatinine levels are relevant to assessment, but the priority blood test is assessment of the RBCs.

A client with diabetes has impaired sensation, circulation, and oxygenation of his feet. He steps on a piece of glass, the wound does not heal, and the tissue area becomes necrotic. The necrotic cell death is characterized by: A) Rapid apoptosis B) Cellular breakage C) Shrinkage and collapse D) Chronic inflammation

Ans: B Feedback: Apoptosis is a programmed cell death, unrelated to cell injury, and occurs in a controlled, organized manner. Necrosis is an unorganized death of cells that initiates the acute inflammatory response with intracellular swelling and resulting cellular breakage (rupture).

While explaining to a postñsurgical knee client about the various forms and function of connective tissue, the nurse gives the example of the client's surgical repair of a torn anterior cruciate ligament (ACL), which is due to: A) A reticular fiber problem that interrupted the framework for capillaries B) A dense regular connective tissue tear that is usually rich in collagen fibers that allows ligaments to join bone to bone C) An irregular, dense connective tissue tear of loose connective tissue that is located in the perichondrium D) Irregular filling of spaces between tissues to facilitate keeping of joints and tendons in their proper place

Ans: B Feedback: Dense regular connective tissues are rich in collagen fibers and form the tendons and aponeuroses that join muscles to bone or other muscles and the ligaments that join bone to bone. Dense irregular connective tissue consists of the same components found in loose connective tissue but exhibits a predominance of collagen fibers and fewer cells. This type of tissue can be found in the fibrous sheaths of cartilage (i.e., perichondrium) and bone (i.e., periosteum). Fibroblasts, the most abundant loose connective tissue cells, synthesize the gel-like substance and collagen, elastin, and reticular fibers. Reticular fibers provide a fibrous framework for capillaries. Adipose tissue helps to fill spaces between tissues and keep organs in place.

Which of the following diseases would be considered to be caused by a lack of a specific vitamin or mineral? A) Anorexia nervosa B) Scurvy C) Sickle cell anemia D) Atherosclerosis

Ans: B Feedback: Dietary deficiencies can occur because of a selective deficiency of a single nutrient. Iron deficiency anemia, scurvy, beriberi, and pellagra are examples of injury caused by a lack of specific vitamins or minerals. Anorexia nervosa, sickle cell anemia, and atherosclerosis are not caused by lack of a vitamin/mineral.

An elderly client asks her health care provider if the reason she has developed aortic stenosis is because she drank so much milk as a child growing up on a farm. Which of the following responses is most accurate? A) ìDrinking lots of real milk as a child may have contributed to the damage in your valve.î B) ìAtherosclerosis is a long process that eventually results in calcification of heart valves.î C) ìThis calcification of your aortic valve is more than likely due to an undiagnosed thyroid problem.î D) ìMore than likely, calcium has left your bones and collected on your aortic valve.î

Ans: B Feedback: Dystrophic calcification represents the macroscopic deposition of calcium salts in injured tissue. Dystrophic calcification is commonly seen in atheromatous lesions of advanced atherosclerosis, in areas of injury in the aorta and large blood vessels, and in damaged heart valves. For example, calcification of the aortic valve is a frequent cause of aortic stenosis in the elderly. Drinking milk, undiagnosed thyroid problems, and calcium loss from bone causing osteoporosis do not cause damaged heart valves.

While caring for a client who has just been diagnosed with leukocyte adhesion deficiency (LAD), the client asks why he is always sick with an infection. The nurse responds, ìLAD is a rare autosomal recessive disorder that results in recurrent infections because of the lack of transmigration. This means: A) Your body doesn't make enough white blood cells.î B) Your white blood cells are not able to leave the blood vessels and move into the area of infection.î C) I don't really understand this, but it sounds like a good question to ask your physician.î D) Your bone marrow is damaged and can't put out enough white blood cells to fight off your infections.î

Ans: B Feedback: Leukocyte adhesion deficiency (LAD) is a rare autosomal recessive disorder characterized by immunodeficiency, resulting in recurrent infections. A WBC differential will reveal extremely elevated levels of neutrophils (on the order of 6ñ10 times normal) because they are unable to leave the blood vessels. Certain integrins play an important role in allowing white blood cells to pass through the vessel wall, a process called transmigration.

Which of the following facts underlies the concept of replicative senescence? A) Genes controlling longevity are present or absent in varying quantities among different individuals. B) Telomeres become progressively shorter in successive generations of a cell. C) The damaging influence of free radicals increases exponentially in later generations of a cell. D) Aging produces mutations in DNA and deficits in DNA repair.

Ans: B Feedback: Replicative senescence implies that cells have limited capacity for reproduction, largely as a result of the shortening of telomeres and consequent chromosomal damage. Genetic theories, the influence of free radicals, and DNA mutation are not central to the concept of replicative senescence.

A 70-year-old male client has been admitted to a hospital for the treatment of a recent hemorrhagic stroke that has left him with numerous motor and sensory deficits. These deficits are most likely the result of which of the following mechanisms of cell injury? A) Free radical injury B) Hypoxia and ATP depletion C) Interference with DNA synthesis D) Impaired calcium homeostasis

Ans: B Feedback: Stroke is characterized by impaired cerebral circulation and consequent death of neurons from cellular hypoxia. Free radical injury, abnormal DNA synthesis, and impaired calcium homeostasis are not direct consequences of lack of blood flow to body cells.

Which describes the function of the nucleus? A) It is basically the site of protein synthesis in the body. B) It contains the genetic code for the individual. C) It is the transformer of cellular energy. D) It initiates the process of aerobic metabolism.

Ans: B Feedback: The nucleus contains DNA, which contains the genetic code that contains the information that controls cells. Ribosomes synthesize protein. Mitochondria transform organic compounds into cellular energy. Mitochondria require oxygen for aerobic metabolism, using hydrogen and carbon combined with oxygen molecules to form carbon dioxide and water as energy is released.

When explaining the function of glycolysis as it relates to anaerobic metabolism, the faculty will mention which of the following key points? Select all that apply. A) Glycolysis requires the use of oxygen to begin the process. B) Glycolysis occurs in mitochondrion-lacking cells. C) Glycolysis provides the majority of the body's energy needs. D) Pyruvic acid is an end result from a series of reactions that converts glucose.

Ans: B, D Feedback: Glycolysis is the anaerobic process by which energy is liberated from glucose, and it is an important source of energy for cells that lack mitochondria. The process also provides a temporary source of energy for cells that are deprived of an adequate supply of oxygen. Glycolysis involves a sequence of reactions that converts glucose to pyruvic acid, with the concomitant production of ATP from ADP. It accounts for a small minority of the body's energy needs and results in NADH, hydrogen ions, ATP, and pyruvic acid.

A mother rushes her 4-year-old child to the emergency department after she found an empty Tylenol (acetaminophen) bottle beside her child. The nurse is trying to explain why it is so important to give the child Ipecac to induce vomiting in order to prevent: A) Renal failure B) Seizures C) Liver failure D) Hemorrhage

Ans: C Feedback: Acetaminophen, a commonly used over-the-counter analgesic drug, is detoxified in the liver, where small amounts of the drug are converted to a highly toxic metabolite. This metabolite is detoxified by a metabolic pathway that uses a substance normally present in the liver. When large amounts of the drug are ingested, this pathway becomes overwhelmed and toxic metabolites accumulate, causing massive liver necrosis.

A client has been diagnosed with a gram-negative bacillus in his blood cultures. The health care providers know these bacteria may produce clinical manifestations such as high temperature, high respiratory rate, and low blood pressure. These manifestations are primarily caused by: A) Disrupting the sodium/potassium ATPase pump B) Interrupting oxidative metabolism processes C) The outer layer of the bacterial membrane acting as an endotoxin D) The bacteria causing a decrease in protein synthesis and function

Ans: C Feedback: Bacteria and viruses can replicate within a cell, thus perpetuating the injuries. Gram-negative bacilli have unique characteristics in the structure of the outer membrane. The outer leaflet of the membrane has a lipid portion that acts as an endotoxin. If this bacillus enters the circulatory system, it causes a toxic reaction, with the sufferer developing a high temperature, high respiration rate, and low blood pressure. Other agents that are injurious to cells are unable to replicate in the cell, but they may disrupt the sodium/potassium pump, interrupt oxidative metabolism, or decrease protein synthesis.

The client is found to have liver disease, resulting in the removal of a lobe of his liver. Adaptation to the reduced size of the liver leads to which phenomenon in the remaining liver cells? A) Metaplasia B) Organ atrophy C) Compensatory hyperplasia D) Physiologic hypertrophy

Ans: C Feedback: Compensatory hyperplasia can be stimulated in response to loss of vital tissue that is capable of regeneration, such as liver cells. Metaplasia involves replacement of one existing cell type with another fully differentiated cell type. Organ atrophy is caused by irreversible loss of cells. Physiologic hypertrophy is increased size of existing cells resulting from increased workload.

A client has experienced significant decreases in mobility and stamina during a 3-week hospital stay for the treatment of a femoral head fracture. Which of the following phenomena most likely accounts for the client's decrease in muscle function? A) Impaired muscle cell metabolism resulting from metaplasia B) Dysplasia as a consequence of inflammation during bone remodeling C) Disuse atrophy of muscle cells during a prolonged period of immobility D) Ischemic atrophy resulting from vascular changes while on bedrest

Ans: C Feedback: Disuse atrophy frequently occurs as a consequence of prolonged periods of muscle inactivity. Metaplasia and dysplasia are not common consequences of immobility and muscle disuse. Similarly, infrequent muscle use does not typically cause vascular changes that result in ischemic atrophy.

A client with diabetes and severe peripheral vascular disease has developed signs of dry gangrene on the great toe of one foot. The client asks, ìHow this can happen?î Which of the following pathophysiologic processes should the nurse explain to this client? ìMore than likely, your gangrene is caused by: A) Inappropriate activation of apoptosis, which means death of your cells.î B) Bacterial invasion into the foot and toe.î C) Impaired arterial blood supply to your toe.î D) Metaplastic cellular changes in your toe.î

Ans: C Feedback: Dry gangrene is often a result of impaired arterial blood supply to the extremities. A bacterial etiology is more common in wet gangrene, whereas neither metaplasia nor activation of apoptosis is implicated in cases of dry gangrene.

Epithelial tissues are classified according to the shape of the cells and the number of layers. Which of the following is a correctly matched description and type of epithelial tissue? A) Simple epithelium: cells in contact with the intercellular matrix; some do not extend to surface. B) Stratified epithelium: single layer of cells; all cells rest on the basement membrane. C) Glandular epithelium: arises from surface epithelia and underlying connective tissue. D) Pseudostratified epithelium: multiple layers of cells; the deepest layer rests on the basement membrane.

Ans: C Feedback: Glandular epithelial tissue is formed by cells to produce fluid and arises from surface epithelium, involving connective tissue. Simple epithelium is a single layer of cells resting on the basement membrane; stratified epithelium has more than one layer, with the deepest layer resting on the basement membrane. Pseudostratified epithelium is in contact with the intercellular matrix and may not extend to the surface.

Exocytosis allows granular content to be released into extracellular fluid by: A) Engulfing and ingesting fluid and proteins for transport B) Killing, degrading, and dissolving harmful microorganisms C) Removing cellular debris and releasing synthesized substances like hormones D) Destruction of particles by lysosomal enzymes for secretion

Ans: C Feedback: In exocytosis, a secretory granule fuses to the inner cell membrane to form an opening, allowing granule contents to be released. The granule contains cellular debris and synthesized substances such as hormones, which it releases into the extracellular fluid. Phagocytosis and pinocytosisótypes of endocytosisófunction to engulf, kill, and present particles to lysosomal enzymes for degradation.

Although energy is not made in mitochondria, they are known as the ìpower plantsî of the cell because they: A) Contain RNA for protein synthesis B) Utilize glycolysis for oxidative energy C) Extract energy from organic compounds D) Store calcium bonds for muscle contractions

Ans: C Feedback: Mitochondria contain the enzymes needed for transforming organic compounds into energy that is easily accessible to the cell. Mitochondria contain their own DNA. Glycolysis is anaerobic metabolism and unrelated to oxidative energy. Mitochondria store phosphate bonds (such as in ATP) to power cellular functions.

Despite the low levels of radiation used in contemporary radiologic imaging, a radiology technician is aware of the need to minimize her exposure to ionizing radiation. What is the primary rationale for the technician's precautions? Radiation: A) Stimulates pathologic cell hypertrophy and hyperplasia B) Results in the accumulation of endogenous waste products in the cytoplasm C) Interferes with DNA synthesis and mitosis D) Decreases the action potential of rapidly dividing cells

Ans: C Feedback: Radiation has a damaging effect on DNA synthesis and mitosis, a process that is especially harmful to rapidly dividing cells. Radiation does not directly influence the action potential of cells or the accumulation of endogenous waste products. Cell changes such as hypertrophy or hyperplasia may result from radiation exposure, but such changes are secondary to interference with DNA synthesis and mitosis.

When performing an assessment on a school-aged child, the nurse notes that the mucous membranes along the gum margins have a noticeable blue-colored line. At this point, the nurse should ask the parents about possible: A) Liver problems as an infant B) Congenital heart problems C) Exposure to lead D) Second-hand smoke exposure

Ans: C Feedback: The formation of a blue lead line along the margins of the gum is one of the diagnostic features of lead poisoning. Liver problems are usually displayed as jaundice (yellowing of the skin/sclera). Congenital heart problems may exhibit cyanosis, but this would not be just in the gum margins. Second-hand smoke exposure may cause accumulation in the lungs.

Which of the following aspects of the function of the nucleus is performed by ribosomal RNA (rRNA)? A) Copying and carrying DNA instructions for protein synthesis B) Carrying amino acids to the site of protein synthesis C) Providing the site where protein synthesis occurs D) Regulating and controlling protein synthesis

Ans: C Feedback: There are three types of ribonucleic acid (messenger RNA, ribosomal RNA, and transfer RNA) that move to the cytoplasm and carry out the actual synthesis of proteins. Messenger RNA (mRNA) copies and carries the DNA instructions for protein synthesis to the cytoplasm; ribosomal RNA (rRNA) is the site of protein synthesis; and transfer RNA (tRNA) transports amino acids to the site of protein synthesis for incorporation into the protein being synthesized.

When exercising outside on an extremely warm day, the client can feel his heart pounding very rapidly. Thinking in terms of the ability of the aorta to stretch to accommodate more blood circulating during exercise and the warm environment, this is an example of the function of which fibrous protein? A) Collagen B) Reticular C) Elastin D) Ligaments

Ans: C Feedback: Three types of fibers are found in the extracellular space: collagen, elastin, and reticular fibers. Elastin acts like a rubber band; it can be stretched and then returns to its original form. Elastin fibers are abundant in structures subjected to frequent stretching, such as the aorta and some ligaments. Collagen is the most common protein in the body. It is a tough, nonliving, white fiber that serves as the structural framework for skin, ligaments, tendons, and many other structures. Reticular fibers are extremely thin fibers that create a flexible network in organs subjected to changes in form or volume, such as the spleen, liver, uterus, or intestinal muscle layer.

Which identifies correctly how the G proteinñlinked receptors are similar? A) Their cytosolic domain has intrinsic enzyme activity. B) Insulin is an example of the second messenger cAMP, which binds to an enzyme-linked receptor. C) These linked receptors are involved in rapid synaptic signaling between cardiac electrical cells. D) They have a ligand-binding extracellular receptor component, which causes changes that activate the G protein on the cytoplasmic side of the cell membrane.

Ans: D Feedback: Although there are differences among the G proteinñlinked receptors, all share a number of features. They all have a ligand-binding extracellular receptor component, which recognizes a specific ligand or first messenger. Upon ligand binding, they all undergo conformational changes that activate the G protein found on the cytoplasmic side of the cell membrane. Instead of having a cytosolic domain that associates with a G protein, enzyme-linked receptors have cytosolic domain either that has intrinsic enzyme activity or that associates directly with an enzyme. The binding of the hormone to a special transmembrane receptor results in activation of the enzyme adenylyl cyclase at the intracellular portion of the receptor. This enzyme then catalyzes the formation of the second messenger cAMP, which has multiple effects on cell function. Insulin, for example, acts by binding to an enzyme-linked receptor. Ion channelñlinked receptors are involved in the rapid synaptic signaling between electrically excitable cells.

A client with a pathophysiologic condition that affects the desmosomes is most likely to exhibit: A) Impaired contraction of skeletal and smooth muscle B) Weakness of the collagen and elastin fibers in the extracellular space C) Impaired communication between neurons and effector organs D) Separation at the junctions between epithelial cells

Ans: D Feedback: Desmosomes are a type of cell junction common in epithelium. Failure of epithelial desmosomes results in the inappropriate separation of adjacent cells. Desmosomes do not contribute to muscle contraction, communication between neurons and effector organs, or the structure of collagen and elastin.

A group of elderly residents were commenting on how many cell functions decline with age. One resident commented that many of his friends who lived under large electromagnetic towers seemed to experience aging at an accelerated rate in comparison with residents who lived nearby in lakefront housing. This observation is the basis for which theory on aging? A) Theories of genetic influences B) Programmed cell receptor theories C) Insufficient telomerase enzyme theory D) Error theory associated with DNA damage

Ans: D Feedback: Error theory suggests that aging results from DNA mutation or faulty repair. Another group of theories of aging focuses on programmed cell changes with genetic influences that systematically cause cell senescence. Elimination of receptor sites is not part of aging theory. Telomerase enzyme is thought to reduce the shortening of the chromosomes and loss of telomere DNA with each cell replication.

Which of the following pathophysiologic processes is most likely to result in metastatic calcification? A) Benign prostatic hyperplasia B) Liver cirrhosis C) Impaired glycogen metabolism D) Hyperparathyroidism

Ans: D Feedback: Metastatic calcification is a result of markedly increased serum calcium levels. Because the parathyroid gland is responsible for the regulation of serum calcium levels, hyperparathyroidism creates a risk for hypercalcemia and consequent metastatic calcification. Benign prostatic hypertrophy, cirrhosis, and impaired glycogen metabolism are not implicated in cases of metastatic calcification.

Impairment in the function of peroxisomes would result in: A) Inadequate sites for protein synthesis B) An inability to transport cellular products across the cell membrane C) Insufficient energy production within a cell D) Accumulation of free radicals in the cytoplasm

Ans: D Feedback: Peroxisomes function in the control of free radicals; unless degraded, these highly unstable chemical compounds damage other cytoplasmic molecules. Peroxisomes do not directly contribute to energy production, protein synthesis, or transport of cellular secretions.

The client asks the health care provider to explain phagocytosis. The provider will respond, ìPhagocytosis: A) Is a cotransport system that helps with the absorption of the medication.î B) Is the movement of particles from an area of higher concentration to one of lower concentration.î C) Uses proteins to form an open channel through which the drug can move into the cell.î D) Is a process where microorganisms are engulfed and subsequently degraded or killed.î

Ans: D Feedback: Phagocytosis, which means ìcell eating,î involves the engulfment and subsequent killing or degradation of microorganisms and other particulate matter. Certain cells, such as macrophages and neutrophils, are adept at engulfing and disposing of invading organisms, damaged cells, and unneeded extracellular constituents. An example of cotransport occurs in the intestine, where the absorption of glucose and amino acids is coupled with sodium transport. The process of diffusion describes particle movement from an area of higher concentration to an area of lower concentration, resulting in an equal distribution of permeable substances across the cell membrane. Ion channels are integral proteins that span the width of the membrane and are normally composed of several polypeptides or protein subunits that form a gating system. Specific stimuli cause the protein subunits to undergo conformational changes to form an open channel or gate through which the ions can move.

To effectively relay signals, cell-to-cell communication utilizes a chemical messenger system to: A) Displace surface receptor proteins B) Accumulate within cell gap junctions C) Bind to contractile microfilaments D) Release secretions into extracellular fluid

Ans: D Feedback: Signals are transmitted by releasing chemical secretions into extracellular fluid. Chemical signals move through cell-to-cell junctions to reach other cells and may attach to surface receptor proteins. The cytoplasmic contractile microfilaments are incapable of transmitting communication signals.

A student asks the instructor what it means when the book states, skeletal muscles are syncytial or multinucleated? The instructor responds: A) ìEach of the skeletal muscle cells has an apical, lateral, and basal surface.î B) ìThey are closely apposed and are joined by cell-to-cell adhesion molecules.î C) ìThis means that these muscles exhibit cross-striations formed by alternating segments of thick and thin protein filaments, with muscle cells having a branched appearance.î D) ìIf a skeletal muscle is injured and a portion dies, the adjacent sections of that same skeletal muscle fiber do not die because they have their own nuclear material.î

Ans: D Feedback: Skeletal muscles are syncytial or multinucleated structures, meaning there are no true cell boundaries within a skeletal muscle fiber. The multinucleated arrangement is important in pathologic states where focal necrosis (death) of a portion of skeletal muscle fibers does not result in necrosis death of the adjacent sections of that same skeletal muscle fiber, because those adjacent sections have their own nuclear material. Cardiac muscle exhibits cross-striations formed by alternating segments of thick and thin protein filaments. In contrast to skeletal muscle, cardiac muscle cells may be branched instead of linear and longitudinal. The other noted qualities are associated with epithelial, not skeletal muscle, cells.

A client has been diagnosed with gram-negative pneumonia of the lower lobe. Knowing that gram-negative bacteria have a lipopolysaccharide endotoxin on their outer cell membrane, the health care provider should be assessing the client for which pathophysiological end result? A) Damage to cellular mitochondria B) Increased ATP levels C) Activation of the p53 protein D) Apoptosis

Ans: D Feedback: The extrinsic pathway of apoptosis involves extracellular signaling proteins that bind to cell surface molecules called death receptors and trigger apoptosis. The end result includes activation of endonucleases that cause fragmentation of DNA and cell death. In addition to TNF and Fas ligand, primary signaling molecules known to activate the extrinsic pathway include TNF-related apoptosis-inducing ligand (TRAIL); the cytokine interleukin-1 (IL-1); and lipopolysaccharide (LPS), the endotoxin found in the outer cell membrane of gram-negative bacteria. DNA damage, activation of the p53 protein, and decreased ATP levels are associated with the intrinsic pathway.

Aerobic metabolism, also known as oxidative metabolism, provides energy to the body by: A) Removing the phosphate bonds from ATP B) Combining hydrogen and oxygen to form water C) Activating pyruvate stored in the cytoplasm D) Breaking down glucose to form lactic acid

Ans: b Feedback: Aerobic metabolism involves the combination of carbon ions from dietary nutrients, hydrogen ions, and oxygen. The result is carbon dioxide and water as energy is released, which is stored in ATP. Phosphate bonds are added (not removed from) to ADP to form ATP. Pyruvate is formed from glucose in the anaerobic process of glycolysis and is converted to lactic acid during anaerobic metabolism.

Glomerulonephritis is usually caused by:

Antigen-antibody complexes

A client has been diagnosed with gram-negative pneumonia of the lower lobe. Knowing that gram-negative bacteria have a lipopolysaccharide endotoxin on their outer cell membrane, the health care provider should be assessing the client for which pathophysiological end result?

Apoptosis

What concept will the nurse need to explain to a client with cancer about the purpose and action of chemotherapy?

Apoptosis

Which of the following is defined as programmed cell death?

Apoptosis

Which accurately explains the functions of the organelles lysosomes? They:

Are sacs filled w/ enzymes that breakdown & remove foreign substances & worn-out cell parts

Which accurately explains the functions of the organelles lysosomes? They:

Are sacs filled with enzymes that breakdown and remove foreign substances and worn-out cell parts

A client is diagnosed with an abdominal aortic aneurysm that the physician just wants to "watch" for now. When teaching the client about signs/symptoms to watch for, the nurse will base the teaching on which of the following physiological principles?

As the aneurysm grows, more tension is placed on the vessel wall, which increases the risk for rupture.

A family brings their elderly mom to the emergency department. The client had a short period of time where she was confused, had slurred speech and appeared to have a weak arm. Now she is back to her normal self. Suspecting a transient ischemic attacks (TIAs), the health care provider will order diagnostic testing looking for:

Atherosclerotic lesions in cerebral vessels

In immune thrombocytopenia purpura (ITP), the client has what type of disorder that primarily destroys which blood component?

Autoimmune; platelets

Unlike the tissue growth that occurs with hypertrophy and hyperplasia, the growth of a malignancy is:

Autonomous

A client has arrived in the emergency department with massive blood loss from a lacerated liver. Knowing the physiological principles behind the capacity for cells to regenerate, the client's body will likely: A. Begin a cell cycle so stem cells can undergo numerous mitotic divisions B. Stimulate rapid proliferation of RBCs by the blood-forming progenitor cells of the bone marrow C. Pull fluid from the tissues to maintain a reasonable blood pressure D. Activate the tissue cells to start producing stem cells

B

patient comes to the clinic following exposure to chicken pox. The patient states he had chicken pox as a child but was worried about getting sick again. Which of the following statements best explains humoral-mediated immunity to the patient?

B lymphocytes (B cells) in the body produce antibodies to fight infections. Once exposed, the cells retain memory and are able to quickly fight off pathogens during re-exposure and prevent the disease from reoccurring.

A client cuts herself with a sharp knife while cooking dinner. The client describes how the wound started bleeding and had a red appearance almost immediately. The nurse knows that in the vascular stage of acute inflammation, the vessels: A. Constrict as a result of "fight/flight" hormone release resulting in pale-colored skin B. Vasodilate causing the area to become congested causing the red color and warmth C. Bleed profusely until the body can compensate and start to send fibrinogen to the wound D. Swell to the point of compromising circulation causing the limb to become cool to touch

B)

A nursing student is cleaning and changing the dressing on a client's sacral ulcer. The student has vigorously cleansed the wound bed to remove all traces of the beefy, red tissue that existed in the wound bed. The student has most likely removed: A. Necrotic tissue B. Granulation tissue C. The extracellular matrix D. Stem cells

B)

Which of the following clients is most likely to experience impaired wound healing? A. A client who takes nebulized bronchodilators several times daily to treat chronic obstructive pulmonary disease B. A client with a diagnosis of type 1 diabetes and a history of poor blood sugar control C. A client with persistent hypertension who takes a β-adrenergic blocker and a potassium-wasting diuretic daily D. A child whose severe cleft lip and palate have required a series of surgeries over several months

B)

Which of the following is defined as programmed cell death? A. Metaplasia B. Apoptosis C. Atrophy D. Hypertrophy

B)

An elderly client asks her health care provider if the reason she has developed aortic stenosis is because she drank so much milk as a child growing up on a farm. Which of the following responses is most accurate? A) "Drinking lots of real milk as a child may have contributed to the damage in your valve." B) "Atherosclerosis is a long process that eventually results in calcification of heart valves." C) "This calcification of your aortic valve is more than likely due to an undiagnosed thyroid problem." D) "More than likely, calcium has left your bones and collected on your aortic valve."

B) "Atherosclerosis is a long process that eventually results in calcification of heart valves." Feedback: Dystrophic calcification represents the macroscopic deposition of calcium salts in injured tissue. Dystrophic calcification is commonly seen in atheromatous lesions of advanced atherosclerosis, in areas of injury in the aorta and large blood vessels, and in damaged heart valves. For example, calcification of the aortic valve is a frequent cause of aortic stenosis in the elderly. Drinking milk, undiagnosed thyroid problems, and calcium loss from bone causing osteoporosis do not cause damaged heart valves.

An elderly client has experienced some hypoxia as a result of chronic respiratory problems. Knowing that oxygen deprived cells result in an accumulation of lactic acid in the cells, physiologically, the client may experience: A) An increase in fat load B) Altered cell membrane permeability C) Retention of lysosomal enzymes D) Cellular shrinkage and dehydration

B) Altered cell membrane permeability

An elderly client has experienced some hypoxia as a result of chronic respiratory problems. Knowing that oxygen -deprived cells result in an accumulation of lactic acid in the cells, physiologically, the client may experience: A) An increase in fat load B) Altered cell membrane permeability C) Retention of lysosomal enzymes D) Cellular shrinkage and dehydration

B) Altered cell membrane permeability Feedback: Altered membrane permeability impairs the balance, allowing too much of some and not enough of other substances to flow in and out of the cell. The altered permeability, lack of ATP, and loss of functional surface receptors make it difficult for glucose to enter the cell. Excess fat accumulates because it is unable to move through the damaged membrane. Injury to the lysosomal membranes results in the leakage (not retention) of destructive lysosomal enzymes into the cytoplasm and enzymatic digestion of cell components.

17. Injured cells become very swollen as a result of: A) Increased cell protein synthesis B) Altered cell volume regulation C) Passive entry of potassium into the cell D) Bleb formation in the plasma membrane

B) Altered cell volume regulation

Injured cells become very swollen as a result of: A) Increased cell protein synthesis B) Altered cell volume regulation C) Passive entry of potassium into the cell D) Bleb formation in the plasma membrane

B) Altered cell volume regulation Feedback: Altered cell volume regulation, caused by the impaired permeability of the cell membrane, leads to sodium retention and substance accumulation within the cell. Sodium attracts water, and the cell swells even more. Protein synthesis is decreased in injured cells. Potassium has difficulty entering the cell and accumulates in the serum, due to loss of the ATPase sodium/potassium pump. Bleb formation occurs in the cell that is already swollen and ready to burst.

A client with diabetes has impaired sensation, circulation, and oxygenation of his feet. He steps on a piece of glass, the wound does not heal, and the tissue area becomes necrotic. The necrotic cell death is characterized by: A) Rapid apoptosis B) Cellular breakage C) Shrinkage and collapse D) Chronic inflammation

B) Cellular breakage

A client with diabetes has impaired sensation, circulation, and oxygenation of his feet. He steps on a piece of glass, the wound does not heal, and the tissue area becomes necrotic. The necrotic cell death is characterized by: A) Rapid apoptosis B) Cellular breakage C) Shrinkage and collapse D) Chronic inflammation

B) Cellular breakage Feedback: Apoptosis is a programmed cell death, unrelated to cell injury, and occurs in a controlled, organized manner. Necrosis is an unorganized death of cells that initiates the acute inflammatory response with intracellular swelling and resulting cellular breakage (rupture).

A 70-year-old male client has been admitted to a hospital for the treatment of a recent hemorrhagic stroke that has left him with numerous motor and sensory deficits. These deficits are most likely the result of which of the following mechanisms of cell injury? A) Free radical injury B) Hypoxia and A TP depletion C) Interference with DNA synthesis D) Impaired calcium homeostasis

B) Hypoxia and A TP depletion

16. A 70-year-old male patient has been admitted to a hospital for the treatment of a recent hemorrhagic stroke that has left him with numerous motor and sensory deficits. These deficits are most likely the result of which of the following mechanisms of cell injury? A) Free radical injury B) Hypoxia and ATP depletion C) Interference with DNA synthesis D) Impaired calcium homeostasis

B) Hypoxia and ATP depletion

A 70-year-old male client has been admitted to a hospital for the treatment of a recent hemorrhagic stroke that has left him with numerous motor and sensory deficits. These deficits are most likely the result of which of the following mechanisms of cell injury? A) Free radical injury B) Hypoxia and ATP depletion C) Interference with DNA synthesis D) Impaired calcium homeostasis

B) Hypoxia and ATP depletion Feedback: Stroke is characterized by impaired cerebral circulation and consequent death of neurons from cellular hypoxia. Free radical injury, abnormal DNA synthesis, and impaired calcium homeostasis are not direct consequences of lack of blood flow to body cells.

3. A person eating peanuts starts choking and collapses. His airway obstruction is partially cleared, but he remains hypoxic until he reaches the hospital. The prolonged cell hypoxia caused a cerebral infarction and resulting __________ in the brain. A) caspase activation B) coagulation necrosis C) rapid phagocytosis D) protein p53 deficiency

B) Rapid phagocytosis

15. The parents of a 4-year-old girl have sought care because their daughter has admitted to chewing and swallowing imported toy figurines that have been determined to be made of lead. Which of the following blood tests should the care team prioritize? A) White blood cell levels with differential B) Red blood cell levels and morphology C) Urea and creatinine levels D) Liver function panel

B) Red blood cell levels and morphology

10. The parents of a 4-year-old girl have sought care because their daughter has admitted to chewing and swallowing imported toy figurines that have been determined to be made of lead. Which of the following blood tests should the care team prioritize? A) White blood cell levels with differential B) Red blood cell levels and morphology C) Urea and creatinine levels D) Liver function panel

B) Red blood cell levels and morphology Feedback: Anemia is a cardinal sign of lead toxicity. Consequently, assessment of the quantity and morphology of RBCs is paramount in cases of suspected lead toxicity. White blood cell and liver studies are not central to the care of this client. Lead is indeed nephrotoxic, and urea and creatinine levels are relevant to assessment, but the priority blood test is assessment of the RBCs.

13. Which of the following diseases would be considered to be caused by a lack of a specific vitamin or mineral? A) Anorexia nervosa B) Scurvy C) Sickle cell anemia D) Atherosclerosis

B) Scurvy

Which of the following diseases would be considered to be caused by a lack of a specific vitamin or mineral? A) Anorexia nervosa B) Scurvy C) Sickle cell anemia D) Atherosclerosis

B) Scurvy Feedback: Dietary deficiencies can occur because of a selective deficiency of a single nutrient. Iron deficiency anemia, scurvy, beriberi, and pellagra are examples of injury caused by a lack of specific vitamins or minerals. Anorexia nervosa, sickle cell anemia, and atherosclerosis are not caused by lack of a vitamin/mineral.

20. Which of the following facts underlies the concept of replicative senescence? A) Genes controlling longevity are present or absent in varying quantities among different individuals. B) Telomeres become progressively shorter in successive generations of a cell. C) The damaging influence of free radicals increases exponentially in later generations of a cell. D) Aging produces mutations in DNA and deficits in DNA repair.

B) Telomeres become progressively in successive generations of a cell.

25. Which of the following facts underlies the concept of replicative senescence? A) Genes controlling longevity are present or absent in varying quantities among different individuals. B) Telomeres become progressively shorter in successive generations of a cell. C) The damaging influence of free radicals increases exponentially in late generations of a cell. D) Aging produces mutations in DNA and deficits in DNA repair.

B) Telomeres become progressively shorter in successive generations of a cell.

Which of the following facts underlies the concept of replicative senescence? A) Genes controlling longevity are present or absent in varying quantities among different individuals. B) Telomeres become progressively shorter in successive generations of a cell. C) The damaging influence of free radicals increases exponentially in later generations of a cell. D) Aging produces mutations in DNA and deficits in DNA repair.

B) Telomeres become progressively shorter in successive generations of a cell. Feedback: Replicative senescence implies that cells have limited capacity for reproduction, largely as a result of the shortening of telomeres and consequent chromosomal damage. Genetic theories, the influence of free radicals, and DNA mutation are not central to the concept of replicative senescence.

8. Injured cells become very swollen as a result of: A) increased cell protein synthesis. B) altered cell volume regulation. C) passive entry of potassium into the cell. D) bleb formation in the plasma membrane.

B) altered cell volume regulation

6. Injured cells have impaired flow of substances through the cell membrane as a result of: A) increased fat load. B) altered permeability. C) altered glucose utilization. D) increased surface receptors.

B) altered permeability

9. A diabetic patient has impaired sensation, circulation, and oxygenation of his feet. He steps on a piece of glass, the wound does not heal, and the area tissue becomes necrotic. The necrotic cell death is characterized by: A) rapid apoptosis. B) cellular rupture. C) shrinkage and collapse. D) chronic inflammation.

B) cellular rupture

Which immune cells bind with free-floating recognized antigens?

B-cells

An elderly client asks her health care provider if the reason she has developed aortic stenosis is because she drank so much milk as a child growing up on a farm. Which of the following responses is most accurate? A. "Drinking lots of real milk as a child may have contributed to the damage in your valve." B. "Atherosclerosis is a long process that eventually results in calcification of heart valves." C. "This calcification of your aortic valve is more than likely due to an undiagnosed thyroid problem." D. "More than likely, calcium has left your bones and collected on your aortic valve."

B. "Atherosclerosis is a long process that eventually results in calcification of heart valves."

A heart failure client has an echocardiogram performed revealing an ejection fraction (EF) of 40%. The nurse knows this EF is below normal and explains to the client: A. "This means you have a lot of pressure built-up inside your heart." B. "This means your heart is not pumping as much blood out of the heart with each beat." C. "You need to increase the amount of exercise you do to get your heart muscle back in shape." D. "Your ventricular muscle is getting too stiff to beat normally."

B. "This means your heart is not pumping as much blood out of the heart with each beat."

A client with a history of heart failure has been referred for an echocardiogram. Results of this diagnostic test reveal the following findings: heart rate 80 beats/minute; end-diastolic volume 120 mL; and end-systolic volume 60 mL. What is this client's ejection fraction? A. 200 mL B. 50% C. 0.80 D. 180 mL

B. 50%

Which of the following individuals is suffering the effects of acute coronary syndrome (ACS)? A. A client whose most recent ECG indicates that silent myocardial ischemia has occurred B. A client who occasionally experiences persistent and severe chest pain when at rest C. A client who sometimes experiences chest pain when climbing stairs D. A client who has recently been diagnosed with variant (vasospastic) angina

B. A client who occasionally experiences persistent and severe chest pain when at rest

Which of the following individuals is experiencing a health problem that is the result of a parasite? A. A college student who contracted Chlamydia trachomatis during an unprotected sexual encounter B. A man who acquired malaria while on a tropical vacation C. A hospital client who has developed postoperative pneumonia D. A woman who developed hepatitis A from eating at an unhygienic restaurant

B. A man who acquired malaria while on a tropical vacation

A decrease in the serum level of which of the following substances is suggestive of liver injury? A. g-Glutamyltransferase (GGT) B. Albumin C. Alanine aminotransferase (ALT) D. Alkaline phosphatase

B. Albumin

The gene responsible for a particular congenital cardiac anomaly is said to have complete penetrance. What are the clinical implications of this fact? Select one: A. The anomaly is a result of polygenetic inheritance. B. All the individuals who possess the gene will exhibit the anomaly. C. Multiple alleles contribute to the defect. D. The heart defect does not result from any other gene.

B. All the individuals who possess the gene will exhibit the anomaly.

Which of the following clients is at the greatest risk for developing an intracellular pathogen infection? A. A teenager who attends a crowded high school B. An AIDS client with a decreased CD4+ TH1 count C. A breast cancer client who has a WBC count of 8000 D. A hyperthyroid client who has received treatment with radioactive iodine

B. An AIDS client with a decreased CD4+ TH1 count

During cell injury caused by hypoxia, why do sodium and water move into the cell? A. Because potassium moves out of the cell, and potassium and sodium are inversely related B. Because the pump that transports sodium out of the cell cannot function because of a decreased in ATP C. Because the osmotic pressure is increased, which pulls additional sodium across the cell membrane D. Because oxygen is not available to bind with sodium to maintain it outside of the cell

B. Because the pump that transports sodium out of the cell cannot function because of a decrease in ATP

In hypoxic injury, why does sodium to enter the cell and cause swelling? A. Because the cell membrane permeability increases for sodium during periods of hypoxia B. Because there is insufficient ATP to maintain the pump that keeps sodium out of the cell C. Because the lactic acid produced by the hypoxia binds with sodium within the cell D. Because sodium cannot be transported in the cytosol to the cell membrane during hypoxia

B. Because there is insufficient ATP to maintain the pump that keeps sodium out of the cell

The most common cancer diagnosed in the US (regardless of race or gender) is: A. Lung B. Breast C. Colon D. Prostate

B. Breast

How does carbon monoxide cause tissue damage? A. By competing with carbon dioxide so that it cannot be excreted B. By binding to hemoglobin so that it cannot carry oxygen C. By destroying the chemical bonds of hemoglobin so it cannot carry oxygen D. By removing iron from hemoglobin so it cannot carry oxygen

B. By binding to hemoglobin so that it cannot carry oxygen

Atherosclerotic peripheral vascular disease is symptomatic with at least 50% occlusion. The primary peripheral symptom, due to ischemia, is: A. Edema (swelling) B. Calf pain C. Varicosities D. Strong pulse

B. Calf pain

While studying to become chemo-certified, the nurse reviews some basic concepts about cancer cells. When a client asks about why the tumor grows so fast, the nurse will respond based on which of the following physiological principles? Select the best answer: A. Cancer cells have shorter cell cycle times than normal cells. B. Cancer cells do not die when they are programmed to die. C. Cancer cells will reach a balance between cell birth and cell death rate. D. Cancer cells never reach a flattened growth rate.

B. Cancer cells do not die when they are programmed to die.

What type of necrosis is often associated with pulmonary tuberculosis? A. Bacteriologic necrosis B. Caseous necrosis C. Liquefactive necrosis D. Gangrenous necrosis

B. Caseous necrosis

A client with diabetes has impaired sensation, circulation, and oxygenation of his feet. He steps on a piece of glass, the wound does not heal, and the tissue area becomes necrotic. The necrotic cell death is characterized by: A. Rapid apoptosis B. Cellular breakage C. Shrinkage and collapse D. Chronic inflammation

B. Cellular breakage

What cancer does not have a known dietary risk factor? A. Colon B. Cervical C. Breast D. Lung

B. Cervical

A client presents for a scheduled Papanicolaou (Pap) smear. The clinician who will interpret the smear will examine cell samples for evidence of: A. The presence of unexpected cell types B. Changes in cell shape, size, and organization C. Ischemic changes in cell samples D. Abnormally high numbers of cells in a specified field

B. Changes in cell shape, size, and organization

Cor pulmonale is often caused by: A. Ischemia of heart muscle B. Chronic lung diseases, such as COPD C. Blood clots in the lower legs D. High blood pressure

B. Chronic lung diseases, such as COPD

A client with nonspecific signs/symptoms has gone to the primary health care provider. The client's chief complaints revolve around extreme fatigue, unplanned weight loss, and being so weak in the muscles. The diagnostic workup included a carcinoembryonic antigen (CEA) tumor marker. The CEA result was elevated. The nurse should anticipate the physician will order which of the following diagnostic tests related to the elevated CEA? A. Testicular ultrasound B. Colonoscopy C. Mammogram D. Brain CT

B. Colonoscopy

Aerobic metabolism, also known as oxidative metabolism, provides energy by: A. Removing the phosphate bonds from ATP B. Combining hydrogen and oxygen to form water C. Activating pyruvate stored in the cytoplasm D. Breaking down glucose to form lactic acid

B. Combining hydrogen and oxygen to form water

The client is found to have liver disease, resulting in the removal of a lobe of his liver. Adaptation to the reduced size of the liver leads to which phenomenon in the remaining liver cells? Select one: A. Metaplasia B. Compensatory hyperplasia C. Organ atrophy D. Physiologic hypertrophy

B. Compensatory hyperplasia

The nucleus , which is essential for function and survival of the cell A. Is the site of protein synthesis B. Contains the genetic code C. Transforms cellular energy D. Initiate aerobic metabolism

B. Contains the genetic code

Following a hypertensive crisis, a client's family asks, Why are the client's eyes so bloodshot? The nurse responds that high arterial pressure: A. From straining to have a bowel movement can rupture small vessels in the eyes B. Damages more fragile blood vessels like those in the eyes to the point of rupture C. Can interfere with clotting factor production, so clients bleed more easily D. Cause heart muscle to pump stronger, sending too much blood to small vessels in the eyes

B. Damages more fragile blood vessels like those in the eyes to the point of rupture

Which arteriole exits the nephron? A. Afferent B. Efferent C. Neither D. Both

B. Efferent

In the days following a tooth cleaning and root canal, a client has developed an infection of the thin, three-layered membrane that lines the heart and covers the valves. What is this client's most likely diagnosis? A. Pericarditis B. Endocarditis C. Myocarditis D. Vasculitis

B. Endocarditis

Chronic stable angina, associated with inadequate blood flow to meet the metabolic demands of the myocardium, is caused by: A. Increased collateral circulation B. Fixed coronary obstruction C. Increased collateral circulation D. You must have a genetic anomaly causing weakened alveolar sacs to rupture.

B. Fixed coronary obstruction

When evaluating the pulmonary function test results for a COPD client, which one correlates to the mismatch of ventilation and perfusion associated with this diagnosis? A. Forced vital capacity (FVC) is elevated. B. Forced expiratory volume (FEV) is decreased. C. Total lung capacity (TLC) is decreased. D. Marked decrease in residual volume (RV).

B. Forced expiratory volume (FEV) is decreased.

Splicing of mRNA during processing permits a cell to: Select one: A. Increase DNA content B. Form different proteins C. Stop copying DNA onto RNA D. Add nucleic acid end pieces

B. Form different proteins

Which of the following statements is true of glycolysis? A. Glycolysis requires oxygen B. Glycolysis occurs in cells without mitochondria C. Glycolysis provides the majority of the body's energy needs D. Glycolysis produces energy, water, and carbon dioxide

B. Glycolysis occurs in cells without mitochondria

The loop of _____________ plays an important role in controlling the concentration of the urine and is divided into three segments. A. Bowman B. Henle C. the juxtaglomerular complex 9or apparatus) D. Capillary

B. Henle

The highest risk for chronic hepatitis outcomes in infants < 1 year of age is due to which viral infection? A. Hepatitis A B. Hepatitis B C. Hepatitis C D. Delta virus

B. Hepatitis B

The mammary glands enlarge during pregnancy primarily as a consequence of: A. compensatory hyperplasia. B. hormonal hyperplasia. C. hormonal anaplasia. D. hormonal dysplasia.

B. Hormonal hyperplasia

A 51-year-old man has been diagnosed with chronic bronchitis after a long history of recurrent coughing. Which of the man's following statements demonstrates a sound understanding of his new diagnosis? A. I'm pretty sure that I first caught bronchitis from the person who has the cubicle next to mine at work. B. If I had quit smoking earlier than I did, I think I could have avoided getting bronchitis. C. I read on the Internet that I might have got bronchitis because I was born with an enzyme deficiency. D. I think that I probably could have prevented this if I had got in the habit of exercising more when I was younger.

B. If I had quit smoking earlier than I did, I think I could have avoided getting bronchitis.

A 1-day-old infant was exposed to an infectious microorganism prior to discharge home from the hospital, but was able to affect a sufficient immune response in the hours and days following exposure. The nurse knows that which of the following immunoglobulins assisted with this process? A. IgA B. IgG C. IgM D. IgD

B. IgG

An employee in your health department has had their Hepatitis profile drawn as part of their clearance to work in the clinic. Their HepBsAntigen is negative, HepB core Antibody (anti-HepBc) is negative and the HepBsAntibody (antiHepBs) is positive. You determine that he is: A. Susceptible to HepB infection B. Immune due to previous Hep B vaccination C. Actively infected with Hep B D. Chronically infected with Hep B

B. Immune due to previous Hep B vaccination

A client with diabetes and severe peripheral vascular disease has developed signs of dry gangrene on the great toe of one foot. The client asks, "How this can happen?" Which of the following pathophysiologic processes should the nurse explain to this client? "More than likely, your gangrene is caused by: Select one: A. Bacterial invasion into the foot and toe." B. Impaired arterial blood supply to your toe." C. Inappropriate activation of apoptosis, which means death of your cells." D. Metaplastic cellular changes in your toe."

B. Impaired arterial blood supply to your toe."

A febrile, 3-week-old infant has been brought to the emergency department by his parents and is currently undergoing a diagnostic workup to determine the cause of his fever. Which of the following statements best conveys the rationale for this careful examination? A. The immature hypothalamus is unable to perform normal thermoregulation. B. Infants are susceptible to serious infections because of their decreased immune function. C. Commonly used antipyretics often have no effect on the core temperature of infants. D. Fever in neonates is often evidence of a congenital disorder rather than an infection.

B. Infants are susceptible to serious infections because of their decreased immune function.

Which of the following individuals most likely has the highest risk of experiencing chronic inflammation? A client who: A. Has recently been diagnosed with type 2 diabetes B. Is morbidly obese and who has a sedentary lifestyle C. Is a carrier of an antibiotic-resistant organism D. Is taking oral antibiotics for an upper respiratory infection

B. Is morbidly obese and who has a sedentary lifestyle

Which describes the function of the nucleus? A. It is basically the site of protein synthesis in the body. B. It contains the genetic code for the individual. C. It is the transformer of cellular energy. D. It initiates the process of aerobic metabolism.

B. It contains the genetic code for the individual.

What type of necrosis is associated with wet gangrene? A. Coagulative necrosis B. Liquefactive necrosis C. Caseous necrosis D. Gangrene necrosis

B. Liquefactive necrosis

Which of the following aspects of kidney function is performed by the juxtaglomerular apparatus? A. Regulating urine concentration B. Matching changes in GFR with renal blood flow C. Regulating sodium and potassium elimination D. Facilitating active transport to reabsorb electrolytes

B. Matching changes in GFR with renal blood flow

What is the name of the method of transport that uses transmembrane proteins with receptors that have a high degree of specificity for the substance being transported? A. Active transport B. Mediated transport C. Transmembranous transport D. Passive transport

B. Mediated transport

Which type of cell adaptation occurs when normal columnar ciliated epithelial cells of the bronchial lining have been replaced by stratified squamous epithelial cells? A. Hyperplasia B. Metaplasia C. Dysplasia D. Anaplasia

B. Metaplasia

Reduced glomerular filtration rate (GFR), with a serum creatinine level that remains in the normal range, is associated with aging because elderly persons tend to have reduced: A. Calcium intake B. Muscle mass C. Drug tolerance D. Renal perfusion

B. Muscle mass

A client asks why his temperature is always below 98.6°F. The nurse responds: A. Some people maintain a core body temperature of 41°C and that is normal for them. B. Normal core temperature varies between individuals within the range of 97.0°F to 99.5°F. C. A person's highest point of core temperature is usually first thing in the morning. D. The best way to bring your body temperature up to normal is to live in a warmer climate.

B. Normal core temperature varies between individuals within the range of 97.0°F to 99.5°F.

A public health nurse has cited a reduction in cancer risk among the many benefits of maintaining a healthy body mass index. Which of the following facts underlies the relationship between obesity and cancer? Select one: A. Increased numbers of body cells increase the statistical chances of neoplastic cell changes. B. Obesity can cause inflammation and hormonal changes that are associated with cancer. C. Increased cardiac workload and tissue hypoxia can interfere with normal cell differentiation. D. Adipose tissue is more susceptible to malignancy than other types of connective tissue.

B. Obesity can cause inflammation and hormonal changes that are associated with cancer.

Which component of the cell produces hydrogen peroxide (H2O2) by using oxygen to remove hydrogen atoms from specific substrates in an oxidative reaction? A. Lysosomes B. Peroxisome C. Ribosomes D. Oxyhydrosomes

B. Peroxisome

Exogenous pyrogens (interleukin-1) and the presence of bacteria in the blood lead to the release of endogenous pyrogens that: A. Stabilize thermal control in the brain B. Produce leukocytosis and shivering C. Block viral replication in cells D. Inhibit prostaglandin release

B. Produce leukocytosis and shivering

A client with a history of emphysema from long-term cigarette smoking has loss of many alveoli. When comparing the type I alveolar cell physiologic function with the primary role of type II alveoli, the nurse would be aware that the type II alveoli are responsible for: A. Facilitation of bronchial circulation B. Production of surfactant C. Gas exchange D. Production of macrophages

B. Production of surfactant

A client is suspected of having the onset of alcoholic liver disease. The nurse should be assessing for which of the following manifestations related to the necrosis of liver cells? A. Tremors of the hands (asterixis) B. Rapid onset of jaundice C. Long muscle group atrophy D. Development of multiple skin nodules

B. Rapid onset of jaundice

The parents of a 4-year-old girl have sought care because their daughter has admitted to chewing and swallowing imported toy figurines that have been determined to be made of lead. Which of the following blood tests should the care team prioritize? A. White blood cell levels with differential B. Red blood cell levels and morphology C. Urea and creatinine levels D. Liver function panel

B. Red blood cell levels and morphology

The parents of a 4-year-old girl have sought care because their daughter has admitted to chewing and swallowing imported toy figurines that have been determined to be made of lead. Which of the following blood tests should the care team prioritize? A. White blood cell levels with differential B. Red blood cell levels and morphology C. Urea and creatinine levels D. Liver function panel

B. Red blood cell levels and morphology

A client has been brought to the emergency department with a large, gaping wound from a farming accident. The client is critically ill and has required blood products and surgery to clean and close the wound. Sharing with a student nurse the steps in wound healing, the nurse discusses the inflammatory phase, stating macrophages: Select all that apply.

B. Remove debris from the wound C. Release growth factors that stimulate epithelial cell growth D. Facilitate the body's ability to grow new vessels in the injured area

Which of the following diseases would be considered to be caused by a lack of a specific vitamin or mineral? Select one: A. Atherosclerosis B. Scurvy C. Sickle cell anemia D. Anorexia nervosa

B. Scurvy

Although clinical manifestations vary with the type of cancer and organs involved, the oncology nurses have observed that the most frequent side effects clients with cancer experience are: A. Copious lymph flow B. Sleep disturbances C. Involuntary weight gain D. Organ expansion

B. Sleep disturbances

What causes the rapid change in the resting membrane potential that initiates an action potential? A. Potassium gates open and potassium rushes into the cell changing the membrane potential from negative to positive. B. Sodium gates open and sodium rushes into the cell changing the membrane potential from negative to positive. C. Sodium gates close allowing potassium into the cell to changing the membrane potential from positive to negative. D. Potassium gates close allowing sodium into the cell to change the membrane potential from positive to negative.

B. Sodium gates open and sodium rushes into the cell changing the membrane potential from negative to positive

The plaques in a client's coronary arteries are plentiful, and most have small- to moderate-sized lipid cores with thick fibrous caps. This form of atherosclerosis is most closely associated with which of the following diagnoses? A. NonST-segment elevation MI B. Stable angina C. ST-segment elevation MI D. Unstable angina

B. Stable angina

Connective tissue contains fibroblasts that are responsible for: A. Providing a fibrous framework for capillaries B. Synthesis of collagen, elastin, and reticular fibers C. Forming tendons and the fascia that covers muscles D. Filling spaces between tissues to keep organs in place

B. Synthesis of collagen, elastin, and reticular fibers

A client is admitted to hospital to rule out Legionnaire disease following a canoe trip where he was sprayed in the face with a lot of creek water. Which of the following manifestations are characteristic of Legionnaire pneumonia? S A. Productive cough with thick, yellow secretions B. Temperature of 103.5°, pulse 80 C. Temperature of 103.5°, pulse 140 D. Normal chest Xray

B. Temperature of 103.5°, pulse 80

What determines the permeability of the glomerular capillary membrane? A. Glomerular blood flow B. The basement membrane C. Bowman's capsule D. The juxtaglomerular cells

B. The basement membrane

A client with poor arterial circulation in the lower limbs has developed areas of inflammation and weeping clear serous exudate. Since chronic inflammation lasts for a long time, it has been associated with which of the following changes in physiological response? A. Loss of ability to develop new blood vessels B. The death of one or more cells in the body within a localized area C. Release of a number of potent inflammatory mediators, altering adhesive properties D. Regulation and modulation of the immune response through synthesis and release of inflammatory mediators E. Release of scavenger cells capable of engulfing bacteria through phagocytosis

B. The death of one or more cells in the body within a localized area

Which of the following is an accurate characteristics of prions? A. Disease usually progresses quickly. B. The primary manifestation includes ataxia and dementia. C. Prions are protease sensitive. D. Prions lack reproductive functions so are not very harmful to humans.

B. The primary manifestation includes ataxia and dementia.

A college student is training for a marathon in the mountains. One day, she experiences a sharp pain and suddenly becomes short of breath. At the emergency room, chest x-ray reveals a spontaneous pneumothorax. The client asks the nurse to explain why this happened. The nurse states, For unknown reasons, you lose intrapleural negative pressure. A. You must have experienced a forced expiration against a closed glottis to cause the lung to deflate. B. This means your lungs collapsed and expelled its air when you lose negative pressure. C. You must have coughed too forcibly and your air sacs burst. D. You must have a genetic anomaly causing weakened alveolar sacs to rupture.

B. This means your lungs collapsed and expelled its air when you lose negative pressure.

The most important complication of atherosclerosis that may cause occlusion of small heart vessels is: A. Ulceration B. Thrombosis C. Fatty streaks D. Fibrous plaque

B. Thrombosis

A man sustained a puncture injury to his chest that caused a tension pneumothorax to form. This is a life-threatening condition because: A. Expired air exits the bleeding wound. B. Trapped, inspired air collapses the lung. C. The opposite lung hyperinflates. D. Blebs on the lung surface rupture.

B. Trapped, inspired air collapses the lung.

The nurse caring for a population of HIV clients needs to be assessing for which of the following diseases that amounts to the leading cause of death for people with HIV? A. Leukemia B. Tuberculosis C. Pneumonia D. Toxoplasmosis

B. Tuberculosis

A client cuts herself with a sharp knife while cooking dinner. The client describes how the wound started bleeding and had a red appearance almost immediately. The nurse knows that in the vascular stage of acute inflammation, the vessels: A. Bleed profusely until the body can compensate and start to send fibrinogen to the wound B. Vasodilate causing the area to become congested causing the red color and warmth C. Vasodilate causing the area to become congested causing the red color and warmth D. Swell to the point of compromising circulation causing the limb to become cool to touch

B. Vasodilate causing the area to become congested causing the red color and warmth

Which of the following statements about vascular compliance is accurate? A. Arteries are much more distensible than veins. B. Veins can act as a reservoir for storing large quantities of blood. C. Arteries have thick muscular walls that constrict tightly, thereby ejecting blood without storing it for later use. D. A continuous flow through the capillaries occurs primarily during systole.

B. Veins can act as a reservoir for storing large quantities of blood.

An older mother (age 41) is worried about having a baby with birth defects. She wants to get tested so she can be prepared for the outcome. Which of the following tests should the clinic nurse prepare the client for? Select all that apply.

B. Withdrawing a sample of amniotic fluid will reveal any chromosomal defects. E. A blood test (circulating cell-free DNA) will tell if the baby is mentally retarded

A client asks, Why do I have clogged arteries but my neighbor has higher 'bad cholesterol' levels and yet he is just fine? The health care provider bases the reply on which of the following physiological principles about lipoprotein? A. Your neighbor probably has higher amounts of good cholesterol (HDL) as well. B. You more than likely have small, dense type of 'bad cholesterol' (LDL). C. Your neighbor has larger 'bad cholesterol' particles that can move into blood vessels but park in joints/tendons. D. You must have a genetic predisposition to having clogged arteries.

B. You more than likely have small, dense type of 'bad cholesterol' (LDL).

When a second message is necessary for extracellular communication to be activated, it is provided by: A. guanosine triphosphate (GTP). B. adenosine monophosphate (AMP). C. adenosine triphosphate (ATP). D. guanosine diphosphate (GDP).

B. adenosine monosphosphate

The chief reason for liver transplantation int he US is: A. alcoholic liver disease B. chronic viral hepatitis C. non-alcoholic steatohepatitis (NASH) D. acetominophen (Tyloenol) overdose

B. chronic viral hepatitis

Serum _____________ reflects the GFR (glomerular filtration rate) and can be used as an estimate of renal function. A. protein B. creatinine C. blood glucose D. urates

B. creatinine

Under anaerobic conditions, the process of _____ provides energy for the cell. A. oxidative phosphorylation B. glycolysis C. lactolysis D. passive transport

B. glycolysis

After several months of trying to conceive, a couple is undergoing fertility testing. Semen analysis indicates that the man's sperms are energetically active but have decreased motility, a finding that thought to underlie the couple's inability become pregnant. Which of the following cellular components may be defective within the man's sperm? A. Ribosomes B. microtubules C. Mitochondria D. Microfilaments

B. microtubules

Portal circulation involves blood draining from the following, among other sources: A. lower extremities B. pancreas C. kidneys D. lungs

B. pancreas

Nephrotic syndrome is associated with increased glomerular permeability to plasma ____________, resulting in generalized edema. A. sodium B. protein C. glucose D. lipids

B. protein

The liver can be distinguished from the other visceral organs due to its ability to A. hypertrophy B. regenerate C. compensate D. atrophy

B. regenerate

Both prehepatic and posthepatic causes of portal hypertension include the formation of: A. fibrous nodules. B. venous thromboses or obstructions. C. collateral circulation. D. portosystemic shunts.

B. venous thromboses or obstructions.

Following several weeks of increasing fatigue and a subsequent diagnostic workup, a client has been diagnosed with mitral valve regurgitation. Failure of this heart valve would have which of the following consequences?

Backflow from the left ventricle to the left atrium

Which one of the following infectious agents contains DNA and RNA

Bacteria

Bacterium

Bacteria in the bloodstream.

The results of a 44 year old obese man's recent diagnostic work up have culminated in a new diagnosis of type 2 diabetes. Which of the following pathophysiologic processes the client's new diagnosis?

Beta cell exhaustion due to long-standing insulin resistance

As other mechanisms prepare to respond to a pH imbalance, immediate buffering is a result of increased:

Bicarbonate/carbonic acid regulation

A client who had his gallbladder removed asks the nurse, "Why do I feel sick every time I go through a drive in fast food restaurant and order a burger and fries?" The nurse bases the response knowing that:

Bile from the GB is secreted to breakdown lipids

A client presents to the emergency clinic not feeling well. Which of the following complaints leads the health care provider to suspect the client may have acute leukemia?

Bleeding from the gums, not related to brushing the teeth

Hypoparathyroidism causes hypocalcemia by:

Blocking release of calcium from bone

From the following list of physical assessment results, which would be interpreted as the client has developed obesity?

Body mass index (BMI) of 40

The most reliable method for measuring body water or fluid volume increase is by assessing:

Body weight change

While taking a client history, which of the following findings may lead the nurse to suspect the client is at risk for development of osteonecrosis?

Bone marrow ischemia due to radiation therapy for cancer

A client with a complete tear of the rotator cuff in the right shoulder was given the choice between surgery and stem cell transplant. The client chose to try the stem cell injection. The client asked the health care provider, "How does this work on my shoulder?" The best response would be:

Bone marrow stem cells have been shown to generate cartilage, bone, and muscle when injected directly into the injured site.

Client is experiencing the early stages of multiple myeloma. What is the earliest symptom?

Bone pain

Although the basic structure of the cell plasma membrane is formed by a lipid bilayer, most of the specific membrane functions are carried out by

Bound & transmembrane proteins

Although the basic structure of the cell plasma membrane is formed by a lipid bilayer, most of the specific membrane functions are carried out by:

Bound and transmembrane proteins

A client presents to the clinic with a swollen, painful "hang nail" on the index finger. There is a large pustule over the site that needs to be lanced. The health care worker knows that which mediator of inflammation causes this increase in capillary permeability and pain?

Bradykinin

Client presents to the clinic with a swollen, painful "hang nail" on the index finger. There is a large pustule over the site that needs to be lanced. The health care worker knows that which chemical mediator of inflammation promotes capillary permeability and causes pain?

Bradykinin

A teenager has been in a car accident and experienced acceleration-deceleration head injury. Initially, the client was stable but then started to develop neurological signs/symptoms. The nurse caring for this client should be assessing for which type of possible complication?

Brain contusions and hematomas

Genetic screening may be indicated for individuals who have a family history of which of the following neoplasms?

Breast cancer

A breast cancer client has just learned that her tumor clinical stage is T3, N2, M0. After the physician leaves, the client asks the nurse to explain this to her again. The nurse will use which of the following statements in his or her answer? Your: A. Tumor has metastasized to at least three distal sites and you have cancer in your lymph nodes. B. Tumor is very small and has zero number of mitoses. C. Tumor is large and at least two lymph nodes are positive for cancer cells. D. Extent of disease is unknown, but it looks like your cancer has stayed intact and not spread to the bloodstream.

C

A lung biopsy and magnetic resonance imaging have confirmed the presence of a benign lung tumor in a client. Which of the following characteristics is associated with this client's neoplasm? A. The tumor is likely to infiltrate the lung tissue that presently surrounds it. B. The cells that constitute the tumor are undifferentiated, with atypical structure. C. The tumor will grow by expansion and is likely encapsulated. D. If left untreated, the client's tumor is likely to metastasize.

C

An 84-year-old client's blood cultures have come back positive, despite the fact that his oral temperature has remained within normal range. Which of the following phenomena underlies the alterations in fever response that occur in the elderly? A. Increased heat loss by evaporation B. The presence of comorbidities that is associated with lowered core temperature C. Disturbance in the functioning of the thermoregulatory center D. Persistent closure of arteriovenous shunts

C

A client has experienced significant decreases in mobility and stamina during a 3-week hospital stay for the treatment of a femoral head fracture. Which of the following phenomena most likely accounts for the client's decrease in muscle function? A. Dysplasia as a consequence of inflammation during bone remodeling B. Impaired muscle cell metabolism resulting from metaplasia C. Disuse atrophy of muscle cells during a prolonged period of immobility D. Ischemic atrophy resulting from vascular changes while on bedrest

C)

A couple has chosen to pay for the harvesting and storage of umbilical cord blood after the delivery of their child to secure a future source of embryonic stem cells. What is the most likely rationale for the couple's decision? A. The stem cells can change the proliferative capacity of other cells. B. The embryonic stem cells remove cyclin-dependent kinase inhibitors from the body. C. The stem cells may be able to produce a wide range of body cells. D. The embryonic stem cells allow stable and permanent tissues to enter mitosis.

C)

The clinician who will interpret the smear will examine cell samples for evidence of: A. Ischemic changes in cell samples B. The presence of unexpected cell types C. Changes in cell shape, size, and organization D. Abnormally high numbers of cells in a specified field

C)

7. Reversible adaptive intracellular responses are initiated by: A) stimulus overload. B) genetic mutations. C) chemical messengers. D) mitochondrial DNA.

C) Chemical messengers

The client is found to have liver disease, resulting in the removal of a lobe of his liver. Adaptation to the reduced size of the liver leads to which phenomenon in the remaining liver cells? A) Metaplasia B) Organ atrophy C) Compensatory hyperplasia D) Physiologic hypertrophy

C) Compensatory hyperplasia Feedback: Compensatory hyperplasia can be stimulated in response to loss of vital tissue that is capable of regeneration, such as liver cells. Metaplasia involves replacement of one existing cell type with another fully differentiated cell type. Organ atrophy is caused by irreversible loss of cells. Physiologic hypertrophy is increased size of existing cells resulting from increased workload.

A client has experienced significant decreases in mobility and stamina during a 3-week hospital stay for the treatment of a femoral head fracture. Which of the following phenomena most likely accounts for the client's decrease in muscle function? A) Impaired muscle cell metabolism resulting from metaplasia B) Dysplasia as a consequence of inflammation during bone remodeling C) Disuse atrophy of muscle cells during a prolonged period of immobility D) Ischemic atrophy resulting from vascular changes while on bedrest

C) Disuse atrophy of muscle cells during a prolonged period of immobility Feedback: Disuse atrophy frequently occurs as a consequence of prolonged periods of muscle inactivity. Metaplasia and dysplasia are not common consequences of immobility and muscle disuse. Similarly, infrequent muscle use does not typically cause vascular changes that result in ischemic atrophy.

11. An 89-year-old female patient has experienced significant decreases in her mobility and stamina during a 3-week hospital stay for the treatment of a femoral head fracture. Which of the following phenomena most likely accounts for the patient's decrease in muscle function that underlies her reduced mobility? A) Impaired muscle cell metabolism resulting from metaplasia B) Dysplasia as a consequence of inflammation during bone remodeling C) Disuse atrophy of muscle cells during a prolonged period of immobility D) Ischemic atrophy resulting from vascular changes while on bedrest

C) Disuse atrophy of muscle cells during prolonged period of immobility

When performing an assessment on a school-aged child, the nurse notes that the mucous membranes along the gum margins have a noticeable blue-colored line. At this point, the nurse should ask the parents about possible: A) Liver problems as an infant B) Congenital heart problems C) Exposure to lead D) Second-hand smoke exposure

C) Exposure to lead Feedback: The formation of a blue lead line along the margins of the gum is one of the diagnostic features of lead poisoning. Liver problems are usually displayed as jaundice (yellowing of the skin/sclera). Congenital heart problems may exhibit cyanosis, but this would not be just in the gum margins. Second-hand smoke exposure may cause accumulation in the lungs.

19. A patient with severe peripheral vascular disease has developed signs of dry gangrene on the great toe of one foot. Which of the following pathophysiologic processes most likely contributed to this diagnosis? A) Inappropriate activation of apoptosis B) Bacterial invasion C) Impaired arterial blood supply D) Metaplastic cellular changes

C) Impaired arterial blood supply

A client with diabetes and severe peripheral vascular disease has developed signs of dry gangrene on the great toe of one foot. The client asks, "How this can happen?" Which of the following pathophysiologic processes should the nurse explain to this client? "More than likely, your gangrene is caused by: A) Inappropriate activation of apoptosis, which means death of your cells." B) Bacterial invasion into the foot and toe." C) Impaired arterial blood supply to your toe." D) Metaplastic cellular changes in your toe."

C) Impaired arterial blood supply to your toe."

A client with diabetes and severe peripheral vascular disease has developed signs of dry gangrene on the great toe of one foot. The client asks, "How this can happen?" Which of the following pathophysiologic processes should the nurse explain to this client? "More than likely, your gangrene is caused by: A) Inappropriate activation of apoptosis, which means death of your cells." B) Bacterial invasion into the foot and toe." C) Impaired arterial blood supply to your toe." D) Metaplastic cellular changes in your toe."

C) Impaired arterial blood supply to your toe." Feedback: Dry gangrene is often a result of impaired arterial blood supply to the extremities. A bacterial etiology is more common in wet gangrene, whereas neither metaplasia nor activation of apoptosis is implicated in cases of dry gangrene.

Despite the low levels of radiation used in contemporary radiologic imaging, a radiology technician is aware of the need to minimize her exposure to ionizing radiation. What is the primary rationale for the technician's precautions? Radiation: A) Stimulates pathologic cell hypertrophy and hyperplasia B) Results in the accumulation of endogenous waste products in the cytoplasm C) Interferes with DNA synthesis and mitosis D) Decreases the action potential of rapidly dividing cells

C) Interferes with DNA synthesis and mitosis Feedback: Radiation has a damaging effect on DNA synthesis and mitosis, a process that is especially harmful to rapidly dividing cells. Radiation does not directly influence the action potential of cells or the accumulation of endogenous waste products. Cell changes such as hypertrophy or hyperplasia may result from radiation exposure, but such changes are secondary to interference with DNA synthesis and mitosis.

A mother rushes her 4-year-old child to the emergency department after she found an empty Tylenol (acetaminophen) bottle beside her child. The nurse is trying to explain why it is so important to give the child Ipecac to induce vomiting in order to prevent: A) Renal failure B) Seizures C) Liver failure D) Hemorrhage

C) Liver failure Feedback: Acetaminophen, a commonly used over-the-counter analgesic drug, is detoxified in the liver, where small amounts of the drug are converted to a highly toxic metabolite. This metabolite is detoxified by a metabolic pathway that uses a substance normally present in the liver. When large amounts of the drug are ingested, this pathway becomes overwhelmed and toxic metabolites accumulate, causing massive liver necrosis.

14. Despite the low levels of radiation used in contemporary radiologic imaging, a radiology technician is aware of the need to minimize her exposure to ionizing radiation. What is the primary rationale for the technician's precautions? A) Radiation stimulates pathologic cell hypertrophy and hyperplasia. B) Radiation results in the accumulation of endogenous waste products in the cytoplasm. C) Radiation interferes with DNA synthesis and mitosis. D) Radiation decreases the action potential of rapidly dividing cells.

C) Radiation interferes with DNA synthesis and mitosis.

A client has been diagnosed with a gram-negative bacillus in his blood cultures. The health care providers know these bacteria may produce clinical manifestations such as high temperature, high respiratory rate, and low blood pressure. These manifestations are primarily caused by: A) Disrupting the sodium/potassium A TPase pump B) Interrupting oxidative metabolism processes C) The outer layer of the bacterial membrane acting as an endotoxin D) The bacteria causing a decrease in protein synthesis and function

C) The outer layer of the bacterial membrane acting as an endotoxin

A client has been diagnosed with a gram-negative bacillus in his blood cultures. The health care providers know these bacteria may produce clinical manifestations such as high temperature, high respiratory rate, and low blood pressure. These manifestations are primarily caused by: A) Disrupting the sodium/potassium ATPase pump B) Interrupting oxidative metabolism processes C) The outer layer of the bacterial membrane acting as an endotoxin D) The bacteria causing a decrease in protein synthesis and function

C) The outer layer of the bacterial membrane acting as an endotoxin Feedback: Bacteria and viruses can replicate within a cell, thus perpetuating the injuries. Gram-negative bacilli have unique characteristics in the structure of the outer membrane. The outer leaflet of the membrane has a lipid portion that acts as an endotoxin. If this bacillus enters the circulatory system, it causes a toxic reaction, with the sufferer developing a high temperature, high respiration rate, and low blood pressure. Other agents that are injurious to cells are unable to replicate in the cell, but they may disrupt the sodium/potassium pump, interrupt oxidative metabolism, or decrease protein synthesis.

2. The patient is found to have liver disease, resulting in the removal of a lobe of his liver. Adaptation to the reduced size of the liver leads to ___________ of the remaining liver cells. A) metaplasia B) organ atrophy C) compensatory hyperplasia D) physiologic hypertrophy

C) compensatory hyperplasia

4. Bacteria and viruses cause cell damage by _______, which is unique from the intracellular damage caused by other injurious agents. A) disrupting the sodium/potassium ATPase pump B) interrupting oxidative metabolism processes C) replicating and producing continued injury D) decreasing protein synthesis and function

C) replicating and producing continued injury

Following routine colonoscopy screening, a client is told that he had several polyps removed. The client began crying stating, "I just can't deal with cancer. I'm too young." The nurse responds: Select one: A. "Don't worry. We have some great cancer doctors on staff. I'm sure chemo will help you fight it." B. "Maybe if you're lucky, they have stopped it from metastasizing to your liver." C. "Most colon polyps are not cancerous. The biopsy results will direct your care." D. "A simple intestinal surgery will cure you."

C. "Most colon polyps are not cancerous. The biopsy results will direct your care.

Which of the following dietary guidelines should a nurse provide to a group of older adults to possibly decrease their risks of developing colon cancer? A. "As much as possible, try to eat organic foods." B. "Regular vitamin supplements and a low-carbohydrate diet are beneficial." C. "Try to minimize fat and maximize fiber when you're planning your meals." D. Eat enough fiber in your diet that you have bowel movement at least once daily.

C. "Try to minimize fat and maximize fiber when you're planning your meals."

How many nephrons are in a normal human body? A. 500,000 B. 1 million C. 2 million D. 4 million

C. 2 million

A client has had a myocardial infarction (MI) that damaged the right atrium, which has interfered with the SA node. The compensatory mechanism, the AV node, becomes the pacemaker of the heart and beats how many times/minute? A. 10 to 20 beats/minute B. 21 to 30 beats/minute C. 45 to 50 beats/minute D. 55 to 60 beats/minute

C. 45 to 50 beats/minute

When discussing the AV node's role in the electrical conduction of the heart with a client newly diagnosed with an AV block, which of the following statements are accurate? A. The AV node offers a two-way conduction area between the atria and the ventricles. B. The velocity of conduction through the AV junctional fibers is very fast, which greatly increases impulse transmission. C. A block at the AV bundle of His interferes with the normal delay of the impulse, thereby interfering with complete ejection of blood from the atria prior to ventricular contraction. D. When there is an AV block, impulses from the atria and ventricles beat independently of each other so, the heart rhythm is regular with a normal rate.

C. A block at the AV bundle of His interferes with the normal delay of the impulse, thereby interfering with complete ejection of blood from the atria prior to ventricular contraction.

Which of the following clients have a pathophysiologic process capable of causing fever by inducing the production of pyrogens? Select all that apply.

C. A newly diagnosed Hodgkin lymphoma client D. A farmer who cut his arm while sharpening his tools coming to clinic because of acute inflammation signs like fever and redness E. An older adult recuperating following a myocardial infarction

A client had excessive blood loss and prolonged hypotension during surgery. His postoperative urine output is sharply decreased, and his blood urea nitrogen (BUN) is elevated. The most likely cause for the change is acute: A. Prerenal inflammation B. Bladder outlet obstruction C. Acute tubular necrosis D. Intrarenal nephrotoxicity

C. Acute tubular necrosis

When should women begin screening for cervical cancer? A. At age 18 B. Within 3 years of onset of sexual activity. C. Age 21 D. When parent consents

C. Age 21

Which of the following clients should most likely be assessed for orthostatic hypotension? A. A 78-year-old woman who has begun complaining of frequent headaches unrelieved by over-the-counter analgesics B. A 65-year-old client whose vision has become much less acute in recent months and who has noticed swelling in her an C. An 80-year-old elderly client who has experienced two falls since admission while attempting to ambulate to the bathroom D. A 42-year-old client who has a history of poorly controlled type 1 diabetes

C. An 80-year-old elderly client who has experienced two falls since admission while attempting to ambulate to the bathroom

A child has experienced a bee-sting while at the park. The health care provider is walking by and notices the child has swelling around the eyes, lips, and face in general. What priority assessment should the nurse make at this time? A. Palpate for carotid pulses in the neck. B. Assess skin on the truck and back for development of hives. C. Assess and establish an open airway. D. Try to listen to breath sounds by placing your ear on the child's chest.

C. Assess and establish an open airway.

During cell injury caused by hypoxia, why is there an increase in the osmotic pressure within the cell? A. Because plasma proteins enter the cell B. Because the ATPase-driven pump is stronger during hypoxia C. Because sodium chloride enters the cell D. Because there is an influx of glucose through the injured cell membranes

C. Because sodium chloride enters the cell

Individuals with liver disease often experience the effects of excess serum ammonia as a result of impairment of the liver's ability to process ammonia. How does the liver process ammonia in healthy individuals? A. By converting it into bilirubin which is then excreted intestinally. B. By processing ammonia into nitrogen and hydrogen ions for excretion. C. By processing it into urea and releasing it into the circulation. D. By combining it with oxygen to create ammonium oxide.

C. By processing it into urea and releasing it into the circulation.

During ischemia, what effect does the loss of adenosine triphosphate (ATP) level have on cells? A. Cells shrink because of the influx of calcium (Ca). B. Cells shrink because of the influx of potassium chloride (KCl). C. Cells swell because of the influx of sodium chloride (NaCl). D. Cells swell because of the influx of nitric oxide (NO).

C. Cells swell because of the influx of sodium chloride (NaCl)

A patient who has diarrhea receives a 3% saline solution intravenously to replace the sodium and chloride lost in the stool. What effect will this fluid replacement have on cells? A. Cells will become hydrated. B. Cells will swell or burst. C. Cells will shrink. D. Cells will divide.

C. Cells will shrink

A hospital client was swabbed on admission for antibiotic-resistant organisms and has just been informed that methicillin-resistant Staphylococcus aureus (MRSA) is present in his groin. The client has a normal core temperature and white blood cell count. This client is experiencing: A. Infection B. Proliferation C. Colonization D. Inflammation

C. Colonization

The client is found to have liver disease, resulting in the removal of a lobe of his liver. Adaptation to the reduced size of the liver leads to which phenomenon in the remaining liver cells? A. Metaplasia B. Organ atrophy C. Compensatory hyperplasia D. Physiologic hypertrophy

C. Compensatory hyperplasia

Contact with poison ivy has resulted in intense pruritus, erythema, and weeping on a client's forearm. Which of the following processes resulted in the client's signs and symptoms? A. IgE-mediated mast cell degranulation B. Formation of antigenantibody complexes C. Cytokine release by sensitized T cells D. Formation of antibodies against cell surface antigens

C. Cytokine release by sensitized T cells

A large increase in heart rate can cause: A. Increased blood viscosity B. Loss of action potential C. Decreased stroke volume D. Reduced cardiac contractility

C. Decreased stroke volume

Which cell component is the most vulnerable target of radiation? A. Plasma membrane B. Mitochondria C. Deoxyribonucleic acid (DNA) D. Golgi body

C. Deoxyribonucleic acid (DNA)

A client has experienced significant decreases in mobility and stamina during a 3-week hospital stay for the treatment of a femoral head fracture. Which of the following phenomena most likely accounts for the client's decrease in muscle function? A. Impaired muscle cell metabolism resulting from metaplasia B. Dysplasia as a consequence of inflammation during bone remodeling C. Disuse atrophy of muscle cells during a prolonged period of immobility D. Ischemic atrophy resulting from vascular changes while on bedrest

C. Disuse atrophy of muscle cells during a prolonged period of immobility

A client has experienced significant decreases in mobility and stamina during a 3-week hospital stay for the treatment of a femoral head fracture. Which of the following phenomena most likely accounts for the client's decrease in muscle function? A. Impaired muscle cell metabolism resulting from metaplasia B. Dysplasia as a consequence of inflammation during bone remodeling C. Disuse atrophy of muscle cells during a prolonged period of immobility D. Ischemic atrophy resulting from vascular changes while on bedrest

C. Disuse atrophy of muscle cells during a prolonged period of immobility

A 9-month-old infant has been diagnosed with botulism after he was fed honey. The child's mother was prompted to seek care because of this child's sudden onset of neuromuscular deficits, which were later attributed to the release of substances by Clostridium botulinum. Which virulence factor contributed to this child's illness? A. Endotoxins B. Adhesion factors C. Exotoxins D. Evasive factors

C. Exotoxins

When performing an assessment on a school-aged child, the nurse notes that the mucous membranes along the gum margins have a noticeable blue-colored line. At this point, the nurse should ask the parents about possible: A. Liver problems as an infant B. Congenital heart problems C. Exposure to lead D. Second-hand smoke exposure

C. Exposure to lead

When performing an assessment on a school-aged child, the nurse notes that the mucous membranes along the gum margins have a noticeable blue-colored line. At this point, the nurse should ask the parents about possible: A. Liver problems as an infant B. Congenital heart problems C. Exposure to lead D. Second-hand smoke exposure

C. Exposure to lead

When performing an assessment on a school-aged child, the nurse notes that the mucous membranes along the gum margins have a noticeable blue-colored line. At this point, the nurse should ask the parents about possible: Select one: A. Liver problems as an infant B. Second-hand smoke exposure C. Exposure to lead D. Congenital heart problems

C. Exposure to lead

Although energy is not made in mitochondria, they are known as the "power plants" of the cell because they A. Contain RNA for protein synthesis B. Utilize glucose for oxidative energy C. Extract energy from organic compounds D. Store calcium bonds for muscle contractions

C. Extract energy from organic compounds

Which of the following facts underlies the application of RNA interference in the treatment of disease? Select one: A. Individual differences are attributable to a very small percentage of the genes in the human body. B. It is possible to produce proteins that have therapeutic properties. C. Faulty gene activity that produces unwanted proteins can sometimes be stopped. D. Restriction enzymes are able to cleave genetic molecules at predictable sites.

C. Faulty gene activity that produces unwanted proteins can sometimes be stopped.

Epithelial tissues are classified according to the shape of the cells and the number of layers. Which of the following is a correctly matched description and type of epithelial tissue? A. Simple epithelium: cells in contact with intercellular matrix; some do not extend to surface B. Stratified epithelium: single layer of cells; all cells rest on basement membrane C. Glandular epithelium: arise from surface epithelia and underlying connective tissue D. Pseudostratified epithelium: multiple layers of cells; deepest layer rests on basement membrane

C. Glandular epithelium: arise from surface epithelia and underlying connective tissue

Epithelial tissues are classified according to the shape of the cells and the number of layers. Which of the following is a correctly matched description and type of epithelial tissue? A. Simple epithelium: cells in contact with the intercellular matrix; some do not extend to surface. B. Stratified epithelium: single layer of cells; all cells rest on the basement membrane. C. Glandular epithelium: arises from surface epithelia and underlying connective tissue. D. Pseudostratified epithelium: multiple layers of cells; the deepest layer rests on the basement membrane.

C. Glandular epithelium: arises from surface epithelia and underlying connective tissue.

The angiogenesis process, which allows tumors to develop new blood vessels, is triggered and regulated by tumor-secreted: Select one: A. Procoagulants B. Proteolytic enzymes C. Growth factors D. Attachment factors

C. Growth factors

Some viruses have the ability to transform host cells into cancer cells. For which of the following viruses should the client be assessed regularly for the potential development of cancer? Select all that apply. A. Hepatitis A B. Hepatitis E C. Human papillomavirus D. Varicella

C. Human papillomavirus

Which of the following pathophysiologic processes is most likely to result in metastatic calcification? Select one: A. Liver cirrhosis B. Benign prostatic hyperplasia C. Hyperparathyroidism D. Impaired glycogen metabolism

C. Hyperparathyroidism

The nurse is providing care for a client who has a diagnosis of kidney failure. Which of the following laboratory findings is consistent with this client's diagnosis? A. Elevation in vitamin D levels B. Hypophosphatemia C. Hypocalcemia D. Hypokalemia

C. Hypocalcemia

A 70-year-old male client has been admitted to a hospital for the treatment of a recent hemorrhagic stroke that has left him with numerous motor and sensory deficits. These deficits are most likely the result of which of the following mechanisms of cell injury? A. Free radical injury B. Interference with DNA synthesis C. Hypoxia and ATP depletion D. Impaired calcium homeostasis

C. Hypoxia and ATP depletion

A client with diabetes and severe peripheral vascular disease has developed signs of dry gangrene on the great toe of one foot. The client asks, How this can happen? Which of the following pathophysiologic processes should the nurse explain to this client? More than likely, your gangrene is caused by: A. Inappropriate activation of apoptosis, which means death of your cells. B. Bacterial invasion into the foot and toe. C. Impaired arterial blood supply to your toe. D. Metaplastic cellular changes in your toe.

C. Impaired arterial blood supply to your toe.

A farmer's long-term exposure to pesticides has made the cells in his alveoli and bronchial tree susceptible to malignancy. Which of the following processes has taken place in the farmer's lungs? A. Promotion B. Progression C. Initiation D. Differentiation

C. Initiation

Despite the low levels of radiation used in contemporary radiologic imaging, a radiology technician is aware of the need to minimize her exposure to ionizing radiation. What is the primary rationale for the technician's precautions? Radiation: A. Stimulates pathologic cell hypertrophy and hyperplasia B. Results in the accumulation of endogenous waste products in the cytoplasm C. Interferes with DNA synthesis and mitosis D. Decreases the action potential of rapidly dividing cells

C. Interferes with DNA synthesis and mitosis

Despite the low levels of radiation used in contemporary radiologic imaging, a radiology technician is aware of the need to minimize her exposure to ionizing radiation. What is the primary rationale for the technician's precautions? Radiation: A. Stimulates pathologic cell hypertrophy and hyperplasia B. Results in the accumulation of endogenous waste products in the cytoplasm C. Interferes with DNA synthesis and mitosis D. Decreases the action potential of rapidly dividing cells

C. Interferes with DNA synthesis and mitosis

A client has sought care because of recent malaise and high fever. Upon assessment, the client states that his current fever began 2 days earlier, although he states that for the last 2 weeks he is in a cycle of high fever for a couple of days followed by a day or two of normal temperature. Which of the following fever patterns is this client experiencing? A. Recurrent fever B. Remittent fever C. Intermittent fever D. Sustained fever

C. Intermittent fever

When comparing and contrasting the various forms of RNA, the pathophysiology instructor identifies that ribosomal RNA (rRNA) is unique in that it: Select one: A. Delivers activated amino acids B. Coordinates RNA translation C. Is produced in the nucleolus D. Is formed by transcription

C. Is produced in the nucleolus

During ventricular systole, closure of the atrioventricular (AV) valves coincides with: A. Atrial chamber filling B. Aortic valve opening C. Isovolumetric contraction D. Semilunar valves opening

C. Isovolumetric contraction

What is the most common site of lipid accumulation? A. Coronary and other arteries B. Kidneys C. Liver D. Subcutaneous tissue

C. Liver

A mother rushes her 4-year-old child to the emergency department after she found an empty Tylenol (acetaminophen) bottle beside her child. The nurse is trying to explain why it is so important to give the child Ipecac to induce vomiting in order to prevent: A. Renal failure B. Seizures C. Liver failure D. Hemorrhage

C. Liver failure

In contrast to acute inflammation, chronic inflammation is characterized by which of the following phenomena? A. Profuse fibrinous exudation B. A shift to the left of granulocytes C. Lymphocytosis and activated macrophages D. Metabolic and respiratory alkalosis

C. Lymphocytosis and activated macrophages

During a lecture on inflammation, the physiology instructor discusses the major cellular components involved in the inflammation response. The instructor asks, Which of the following cells arrives early in great numbers? The student with the correct response is: A. Basophils B. Lymphocytes C. Neutrophils D. Monocytes

C. Neutrophils

A client had a positive Pap smear. The surgeon diagnosed "cancer in situ of the cervix." The client asks, "What does this mean?" From the following statements, which is most appropriate in response to this question? The tumor has: Select one: A. Developed a distant infiltration B. Been walled off within a strong fibrous capsule C. Not crossed the basement membrane, so it can be surgically removed with little chance of growing back D. Grown undifferentiated cells that no longer look like the tissue from which it arose

C. Not crossed the basement membrane, so it can be surgically removed with little chance of growing back

A client had a positive Pap smear. The surgeon diagnosed cancer in situ of the cervix. The client asks, What does this mean? From the following statements, which is most appropriate in response to this question? The tumor has: A. Been walled off within a strong fibrous capsule B. Developed a distant infiltration C. Not crossed the basement membrane, so it can be surgically removed with little chance of growing back D. Grown undifferentiated cells that no longer look like the tissue from which it arose

C. Not crossed the basement membrane, so it can be surgically removed with little chance of growing back

*Match the structure with its function. (Answers A through D may be used more than once.) A. Passive-mediated transport B. Active-mediated transport C. Osmosis D. Diffusion 5. Movement of water 6. Protein carrier 7. Facilitated diffusion

C. Omosis B. Active-mediated transport A. Passive-mediated transport

What is the mechanism by which the energy produced from carbohydrates, proteins and lipids is transferred to adenosine triphosphate (ATP)? A. Anaerobic glycolysis B. Oxidative cellular metabolism C. Oxidative phosphorylation D. Tricarboxylic acid phosphorylation

C. Oxidative phosphorylation

A liver failure client asks, How does the liver process ammonia in healthy individuals? The health care provider explains that ammonia is toxic to tissues, especially neurons, so this ammonia is removed from the blood by the liver, which: A. Converts it into bilirubin, which is then excreted intestinally B. Processes ammonia into nitrogen and hydrogen ions for excretion C. Processes it into urea, releasing it into the circulation D. Combines it with oxygen to create ammonium oxide

C. Processes it into urea, releasing it into the circulation

Which of the following is an application of recombinant DNA technology? Select one: A. Gene dosage studies B. DNA fingerprinting C. Production of human insulin D. Somatic cell hybridization

C. Production of human insulin

A child's thymus gland is fully formed and proportionately larger than an adult's. Which of the following processes that contributes to immunity takes place in the thymus gland? A. Differentiation of B cells B. Production of natural killer (NK) cells C. Proliferation of T cells D. Filtration of antigens from the blood

C. Proliferation of T cells

The highest incidence of cancer in US men is of what type? A. Lung B. Colon C. Prostate D. Testicular

C. Prostate

What organic compound facilitates transportation across cell membranes by acting as receptors, transport channels for electrolytes, and enzymes to drive active pumps? A. Lipids B. Proteolytic cascade C. Proteins D. Carbohydrates

C. Proteins

Which of the following aspects of the function of the nucleus is performed by the ribosomal RNA (rRNA)? A. Copying and carrying DNA instructions for protein synthesis B. Carrying amino acids to the site of protein synthesis occurs C. Providing the site where protein synthesis occurs D. Regulating and controlling protein synthesis

C. Providing the site where protein synthesis occurs

As a result of hypoxemia and polycythemia, persons with chronic obstructive bronchitis are prone to: A. Breakdown of elastin B. Left-sided heart failure C. Pulmonary hypertension D. Expiratory airway collapse

C. Pulmonary hypertension

A diabetic client has injured his foot while walking barefoot on the lawn. On admission, which of the following assessment findings would be considered a localized cardinal sign of acute inflammation? A. Temperature of 101°F B. Fatigue with listlessness C. Redness and edema at the injured site D. Urine output of less than 500 mL/24 hours (low)

C. Redness and edema at the injured site

Exocytosis, the reverse of endocytosis, is important into the extracellular fluid A. Engulfing and ingesting fluid and proteins for transport B. Killing, degrading, and dissolving harmful microorganisms C. Removing cellular debris and releasing synthesized substances D. Destruction of particles by lysomal enezymes for secretion

C. Removing cellular debris and releasing synthesized substances

Exocytosis allows granular content to be released into extracellular fluid by: A. Engulfing and ingesting fluid and proteins for transport B. Killing, degrading, and dissolving harmful microorganisms C. Removing cellular debris and releasing synthesized substances like hormones D. Destruction of particles by lysosomal enzymes for secretion

C. Removing cellular debris and releasing synthesized substances like hormones

Which of the following would be considered a clinical manifestation of acute pericarditis? A. Abdominal pain B. Muffled heart sounds C. Sharp, abrupt onset of chest pain that radiates to the neck D. Normal ECG

C. Sharp, abrupt onset of chest pain that radiates to the neck

A newborn has been lethargic, is not nursing well, and is basically looking ill. Following lab tests, it has been found that the newborn has IgM present in his blood. How should the nurse interpret this finding? A. All newborns have IgM in their blood so this is a normal finding. B. IgM in the blood means the infant may be allergic to breast milk. C. The presence of IgM suggests the infant has a current infection. D. IgM is usually only found in saliva so the specimen must be contaminated.

C. The presence of IgM suggests the infant has a current infection.

A lung biopsy and magnetic resonance imaging have confirmed the presence of a benign lung tumor in a client. Which of the following characteristics is associated with this client's neoplasm? Select one: A. The tumor is likely to infiltrate the lung tissue that presently surrounds it. Incorrect B. The cells that constitute the tumor are undifferentiated, with atypical structure. C. The tumor will grow by expansion and is likely encapsulated. D. If left untreated, the client's tumor is likely to metastasize.

C. The tumor will grow by expansion and is likely encapsulated.

When the heart's workload increases, what changes occur to the myocardial cells? A. They divide. B. They increase in size. C. They increase in number. D. They undergo metaplasia.

C. They increase in size

A breast cancer client has just learned that her tumor clinical stage is T3, N2, M0. After the physician leaves, the client asks the nurse to explain this to her again. The nurse will use which of the following statements in his or her answer? Your: A. Tumor is very small and has zero number of mitoses. B. Tumor has metastasized to at least three distal sites and you have cancer in your lymph nodes. C. Tumor is large and at least two lymph nodes are positive for cancer cells. D. Extent of disease is unknown, but it looks like your cancer has stayed intact and not spread to the bloodstream.

C. Tumor is large and at least two lymph nodes are positive for cancer cells.

A client with hepatitis B asks the nurse, How did I get this hepatitis? The nurse responds that the mode of transmission is predominantly by: A. Sharing oral secretions by kissing B. Fecal-oral route C. Unprotected sexual intercourse D. Inhalation of airborne droplets

C. Unprotected sexual intercourse

Which of the following membrane transport mechanisms requires the greatest amount of energy? A. Facilitated diffusion B. Passive transport C. Vesicular transport D. Simple diffusion

C. Vesicular transport

_______________ tumor (nephroblastoma) usually presents between 3 and 5 years of age and is the most common malignant abdominal tumor in children. A. Renal cell carcinoma B. Neuroblastoma C. Wilm's D. Adenocarcinoma

C. Wilm's

Cerebral dysfunction in end-stage liver disease can be attributed to the accumulation of: A. glucose B. bilrubin C. ammonia D. ketones

C. ammonia

An accumulation of nitrogenous waste products in the blood is called _____________. A. septicemia B. nitrosemia C. azotemia D. creatinemia

C. azotemia

The patient has right upper quadrant pain caused by acute "choledocholithiasis" (a gallstone in the common bile duct). If the common bile duct becomes obstructed, manifestations will also include: A. ascites. B. vomiting. C. bilirubinuria. D. hemorrhage.

C. bilirubinuria.

The majorityof bilirubin is derived from: A. dietary intake of carotenes B. iron metabolism C. breakdown of red blood cells D. pancreatic metabolism of bile

C. breakdown of red blood cells

Most kidney stones are composed of_________. A. urate. B. sodium. C. calcium. D. protein.

C. calcium.

A glycopeptide hormone produced by kidney fibroblasts and stimulates red blood cell production in the bone marrow is_______________________. A. aldosterone. B. renin. C. erythropoetin. D. angiotensin II.

C. erythropoetin.

Active transport occurs across membranes that: A. have a higher concentration of the solute on the outside of the cell. B. are semipermeable to water and small electrically uncharged molecules. C. have receptors that are capable of binding with the substances to be transported. D. have a cell membrane that is hydrophobic rather than hydrophilic.

C. have receptors that are capable of binding with the substances to be transported

Kupffer cells function as ______ to remove harmful substances or cells from the portal blood as it moves through the venous sinusoids. A. filters B. channels C. phagocytes D. cytotoxic cells

C. phagocytes

The late manifestations of cirrhosis are related to liver failure and: A. hepatomegaly. B. diffuse liver fibrosis. C. portal hypertension. D. hepatorenal syndrome.

C. portal hypertension.

Bacteria and viruses cause cell damage by _______, which is unique from the intracellular damage caused by other injurious agents. A. disrupting the sodium/potassium ATPase pump B. interrupting oxidative metabolism processes C. replicating and producing continued injury D. decreasing protein synthesis and function

C. replicating and producing continued injury

Cellular reproduction is a process that: A. often takes months or years to complete. B. typically has a short interphase. C. results in two diploid cells called daughter cells. D. involves the interaction of male and female cells.

C. results in two diploid cells called daughter cells

The functional unit of the kidney is the ____________. A. the renal tubule. B. the capillary. C. the nephron. D. the calyx.

C. the nephron.

What is the best screen for early kidney damage due to diabetes? A. serum protein B. serum creatinine C. urine microalbumin D. urine total protein

C. urine microalbumin

A mother accompanies her 12 month old child to the health clinic for his scheduled immunizations. What statement by the mother would cause the nurse to question immunization administration at this visit? a. "My child has had a runny nose for the past week." b. "Joey hates getting these shots, it makes him so mad- watch yourself." c. "Joey had a slight temperature this morning but it went away after I gave him some Tylenol." d. "Last time Joey got a shot, his little thigh was red and sore afterwards for two days!"

C: "Joey had a slight temperature this morning but it went away after I gave him some Tylenol." Rationale: A child should not receive immunizations generally if he/she has been running a fever or there are signs of moderate to severe illness even without a fever. Choice "A" would not be a contraindication for the immunizations if there are no other signs and symptoms of illness. Choice "B" would not be a contraindication for the immunization, most children do not like shots and object to them. Choice "D" is not a contraindication as this is considered a mild reaction to the injection.

As a nurse working in the newborn nursery and intensive care unit, when an infant has been diagnosed with failure to thrive, which of the following statements about hormones' effect on the body will the nurse explain to the family?

CRF increases somatostatin levels, which inhibits secretion of growth hormone.

With the exception of the first two steps, what is required in all steps of the clotting process?

Calcium

While explaining to the parents of a child diagnosed with postural scoliosis, the nurse will emphasize which of the following concepts? This form of scoliosis:

Can be corrected with bending and active/passive exercises

Nurse in the oncology unit has just admitted a client with metastatic cancer. The client asks how cancer moves from one place to another in the body. What would the nurse answer?

Cancer enter the body's lymph system and thereby spread to other parts of the body.

A client develops interstitial edema as a result of decreased:

Capillary Colloidal osmotic pressure

A 5-year-old girl's diagnosis of bone cancer required an aggressive treatment regimen. The client immediately receives doxorubicin chemotherapy. The nurse administering this medication should perform a detailed assessment of which body system that has been greatly affected by this drug?

Cardiac

Which of the following assessments should be prioritized in the care of a patient who is being treated for hypokalemia?

Cardiac monitoring

Which of the following assessments should be prioritized in the care of a client who is being treated for a serum potassium level of 2.7 mEq/L?

Cardiac monitoring looking for prolonged PR interval and flattening of the T wave

Which tissue types will not undergo hyperplasia?

Cardiac muscle cells

Following yearly routine physical examination by the health care provider, a client has been diagnosed with upper body obesity along with central fat distribution. The client is at greater risk for developing which of the following disease processes?

Cardiometabolic disorders

A client with type 1 diabetes has started a new exercise routine. Knowing there may be some increase risks associated with exercise, the health care provider should encourage the client to:

Carry a snack with carbs to prevent profound hypoglycemia

What are the three main parts of a typical cell? Select all that apply.

Cell Membrane Nucleus Cytoplasm

Which of the following processes associated with cellular injury is most likely to be reversible?

Cell damage resulting from accumulation of fat in the cytoplasm

During a discussion of different cell types in the body, the instructor mentions that which types of cells renew themselves continuously throughout life?

Cells lining the GI tract

A client with diabetes has impaired sensation, circulation, and oxygenation of his feet. He steps on a piece of glass, the wound does not heal, and the tissue area becomes necrotic. The necrotic cell death is characterized by:

Cellular breakage

A client with diabetes has impaired sensation, circulation, and oxygenation of his feet. He steps on glass and the tissue becomes necrotic. The cell death is characterized by:

Cellular breakage

What process associated with cellular injury is most likely to be reversible?

Cellular damage resulting from accumulation of fat in the cytoplasm

Which of the following cell structures is most likely responsible for the plant's wilted appearance?

Central vacuole

An elderly client has been brought to his primary care provider by his wife, who is concerned about his recent decrease in coordination. Upon assessment his primary care provider notes that the client's gait is wide-based, unsteady, and lacking in fluidity, although his muscle tone appears normal. The client require further assessment for which of the following health problems?

Cerebellar disorders

A client presents for a scheduled Papanicolaou (Pap) smear. The clinician who will interpret the smear will examine cell samples for evidence of

Changes in cell shape, size, and organization

A client presents for a scheduled Papanicolaou (Pap) smear. The clinician who will interpret the smear will examine cell samples for evidence of:

Changes in cell shape, size, and organization

A client has suffered a stroke that has affected his speech. The physician has identified the client as having expressive aphasia. Later in the day, the family asks the nurse to explain what this means. The most accurate response would be aphasia that is:

Characterized by an inability to easily communicate spontaneously or translate thoughts or ideas into meaningful speech or writing

Chemotaxis

Chemical substances that either attract or repel organisms

The process by which immune cells are attracted to sites of inflammation via the release of chemicals from bacteria and other immune cells is called:

Chemotaxis

Which of the following plant cell structures is most likely to contain chlorophyll a?

Chloroplast

Polycythemia develops in clients with lung disease as a result of:

Chronic hypoxia

Impaired and delayed healing in a person with diabetes is caused by long-term complications that include:

Chronic neuropathies

In men experiencing nonrelaxing external sphincter with associated urine retention, the health care worker should assess for which of the following possible causes?

Chronic prostatitis

A client underwent an open cholecystectomy 4 days ago, and her incision is now in the proliferative phase of healing. What is the dominant cellular process that characterizes this phase of the client's healing?

Collagen secretion by fibroblasts

A client develops interstitial edema as a result of decreased:

Colloidal osmotic pressure

Aerobic metabolism, also known as oxidative metabolism, provides energy to the body by:

Combining hydrogen & oxygen to form water

The client is found to have liver disease, resulting in the removal of a lobe of his liver. Adaptation to the reduced size of the liver leads to which phenomenon in the remaining liver cells?

Compensatory hyperplasia

Select the response that best describes a granulomatous inflammatory response to foreign body

Connective tissue encapsulates it and isolates it for removal

A client has been diagnosed with systemic lupus erythematosus, a disease that affects the structure and function of collagen. The nurse should expect to assess this client for:

Connective tissue weakness

A patient has a worsening cough. The patient is a long time smoker. No pneumonia is found. What is the best way to explain the patient's condition?

Constant irritation by smoking caused the ciliated epithelium of trachea to undergo metaplasia, turning them into squamous epithelium. The body can no longer clear normal airways secretion on its own.

A physiology instructor asks the students about the purpose of the promoter region on a DNA strand. Which student response is most accurate?

Contains amino acids that the RNA polymerase recognized and binds to, thus starting the replication process

A postsurgical client who is recovering in the postanesthetic recovery unit states that she is "freezing cold." Which of the following measures is likely to be initiated in the client's hypothalamus in an effort to reduce heat loss?

Contraction of pilomotor muscles

A large, high-calorie meal has resulted in the intake of far more energy than a person requires. What will the individual's body do with the excess carbohydrates provided by this meal?

Convert them into glucose and store them in the liver and muscles

According to Walter B. Cannon, homeostasis is a stable internal environment achieved through a system of:

Coordinated physiologic processes that oppose change

A nurse reading a sigmoidoscopy report notes that a patient was found to have skip lesions. The nurse interprets this as an indication of which of the following?

Crohn disease

A mutation has occurred during mitosis of an individual's bone marrow cell. This event may be the result of the failure of which of the following?

Cyclins

What is an accurate explanation of why a client with iron deficiency anemia experiences fatigue?

Cytochromes in skeletal muscle are less effective at transporting oxygen.

Which cells bind with MHC class I molecules?

Cytotoxic T cells

A client with diabetes has impaired sensation, circulation, and oxygenation of his feet. He steps on a piece of glass, the wound does not heal, and the tissue area becomes necrotic. The necrotic cell death is characterized by: A. Shrinkage and collapse B. Chronic inflammation C. Rapid apoptosis D. Cellular breakage

D

Which of the following dietary guidelines should a nurse provide to a group of older adults to possibly decrease their risks of developing colon cancer? A. "Regular vitamin supplements and a low-carbohydrate diet are beneficial." B. "Eat enough fiber in your diet that you have bowel movement at least once daily." C. "As much as possible, try to eat organic foods." D. "Try to minimize fat and maximize fiber when you're planning your meals."

D

A client has experienced a myocardial infarction with accompanying necrosis of cardiac muscle, a permanent tissue. What are the ramifications of the fact that cardiac muscle is a permanent tissue? A. Regeneration of the client's cardiac muscle will be exceptionally slow. B. The necrotic cells will be replaced with muscle cells that have limited metabolism. C. The cardiac muscle cells will remain perpetually in the G1 stage of mitosis. D. The cells will not proliferate and will be replaced with scar tissue.

D)

A client with diabetes and severe peripheral vascular disease has developed signs of dry gangrene on the great toe of one foot. The client asks, "How this can happen?" Which of the following pathophysiologic processes should the nurse explain to this client? "More than likely, your gangrene is caused by: A. Metaplastic cellular changes in your toe." B. Bacterial invasion into the foot and toe." C. Inappropriate activation of apoptosis, which means death of your cells." D. Impaired arterial blood supply to your toe."

D)

A client has been diagnosed with gram-negative pneumonia of the lower lobe. Knowing that gram-negative bacteria have a lipopolysaccharide endotoxin on their outer cell membrane, the health care provider should be assessing the client for which pathophysiological end result? A) Damage to cellular mitochondria B) Increased A TP levels C) Activation of the p53 protein D) Apoptosis

D) Apoptosis

A client has been diagnosed with gram-negative pneumonia of the lower lobe. Knowing that gram-negative bacteria have a lipopolysaccharide endotoxin on their outer cell membrane, the health care provider should be assessing the client for which pathophysiological end result? A) Damage to cellular mitochondria B) Increased ATP levels C) Activation of the p53 protein D) Apoptosis

D) Apoptosis Feedback: The extrinsic pathway of apoptosis involves extracellular signaling proteins that bind to cell surface molecules called death receptors and trigger apoptosis. The end result includes activation of endonucleases that cause fragmentation of DNA and cell death. In addition to TNF and Fas ligand, primary signaling molecules known to activate the extrinsic pathway include TNF-related apoptosis-inducing ligand (TRAIL); the cytokine interleukin-1 (IL-1); and lipopolysaccharide (LPS), the endotoxin found in the outer cell membrane of gram-negative bacteria. DNA damage, activation of the p53 protein, and decreased ATP levels are associated with the intrinsic pathway.

A group of elderly residents were commenting on how many cell functions decline with age. One resident commented that many of his friends who lived under large electromagnetic towers seemed to experience aging at an accelerated rate in comparison with residents who lived nearby in lakefront housing. This observation is the basis for which theory on aging? A) Theories of genetic influences B) Programmed cell receptor theories C) Insufficient telomerase enzyme theory D) Error theory associated with DNA damage

D) Error theory associated with DNA damage

A group of elderly residents were commenting on how many cell functions decline with age. One resident commented that many of his friends who lived under large electromagnetic towers seemed to experience aging at an accelerated rate in comparison with residents who lived nearby in lakefront housing. This observation is the basis for which theory on aging? A) Theories of genetic influences B) Programmed cell receptor theories C) Insufficient telomerase enzyme theory D) Error theory associated with DNA damage

D) Error theory associated with DNA damage Feedback: Error theory suggests that aging results from DNA mutation or faulty repair. Another group of theories of aging focuses on programmed cell changes with genetic influences that systematically cause cell senescence. Elimination of receptor sites is not part of aging theory. Telomerase enzyme is thought to reduce the shortening of the chromosomes and loss of telomere DNA with each cell replication.

13. Which of the following pathophysiologic processes is most likely to result in metastatic calcification? A) Benign prostatic hyperplasia B) Liver cirrhosis C) Impaired glycogen metabolism D) Hyperparathyroidism

D) Hyperparathyroidism

Which of the following pathophysiologic processes is most likely to result in metastatic calcification? A) Benign prostatic hyperplasia B) Liver cirrhosis C) Impaired glycogen metabolism D) Hyperparathyroidism

D) Hyperparathyroidism Feedback: Metastatic calcification is a result of markedly increased serum calcium levels. Because the parathyroid gland is responsible for the regulation of serum calcium levels, hyperparathyroidism creates a risk for hypercalcemia and consequent metastatic calcification. Benign prostatic hypertrophy, cirrhosis, and impaired glycogen metabolism are not implicated in cases of metastatic calcification.

18. The extrinsic pathway of apoptosis can be initiated by: A) damage to cellular DNA. B) decreased ATP levels. C) activation of the p53 protein. D) activation of death receptors on the cell surface.

D) activation of death receptors on the cell surface

A client asks the health care provider to explain phagocytosis. The provider will respond, Phagocytosis: A. Is a cotransport system that helps with the absorption of the medication. B. "Causes cells to explode." C. "Uses proteins to form an open channel through which the drug can move into the cell." D. "Is a process where microorganisms are engulfed and subsequently degraded or killed."

D. "Is a process where microorganisms are engulfed and subsequently degraded or killed."

The risk of chronic hepatitis in patients with acute Hepatitis C is: A. 1-3% B. 25% C. 50% D. 75%

D. 75%

Which of the following individuals likely faces the greatest risk for the development of chronic kidney disease? A. A first-time mother who recently lost 1.5 L of blood during a postpartum hemorrhage B. A client whose diagnosis of thyroid cancer necessitated a thyroidectomy C. A client who experienced a hemorrhagic stroke and now has sensory and motor deficits D. A client with a recent diagnosis of type 2 diabetes who does not monitor his blood sugars or control his diet

D. A client with a recent diagnosis of type 2 diabetes who does not monitor his blood sugars or control his diet

An early sign of acute symptomatic viral hepatitis may include: A. distaste for coffee B. severe anorexia C. inability to smoke cigarettes D. A, B, and C E. B and C

D. A, B, and C

Impairment in the function of peroxisomes would results in: A. Inadequate sites for protein synthesis B. An inability to transport cellular products across the cell membrane C. Insufficient energy production within a cell D. Accumulation of free radicals in the cytoplasm

D. Accumulation of free radicals in the cytoplasm

Following several weeks of increasing fatigue and a subsequent diagnostic workup, a client has been diagnosed with mitral valve regurgitation. Failure of this heart valve would have which of the following consequences? A. Backup of blood from the right atrium into the superior vena cava B. Backflow from the right ventricle to the right atrium during systole C. Inhibition of the SA node's normal action potential D. Backflow from the left ventricle to the left atrium

D. Backflow from the left ventricle to the left atrium

A child with cystic fibrosis (CF) asks the nurse why he has this disease, but his parents are perfectly healthy. The nurse explains: Select one: A. Environmental conditions affect on alleles and therefore who get CF B. Both parents have homozygous pairing of two alleles for CF. C. One of your parents may have a mild recessive form of CF. D. Both parents are carriers and have a recessive genotype with alleles for CF.

D. Both parents are carriers and have a recessive genotype with alleles for CF.

How do free radicals cause cell damage? A. By stealing the cell's oxygen to stabilize the electron, thus causing hypoxia B. By stimulating the release of lysosomal enzymes that digest the cell membranes C. By transferring one of its charged, stabilized atoms to the cell membrane causing lysis D. By giving up an electron causing injury to the chemical bonds of the cell membrane

D. By giving up an electron causing injury to the chemical bonds of the cell membrane

How is the transport of glucose from the blood to the cell accomplished? A. By active-mediated transport (active transport) B. By active diffusion C. By passive osmosis D. By passive-mediated transport (facilitated diffusion)

D. By passive-mediated transport (facilitated diffusion)

Which elevated serum marker for systemic inflammation is now considered a major risk factor for atherosclerosis and vascular disease? A. Leukocytosis B. Homocysteine C. Serum lipoprotein D. C-reactive protein

D. C-reactive protein

Which of the following processes associated with cellular injury is most likely to be reversible? Select one: A. Cellular changes as a result of ionizing radiation B. Cell damage resulting from accumulation of free radicals C. Apoptosis D. Cell damage resulting from accumulation of fat in the cytoplasm

D. Cell damage resulting from accumulation of fat in the cytoplasm

How do cells receive communication from the extracellular fluid surrounding them? A. Protein channel (gap junction) B. Plasma membrane-bound signaling molecules (involving receptors) C. Hormone secretion such as neurotransmitters D. Chemical messengers such as ligands

D. Chemical messengers such as ligands

A 51-year-old client has been diagnosed with stage IV breast cancer with lung metastases. The oncologist sits down with the client/family to explain treatment options. The nurse knows that which of the following treatment options will be discussed for her cancers? Select one: A. Hormone therapy B. Radiation therapy C. Surgery D. Chemotherapy

D. Chemotherapy

A 78 year old woman presented for her annual physical. Her last colonoscopy was 12 years ago, and her last mammogram was three years ago. Both were normal as all screenings have been in her well-women health care in the past 40 years. What screening tests do you recommend that are also recommended by the US Preventive Services Task Force??. A. Mammogram B. Mammogram and colonoscopy C. Mammogram. colonoscopy and Pap smear D. Colonoscopy

D. Colonoscopy

A respiratory therapist has asked a client to breathe in as deeply as possible during a pulmonary function test. Inspiration is normally the result of which of the following phenomena? A. Decreased intrapulmonary pressure B. Increased airway pressure C. Increased intrapleural pressure D. Decreased intrathoracic pressure

D. Decreased intrathoracic pressure

A group of teenagers spent an entire day on the beach without using sunscreen. The first night, their skin was reddened and painful to touch. The second day, they awoke to find large fluid-filled blisters over several body areas. The nurse recognizes the development of blisters as which type of inflammatory response A. Cellular response B. Immediate transient response C. Continuous response D. Delayed response

D. Delayed response

Free radicals damage cells by: Select one: A. Altering the immune response of the cell B. Inactivation of enzymes and mitochondria C. Disrupting calcium storage in the cell D. Destroying phospholipids in the cell membrane

D. Destroying phospholipids in the cell membrane

A client has experienced significant decreases in mobility and stamina during a 3-week hospital stay for the treatment of a femoral head fracture. Which of the following phenomena most likely accounts for the client's decrease in muscle function? Select one: A. Dysplasia as a consequence of inflammation during bone remodeling B. Impaired muscle cell metabolism resulting from metaplasia C. Ischemic atrophy resulting from vascular changes while on bedrest D. Disuse atrophy of muscle cells during a prolonged period of immobility

D. Disuse atrophy of muscle cells during a prolonged period of immobility

Which of the following statements about how smooth muscle differs from skeletal or cardiac muscle are accurate? Smooth muscle: A. Has multiple nuclei in each cell. B. Contains sarcomeres between Z lines and M bandsHas dense bodies attached to actin filaments C. Contracts more rapidly than skeletal muscles D. Has one centrally located nucleus

D. Has one centrally located nucleus

Which mode of chemical signaling uses blood to transport communication to cells some distance away? A. Paracrine signaling B. Autocrine signaling C. Neurotransmitter signaling D. Hormone signaling

D. Hormone signaling

Which of the following manifestations typically accompanies an asthmatic attack? A. Decreased residual volume B. Decreased pulmonary arterial pressure C. Prolonged inspiration D. Hyperinflation of the lungs

D. Hyperinflation of the lungs

Which of the following pathophysiologic processes is most likely to result in metastatic calcification? A. Benign prostatic hyperplasia B. Liver cirrhosis C. Impaired glycogen metabolism D. Hyperparathyroidism

D. Hyperparathyroidism

Antibody testing has confirmed that a man is positive for hepatitis A virus (HAV). Which of the client's statements suggests that he understands his new diagnosis? A. I guess I'm an example of why you should always use condoms. B. I'm embarrassed that I'll be a carrier of hepatitis from now on. C. I'm still trying to deal with the fact that this will forever change my life. D. I don't know why I didn't bother to get vaccinated against this.

D. I don't know why I didn't bother to get vaccinated against this.

Although bacterial toxins vary in their activity and effects on host cells, a small amount of gram-negative bacteria endotoxin: A. Is released during cell growth B. Inactivates key cellular functions C. Uses protein to activate enzymes D. In the cell wall activates inflammation

D. In the cell wall activates inflammation

Paraneoplastic syndromes are manifestations of cancer that often result from: A. Compression of area vessels B. Radiation and chemotherapy C. Tumor-related tissue necrosis D. Inappropriate hormone release

D. Inappropriate hormone release

A client with a history of emphysema is experiencing hypoxemia after a taxing physical therapy appointment. Which of the following physiologic phenomena will occur as a consequence of hypoxemia? A. Peripheral vasodilation B. Necrosis C. Hypoventilation D. Increased heart rate

D. Increased heart rate

Which cell component is capable of cellular autodigestion when it is released during cell injury? A. Ribosome B. Golgi complex C. Smooth endoplasmic reticulum D. Lysosomes

D. Lysosomes

Which of the following aspects of kidney function is performed by the juxtaglomerular apparatus? A. Regulating urine concentration B. Facilitating active transport to reabsorb electrolytes C. Regulating sodium and potassium elimination D. Matching changes in GFR with renal blood flow

D. Matching changes in GFR with renal blood flow

Following routine colonoscopy screening, a client is told that he had several polyps removed. The client began crying stating, I just can't deal with cancer. I'm too young. The nurse responds: A. "Don't worry. We have some great cancer doctors on staff. I'm sure chemo will help you fight it." B. "Maybe if you're lucky, they have stopped it from metastasizing to your liver." C. "Perhaps it will shrink on its own" D. Most colon polyps are not cancerous. The biopsy results will direct your care.

D. Most colon polyps are not cancerous. The biopsy results will direct your care.

Which of the following statements is true of genetic mutations? Select one: A. Mutations that occur in somatic cells are inheritable. B. Errors in DNA replication are most often fatal. C. Errors in DNA duplication are normally irreparable. D. Mutations may result from environmental agents.

D. Mutations may result from environmental agents.

During lecture on wound care, the instructor mentions the final stage of the cellular response of acute inflammation. Of the following statements, which describes what physiologically occurs in the final stage? A. Leukocytes accumulate and begin migration to the site of injury. B. Chemokines direct the trafficking of leukocytes. C. Mediators are transformed into inactive metabolites. D. Neutrophils, monocytes, and macrophages engulf and degrade the bacteria/cellular debris.

D. Neutrophils, monocytes, and macrophages engulf and degrade the bacteria/cellular debris.

A teenager presents to the clinic with an infected wound from a bike accident. He asks, How does my body fight off the germs in my scraped arm? Which is the nurse's best response? The cells that plays the central role related to the innate immune response to an infectious microorganism are: A. T lymphocytes. B. Antibodies. C. B lymphocytes. D. Neutrophils.

D. Neutrophils.

A client who was exposed to hepatitis A at a local restaurant has recovered from the disease. At her annual physical, the client asks the health care provider if she should go to her health department and get the hepatitis A shot. The best response, based on the concepts of adaptive immunity, by the health care provider would be: A. Yes, because you could get a worse case the next time you are exposed. B. Of course. The virus changes every year. C. I wouldn't since the vaccine can damage your liver. D. No, since having an active case, you have already developed antigens against hepatitis A.

D. No, since having an active case, you have already developed antigens against hepatitis A.

A 33-year-old client who is a long-term intravenous user of heroin has been recently diagnosed with hepatitis C. Which of the following portals of entry most likely led to the client's infection? A. Direct contact B. Vertical transmission C. Ingestion D. Penetration

D. Penetration

A client was in car accident client while not wearing a seatbelt and has sustained multiple rib fractures. During assessment, the nurse is having a hard time hearing heart sounds, and the client reports chest pain/pressure repeatedly. This client may be experiencing: A. Cardiomyopathy B. Pericarditis C. Pulmonary hypertension D. Pericardial effusion

D. Pericardial effusion

What prevents water-soluble molecules from entering cells across the plasma membrane? A. Carbohydrate chains B. Glycoprotein channels C. Membrane channel proteins D. Phospholipid bilayer

D. Phospholipid bilayer

To effectively relay signals, cell-to-cell communication utilizes a chemical messenger system to: A. Displace surface receptor proteins B. Accumulate within cell gap junctions C. Bind to contractile microfilaments D. Release secretions into extracellular fluid

D. Release secretions into extracellular fluid

To effectively relay signals, cell-to-cell communication utilizes chemical messenger systems that: A. Displace surface receptor proteins B. Accumulate within cell gap junctions C. Bind to contractile microfilaments D. Release secretions into extracellular fluid

D. Release secretions into extracellular fluid

A patient with pathophysiologic condition that affects the desmosomes is most likely to exhibit: A. Impaired contraction of skeletal and smooth muscle B. Weakness of the collagen and elastin fibers in the extracellular space C. Impaired communication between neurons and effector organs D. Separation at the junctions between epithelial cells

D. Separation at the junctions between epithelial cells

Which of following statements is true of skeletal muscle cells? A. Skeletal muscle cells each have an apical, lateral, and basal surface B. They are closely apposed and are joined by cell-to-cell adhesion molecules C. Their basal surface is attached to a basement membrane D. Skeletal muscle is multinucleated, lacking true cell boundaries

D. Skeletal muscle is multinucleated, lacking true cell boundaries

If the parasympathetic neurotransmitter releases acetylcholine, the nurse should anticipate observing what changes in the ECG pattern? A. Heart rate 150 beats/minute, labeled as supraventricular tachycardia B. Disorganized ventricular fibrillation C. Complete cardiac standstill D. Slowing of heart rate to below 60 beats/minute

D. Slowing of heart rate to below 60 beats/minute

Which of the following factors is likely to result in decreased renal blood flow? A. Action of dopamine B. Release of nitric oxide C. Action of prostaglandins D. Sympathetic nervous system stimulation

D. Sympathetic nervous system stimulation

Which of the following facts underlies the concept of replicative senescence? Select one: A. Genes controlling longevity are present or absent in varying quantities among different individuals. B. Aging produces mutations in DNA and deficits in DNA repair. C. The damaging influence of free radicals increases exponentially in later generations of a cell. D. Telomeres become progressively shorter in successive generations of a cell.

D. Telomeres become progressively shorter in successive generations of a cell.

While looking at cancer cells under a microscope, the instructor asks the students to describe the cells. Which of the student answers are accurate? Select the best answer. A. The cells are the same size and shape. B. The nucleoli are similar to normal cells. C. The cells are attached to an extracellular matrix. D. The cells do not resemble the tissue of origin.

D. The cells do not resemble the tissue of origin.

Which of the following factors accounts for the poor prognosis that typically accompanies a diagnosis of primary hepatocellular cancer? A. Surgical options do not exist because removal of all or part of the liver is a threat to health. B. Liver cancer typically metastasizes at a much earlier stage than other cancers. C. Liver tumors are poorly differentiated due to the low density of hepatic tissue. D. The nonspecific symptomatology of liver cancer means that diagnosis often happens at a late stage.

D. The nonspecific symptomatology of liver cancer means that diagnosis often happens at a late stage.

Autosomal dominant polycystic kidney disease (ADPKD) is a systemic disorder that primarily affects the kidneys, resulting in the formation of fluid-filled __________ in both kidneys with the threat of progression to chronic renal failure. A. vescicles B. capillaries C. tubules D. cysts

D. cysts

The important function of bilirubin deconjugation is accomplished by actions of the: A. gall bladder B. Kuppfer cells C. pancreas D. gut bacteria

D. gut bacteria

Which of the following statements is true of messenger RNA (mRNA)? Select one: A. mRNA delivers the activated form of an amino acid to the protein being synthesized. B. Each mRNA molecule has two recognition sites. C. mRNA is produced in the nucleolus. D. mRNA provides the template for protein synthesis.

D. mRNA provides the template for protein synthesis.

The abnormal proliferation of cells in response to excessive hormonal stimulation is called: A. dysplasia. B. pathologic dysplasia. C. hyperplasia. D. pathologic hyperplasia.

D. pathologic hyperplasia

Kernicterus is an avoidable complication of A. use of peripartum acetominophen (Tylenol) B. infant cretinism C. blood transfusions D. physiologic neonatal jaundice

D. physiologic neonatal jaundice

A common symptom of cholestasis/ intrahepatic bile obstruction is A. headache B. diarrhea C. tingling in fingers D. pruritus (itching)

D. pruritus (itching)

Neurotransmitters affect the postsynaptic membrane by binding to: A. lipids. B. ribosomes. C. amphipathic lipids. D. receptors.

D. receptors

Magnesium is important for the overall function of the body because of its direct role in:

DNA replication and transcription

Megaloblastic anemias caused by folic acid or vitamin B12 deficiencies can seriously affect RBC production. This is because both are necessary for ___ synthesis and ___.

DNA; red blood cell maturation

A very ill client has been admitted to the hospital for testing for possible septic shock. The client reports light-headedness, dizziness, and tingling/numbness of the fingers and toes. The nurse understands that this is likely due to which physiological phenomenon?

Decrease in cerebral blood flow

Which of the following body mechanisms is necessary for food intake control?

Decreased blood glucose

What is NOT a concern for a patient who experienced hypothermic injury?

Decreased blood viscosity and vasodilation

Conditions that predispose to sickling of hemoglobin in persons with sickle cell anemia include:

Decreased oxygen saturation

A client's condition has resulted in a decrease in work demands of most cells in the body; the anticipated result would be: a) Decreased size of organelles b) Decreased programmed cellular death c) Increased mitochondrial growth d) Increased oxygen consumption

Decreased size of organelles Explanation: Cell atrophy results in a decreased number and size of cell organelles. With cell atrophy, the cell has decreased oxygen consumption and growth of the mitochondria. The cell decreases in size. When confronted with a decrease in work demands or adverse environmental conditions, most cells are able to revert to a smaller size and a lower and more efficient level of functioning that is compatible with survival. This decrease in cell size is called atrophy. Cellular death process is not affected

During a client's admission assessment prior to reduction mammoplasty surgery, the nurse notes a reference to a Leiden mutation in the client's history. The nurse assesses this client for an increased risk of developing which of the following postsurgical complications?

Deep vein thrombosis

Processed: Creates immune cells specifically designed to attack that antigen Recognized: Includes a memory that makes future responses against a specific antigen more efficient.

Define the 2 types of adaptive immunity

A group of teenagers spent an entire day on the beach without using sunscreen. The first night, their skin was reddened and painful to touch. The second day, they awoke to find large fluid-filled blisters over several body areas. The nurse recognizes the development of blisters as which type of inflammatory response?

Delayed response

Lymphocytes

Derived from stem cells in bone marrow

Forms antigen - antibody complexes that circulate in blood and cause inflammation and organ complications.

Describe the pathogenesis of lupus

Capillary leak, nasal mucus secretions, conjunctival secretions, itching, redness, headache, sinus pressure.

Describe the s/s in an allergic reaction

Swelling of the conjunctiva, runny nose, swelling of lips, tongue and /or throat, heart and vasculature: fast or slow heart rate, low blood pressure, Skin: hives, itchiness, flushing, pelvic pain, central nervous system: Lightheadedness, loss of consciousness, confusion, headache, anxiety, Respiratory: Shortness of breath, wheezes, hoarseness, pain with swallowing, cough, Gastrointestinal: Crampy abdomen pain, diarrhea, vomiting, loss of bladder control.

Describe the s/s in anaphylaxis

Arthralgias or arthritis, vasculitis and rash, renal disease, hematologic changes, cardiovascular disease.

Describe the s/s of lupas

Protection against intracellular viruses, multicellular parasites, transplanted tissue and cancer cells viruses and cancerous cells. T - lymphocytes: don't make antibodies - lyse targets cells directly.

Describe the steps involved in cell mediated immunity

Antibodies made by plasma cells (activated B lymphocytes) in response to antigen. Antibodies primarily defend against bacteria, viruses, and toxins in blood and lymph - extracellular antigens. B cells produce plasma cells that produce antibodies. Activated B cells differentiate into memory cells that recognize and rapidly respond to the same antigen in future encounters.

Describe the steps involved in humoral immunity to protect against pathogens

Free radicals damage cells by:

Destroying phospholipids in the cell membrane

A school nurse has identified a student with noticeable loss of lean tissues and muscle mass. More than likely, this is caused by protein-calorie malnutrition. The nurse should ask the student if he is experiencing which of the following clinical manifestations that helps confirm this diagnosis?

Diarrhea

The process responsible for generating and conducting membrane potentials is:

Diffusion of current-carrying ion

The process responsible for generating and conducting membrane potentials is:

Diffusion of current-carrying ions

Viral hepatitis is characterized as what type of hypersensitivity.

Direct cell mediated type IV hypersensitivity

What is NOT a concern for a patient who experienced an electrical force energy?

Disruption of acid-base balance due to changing ionic charge

A healthy, primiparous (first time) mother delivered a healthy infant several hours ago, but the mother has experienced postpartum hemorrhage. Which of the following disorders is most likely to underlie the patient's excessive bleeding after delivery?

Disseminated intravascular coagulation

An 84-year-old client's blood cultures have come back positive, despite the fact that his oral temperature has remained within normal range. Which of the following phenomena underlies the alterations in fever response that occur in the elderly?

Disturbance in the functioning of the thermoregulatory center

A client has experienced significant decreases in mobility and stamina during a 3-week hospital stay for the treatment of a femoral head fracture. Which of the following phenomena most likely accounts for the client's decrease in muscle function?

Disuse atrophy of muscle cells during a prolonged period of immobility

An 89 yo female patient has experienced significant decreases in her mobility and stamina during a three week hospital stay for the treatment of a femoral head fracture. Which of the following phenomena most likely accounts for the patient's decrease in muscle function that underlies her reduced mobility?

Disuse atrophy of muscle cells during a prolonged period of immobility.

A client has experienced significant decrease in mobility and stamina during the 3 week hospital stay to treat a femoral head injury. What is the cause of the of his decrease in muscle function?

Disuse atrophy of muscle cells during prolonged period of immobility.

A client is beginning to recover from acute tubular necrosis. The nurse would likely be assessing which of the following manifestations of the recovery phase of ATN?

Diuresis

A patient's recent diagnosis of Parkinson disease has prompted his care provider to promptly begin pharmacologic therapy. The drugs that are selected will likely influence the patient's levels of:

Dopamine

Neutrophils

During a lecture on inflammation, the physiology instructor discusses the major cellular components involved in the inflammation response. The instructor asks, "Which cells arrive early in great numbers?" Which student response is correct?

Aortic stenosis is the abnormal deposition of calcium salts in a process called:

Dystrophic calcification

Liver failure is characterized by: A. pale feces B. dark urine C. hemorrhoids D. A and B E. A, B and C

E. A, B and C

Manifestations of acohol liver disease include A. fatty liver B. cirrhosis C. hepatitis D. A and B E. A, B and C

E. A, B and C

Which client who has undergone a major surgery to repair "leaking" colon with associated peritonitis, is most likely to experience enhanced wound healing?

Eats a high-calorie diet and large amounts of red meat

Which of the following clients, who have undergone a major surgery to repair a "leaking" colon with associated peritonitis, is most likely to experience enhanced wound healing as a result of his or her presurgery diet history? A client who:

Eats a high-calorie diet and large amounts of red meat

Nurse is assessing a client for acute inflammation of a wound. For which of these symptoms of infection does the nurse assess?

Edema

Which of the following measures should a school nurse prioritize in the treatment and prevention of childhood obesity?

Education on exercise and nutrition

When exercising outside on an extremely warm day, the client can feel his heart pounding very rapidly. Thinking in terms of the ability of the aorta to stretch to accommodate more blood circulating during exercise and the warm environment, this is an example of the function of which fibrous protein?

Elastin

Similarities between girls or women with anorexia nervosa and bulimia nervosa include:

Electrolyte imbalances

In the days following a tooth cleaning and root canal, a client has developed an infection of the thin, three-layered membrane that lines the heart and covers the valves. What is this client's most likely diagnosis?

Endocarditis

A nursing student's current clinical placement has been a source of stress due to high client acuity combined with interpersonal conflict with some of the unit staff. At the same time, the student has been fighting a cold for more than 2 weeks and has been unable to regain a normal feeling of health. How might these two phenomena be related?

Endocrine-immune interactions may suppress the student's immune response.

When discussing the various functions of the cellular parts, the pathophysiology professor asks a group of students, "Where are the primary sites of protein synthesis located in a cell?" Which student response is correct?

Endoplasmic reticulum

Which of the following structures is most likely damaged in the cell?

Endoplasmic reticulum

Sepsis

Endotoxins from bacteria cause increased capillary permeability with an escape of large volumes of plasma and cytokines cause further vasodilation.

Phagocytosis

Engulf bacteria and break down by lysosomes

Leukocytes consist of three categories of cells that have different roles in the inflammatory an immune responses. Which of the following leukocytes is correctly matched with its function?

Eosinophils; allergic reaction

Leukocytes consist of three categories of cells that have different roles in the inflammatory and immune responses. Which of the following leukocytes is correctly matched with its function?

Eosinophils—allergic reactions

A mother rushes her toddler into the emergency department stating, "My baby can't breathe." Initial assessment reveals the child is struggling to breathe in an upright position. He has both inspiratory and expiratory stridor and is using his chest muscles to breath. The nurse suspects the child has which of the following acute respiratory infections?

Epiglottitis

The 16 year old boy has enlarged lymph nodes and a sore throat. His girlfriend was recently diagnosed with infectious mononucleosis, which is caused by ___ and commonly transmitted in ____.

Epstein Barr virus; saliva

The 16-year-old boy has enlarged lymph nodes and a sore throat. His girlfriend was recently diagnosed with infectious mononucleosis. While educating this teenager, the nurse emphasizes that infectious mononucleosis is caused by which pathogen and usually transmitted via:

Epstein-Barr virus; saliva

While assessing a teenage girl suspected of having bulimia nervosa, the health care provider may find which of the following clinical manifestations that would confirm the diagnosis? Select all that apply

Eroded tooth enamel leading to sensitive teeth; Large number of teeth with dental caries; Painless parotid gland enlargement

A group of elderly residents were commenting on how many cell functions decline with age. One resident commented that many of his friends who lived under large electromagnetic towers seemed to experience aging at an accelerated rate in comparison with residents who lived nearby in lakefront housing. This observation is the basis for which theory on aging?

Error theory associated with DNA damage

A resident comments that many of his friends who live under large electromagnetic towers seem to experience aging at an accelerated rate. This observation is based of which theory on aging?

Error theory associated with DNA damage

Avoid close contact with family members Immunization Frequent hand washing

Everyone in the family except the mother currently has influenza. What should the nurse tell the mother are appropriate measures to avoid this infection? Select all that apply. (3)

Hypersensitivity immune response

Exaggerated or inappropriate response on exposure to an antigen

In comparison to children with acute otitis media (AOM), those with otitis media with effusion (OME) have:

Excess middle ear fluid

Which of the following clinical manifestations would support the medical diagnosis of Cushing syndrome? Select all that apply.

Excessive facial hair growth; Blood glucose level in 200 mg/dL range; "Buffalo hump" on back

A 6-hour-old newborn develops a critical respiratory problem and is rushed to the ICU. The ICU nurses suspect the infant has respiratory distress syndrome (RDS) based on which findings? Select all that apply.

Expiratory grunting; Substernal retractions with each breathe; Bluish discoloration of the skin and mucous membranes (central cyanosis)

A teratogenic environmental agent can cause birth defects when:

Exposed during early pregnancy

When performing an assessment on a school-aged child, the nurse notes that the mucous membranes along the gum margins have a noticeable blue-colored line. At this point, the nurse should ask the parents about possible:

Exposure to lead

Fetal alcohol syndrome (FAS) is unlike other teratogens in that the harmful effects on the fetus:

Extend throughout the pregnancy

Although energy is not made in mitochondria, they are known as the "power plants" of the cell because they:

Extract energy from organic compounds

The nurse is reviewing assessment documentation of a client's wound and notes "

Exudate containing white blood cells, protein, and tissue debris

A male client with a diagnosis of type 1 diabetes mellitus is experiencing hyperglycemia because he lacks sufficient insulin to increase the availability of glucose transporters in his cell membranes. Consequently, his cells lack intracellular glucose, and it accumulates in his blood. Which of the following processes would best allow glucose to cross his cell membranes?

Facilitated diffusion

A patient had his left lobe of his liver removed due to a hepatocellular carcinoma. His blood test will show persistent elevations of bilirubin for the rest of his life. True False

False

Acute renal failure is not a reversible process. True False

False

Hepatitis A has a high case fatality rate. True False

False

IgG is released first in the immune response to infection, as sign of acute infection. True False

False

Large molecules, such as protein, easily cross the glomerular wall. True False

False

Men and women often have similar clinical presentations when they are suffering an acute coronary syndrome (ACS) or event. True False

False

Most causes of renal obstruction are neoplastic in etiology. True False

False

Obstructive nephropathy is often inherited as a Mendelian trait. True False

False

One of the first renal manifestations of diabetic nephropathy is hypertension. True False

False

One of the strongest vasoconstrictors in the body is aldosterone. True False

False

The USPSTF ensures payment for all preventive services True False

False

The nurse is teaching her client with hepatobillary disease about her diet. She tells her that she may have steatorrhea, which the malabsorption which dietary component?

Fat

Nurse is assessing a client who is in the prodrome stage of a fever. The nurse anticipates the client will exhibit

Fatigue

Which of the following facts underlies the application of RNA interference in the treatment of disease?

Faulty gene activity that produces unwanted proteins can sometimes be stopped.

Client asks the nurse what is the most common objective symptom of systemic inflammation during the acute-phase response.

Fever

The basement membrane surrounding a client's foot wound remains intact, a fact that bodes well for the wound-healing process. Which types of substances/cells are responsible for the synthesis of collagen and elastic fibers?

Fibroblasts

The basement membrane surrounding a client's foot wound remains intact, good news for wound healing.What substances are responsible for the synthesis of collagen and elastic fibers?

Fibroblasts

When caring for a client during the proliferative phase of wound healing, the nurse teaches the client that which of these processes is taking place?

Fibroblasts secrete collagen for wound healing .

The "cold viruses" are rapidly spread from person to person. The greatest source of spread is:

Fingers

Which assessment findings of a client with an elevated temperature would be considered a "normal" finding? Select all that apply.

Flushed skin Pain when moving joints to sit up in bed Unusual fatigue and drowsiness

A client has been diagnosed with rheumatoid arthritis (RA). What will the nurse tell the client about this disorder's etiology?

Genetic predisposition is very like

Epithelial tissues are classified according to the shape of the cells and the number of layers. Which of the following is correctly matched description and type of epithelial tissue?

Glandular epithelium: arise from surface epithelia and underlying connective tissue.

Epithelial tissues are classified according to the shape of the cells and the number of layers. Which of the following is a correctly matched description and type of epithelial tissue?

Glandular epithelium: arises from surface epithelia & underlying connective tissue

Which of the following is a likely immediate consequence of this damage?

Glucose production in the cell will decrease.

Which statement regarding cell metabolism is most accurate?

Glycolysis also provides energy in situations when delivery of oxygen to the cells is delayed.

When explaining the function of glycolysis as it relates to anaerobic metabolism, the faculty will mention which of the following key points? Select all that apply.

Glycolysis occurs in mitochondrion-lacking cells; Pyruvic acid is an end result from a series of reactions that converts glucose.

In which of the following structures are the hydrolytic enzymes most likely to accumulate?

Golgi complex

A nursing student is cleaning and changing the dressing on a client's sacral ulcer. The student has vigorously cleansed the wound bed to remove all traces of the beefy, red tissue that existed in the wound bed. The student has most likely removed:

Granulation tissue

A student scrubs off all traces of the beefy, red tissue that exists in a bed wound. The student has most likely removed:

Granulation tissue

Epithelialization is delayed in open wounds until what tissue is formed?

Granulation tissue

Epithelialization, the first component of the proliferative phase of wound healing, is delayed in open wounds until what type of tissue has formed?

Granulation tissue

Which of the following meals would be considered high in folic acid intake?

Green leafy salad, beef and bean burrito on whole-wheat shell

Which of the following lab results strongly suggest an immunologic response in the client with possible rheumatic heart disease?

Group A (β-hemolytic) streptococcal antibodies

The angiogenesis process, which allows tumors to develop new blood vessels, is triggered and regulated by tumor-secreted:

Growth factors

Following a school shooting, many of the children want to talk about the traumatic event. The school nurse knows this is very therapeutic and would be called:

Guided imagery

In collecting assessment data on the school-aged population, which of the following factors could be the most significant predictor of childhood obesity?

Having parents who are obese

A high school senior sustained a concussion during a football game. Which of the following signs and symptoms would indicate the presence of postconcussion syndrome in the days and weeks following his injury?

Headaches and memory lapses

While taking a history from an adult client newly diagnosed with renal cell cancer, the nurse can associate which of the following high-risk factors with the development of this cancer?

Heavy smoking

Which cells bind with MHC class II molecules?

Helper T cells

The client is an average-sized adult and has abnormal microcytic hypochromic red blood cells due to a long-term, chronic disease. Which of the following complete blood count (CBC) results is characteristic of her type of anemia?

Hemoglobin 8 g/dL

The patient is an average sized adult and has abnormal microcytic hypochromic red blood cells due to a long-term, chronic disease. Which of the following CBC results is characteristic of her type of anemia?

Hemoglobin 8g/dL

When discussing linkage studies, the instructor mentions that colorblindness is found in a small section of the X chromosome and has been linked to development of which of the following diseases? Select all that apply.

Hemophilia A Adrenal hyperplasia

In the ICU, a postsurgical client has developed sepsis and is being treated with multiple medications. During the mid-morning assessment, which finding leads the nurse to suspect the client may be developing a complication called disseminated intravascular coagulation (DIC)? Select all that apply.

Hemorrhage from the surgical site requiring deep pressure dressings

Diabetic retinopathy, the leading cause of acquired blindness in the United States, is characterized by retinal:

Hemorrhages

A client underwent an open cholecystectomy 4 days ago, and her incision is now in the proliferative phase of healing. What is the dominant cellular process that characterizes this phase of the client's healing?

Hemostasis and vasoconstriction

A 40-year-old female has been diagnosed with hepatocellular carcinoma. When asked what causes this cancer, the physician looks at the history and responds, "I see in your history, you have had:

Hepatitis B related to IV drug use as a teenager

A 40-year-old female has been diagnosed with hepatocellular carcinoma. When asked what causes this cancer, the physician looks at the history and responds, "I see in your history, you have had:

Hepatitis B related to IV drug use as a teenager.

A 40-year-old female has been diagnosed with hepatocellular carcinoma. When asked what causes this cancer, the physician looks at the history and responds, "I see in your history, you have had:

Hepatitis B related to IV drug use as a teenager."

Which of the following characteristics distinguishes kwashiorkor from marasmus?

High intake of carbohydrates

Select the statement the best describes apoptosis. a) The release of products of cell death is uncontrolled b) Responsible for initiating an inflammatory response c) Unregulated by enzymatic digestion of cell components d) Highly selective in eliminating injured or aged cells

Highly selective in eliminating injured or aged cells Explanation: Apoptosis is a highly selective process that eliminates injured and aged cells in a manner that maintains the integrity of the plasma membrane and does not initiate inflammation. All the other options describe the occurrences of necrosis

A client with environmental allergies is experiencing respiratory inflammation. Which mediator causes vasodilation during the vascular stage of the inflammatory response?

Histamine

What chemical mediator triggers and starts the inflammatory process?

Histamine

65 year old patient has painless cervical node enlargement and the patient indicates it seems to only be above her diaphragm. Her biopsy shows Reed Sternberg cells. What type of lymphoma do you suspect?

Hodgkin disease

The bacteria are resistant to methicillin. Most antibiotics are useless for treating an infection.

How do bacteria become resistant to an antibiotic?

Bacteria produce a capsule that prevent phagocytosis Bacteria produce toxins. Proliferate at rates > development of protective response Hide from immune responses Change appearance of surface antigens

How do pathogens defend themselves?

1. Skin: Forms shield and secretes chemicals to kill invaders. 2. Mucous membranes and cilia:Sweep mucus into throat for coughing or swallowing. 3. Saliva: Chemicals to break down bacteria. 4. Stomach acid: same. 5. Bladder: flushes out bacteria in urine. 6. Normal bacteria flora.

How do physical barriers protect the body against infection?

HIV

Human immunodeficiency virus. Attacks the immune system, causing deficiency or damage in the immune system. Damages the body's ability to fight disease and infections. Leads to AIDS.

A 77-year-old woman has been brought to the emergency department by her daughter because of a sudden and unprecedented onset of confusion. The patient admits to ingesting large amounts of baking soda since the morning in an effort to treat indigestion. How will the the woman's body attempt to resolve this disruption in acid-base balance?

Hyoventilation

While assessing a client who is experiencing diarrhea caused by Clostridium dificle, the nurse should anticipate hearing

Hyperactive bowel sounds

A female client experienced a random assault several months earlier, and her recent vigilance around her own safety is described as "obsessive" by her friends and family. Which of the following aspects of posttraumatic stress disorder (PTSD) characterizes the client's behavior?

Hyperarousal

Which of the following pathophysiologic processes is most likely to result in metastatic calcification?

Hyperparathyroidism

Which of the following pathophysiologic processes is most likely to result in metastatic calcification? a) Hyperparathyroidism b) Benign prostatic hyperplasia c) Impaired glycogen metabolism d) Liver cirrhosis

Hyperparathyroidism Explanation: Metastatic calcification is a result of markedly increased serum calcium levels. Because the parathyroid gland is responsible for the regulation of serum calcium levels, hyperparathyroidism creates a risk for hypercalcemia and consequent metastatic calcification. Benign prostatic hypertrophy, cirrhosis, and impaired glycogen metabolism are not implicated in cases of metastatic calcification

The nurse is providing care for a client who has a diagnosis of kidney failure. Which of the following laboratory findings is consistent with this client's diagnosis?

Hypocalcemia

A clinician is assessing the muscle tone of a patient who has been diagnosed with a lower motor neuron lesion. Which of the following assessment findings is consistent with the client's new diagnosis?

Hypotonia

A 77-year-old woman has been brought to the emergency department by her daughter because of a sudden and unprecedented onset of confusion. The client admits to ingesting large amounts of baking soda this morning to treat some "indigestion." How will the woman's body attempt to resolve this disruption in acid-base balance?

Hypoventilation

A 70 yr old male client has been admitted to the hospital for treatment of a recent hemorrhagic stroke that left him with numerous motor and sensory deficits. What is most likely the mechanisms of cell injury that resulted in these deficits?

Hypoxia and ATP depletion

A 70-year-old male client has been admitted to a hospital for the treatment of a recent hemorrhagic stroke that has left him with numerous motor and sensory deficits. These deficits are most likely the result of which of the following mechanisms of cell injury?

Hypoxia and ATP depletion

a 70 yo male patient has been admitted to a hospital for the treatment of a recent hemorrhagic stroke that has left him with numerous motor and sensory deficits. These deficits are most likely the result of which of the following mechanisms of cell injury?

Hypoxia and ATP depletion

A 66 year old woman has been diagnosed with diverticular disease based on her recent complaints and the results of a computed tomography (CT) scan. Which of the patients following statements demonstrates an accurate understanding of this diagnosis.

I suppose I should try to put more fiber in my diet

No. May show up negative just after being infected till sufficient antibodies form.

If the above tests are negative, does that mean the person does not have HIV?

A patient has acidosis thta is suspected to be respiratory in etiology. Which of the following is the major cause of acute primary respiratory acidosis?

Impaired alveloar ventilation

A client has dry gangrene. More than likely, the gangrene is caused by:

Impaired arterial blood supply to the area

A client with diabetes and severe peripheral vascular disease has developed signs of dry gangrene on the great toe of one foot. The client asks, "How this can happen?" Which of the following pathophysiologic processes should the nurse explain to this client? "More than likely, your gangrene is caused by:

Impaired arterial blood supply to your toe

A client with diabetes and severe peripheral vascular disease has developed signs of dry gangrene on the great toe of one foot. The client asks, "How this can happen?" Which of the following pathophysiologic processes should the nurse explain to this client? "More than likely, your gangrene is caused by:

Impaired arterial blood supply to your toe."

A patient has been diagnosed with cholecystitis (gallbladder inflammation) that has impaired the normal release of bile. Which of the following gastrointestinal consequences is this patient likely to experience?

Impaired digestion of fats

The pathologic effects of the thalassemias are primarily due to which of the following pathophysiologic processes?

Impaired hemoglobin synthesis

Paraneoplastic syndromes are manifestations of cancer that often result from:

Inappropriate hormone release

Which of the following signs/symptoms would lead the nurse to suspect the post-op client has developed a mechanical bowel obstruction? Select all that apply

Increase in abdominal distention; Nausea and vomiting; Continuous abdominal pain; Sluggish to absent bowel sounds

Which of the following manifestations would be considered a result of stress-induced cortisol hormone secretion?

Increased blood glucose level to 180 with excess hunger and thirst

A college student has just learned that her latest Pap smear revealed atypical cells, a fact that has resulted in stress and an accompanying release of angiotensin II. How would the effects of this hormone be objectively demonstrated?

Increased blood pressure

A client has been receiving intravenous normal saline at a rate of 125 mL/hour since her surgery 2 days earlier. As a result, she has developed an increase in vascular volume and edema. Which of the following phenomena accounts for this client's edema?

Increased capillary filtration pressure

A patient has been receiving intravenous normal saline at a rate of 125 mL per hour since her surgery 2 days earlier. As a result of her consequent increase in vascular volume, she has become edematous. Which of the following phenomena accounts for this patient's eden?

Increased capillary filtration pressure

Which manifestation of stress reflects the nonspecific "fight-or-flight" response?

Increased cardiopulmonary rates

Which manifestation of stress reflects the nonspecific "flight-or-fight" response?

Increased cardiopulmonary rates

Hemolytic anemia is characterized by excessive red blood cell destruction and compensatory:

Increased erythropoiesis

A patient is experiencing significant stress while awaiting the results of her recent lymph node biopsy. Among the hormonal contributors to this response is a release of aldosterone, resulting in which of the following physiologic effects?

Increased sodium absorbtion

A client is experiencing significant stress while awaiting the results of her recent lymph node biopsy. Among the hormonal contributors to this response is a release of aldosterone, resulting in which of the following physiologic effects?

Increased sodium absorption

A 68 year old patient with an 80 pack/year history of smoking was diagnosed with emphysema 18 months ago. The patient's most recent scheduled blood work showed red blood cell indices, a problem that suggests the need for which of the following interventions?

Increased supplementary oxygen therapy

Hyperplasia

Increased workload causes normal cells to proliferate. (increase number of cells-normal cells)

Hypertrophy

Increased workload triggers increased synthesis of normal cell components. (Increase cell size due to increase cellular component)

During accommodation, pupillary dilation partially compensates for the reduced size of the retinal image by:

Increasing light entering the pupil

A febrile, 3-week-old infant has been brought to the emergency department by his parents and is currently undergoing a diagnostic workup to determine the cause of his fever. Which of the following statements best conveys the rationale for this careful examination?

Infants are susceptible to serious infections because of their decreased immune function

A febrile, 3-week-old infant has been brought to the emergency department by his parents and is currently undergoing a diagnostic workup to determine the cause of his fever. Which of the following statements best conveys the rationale for this careful examination?

Infants are susceptible to serious infections because of their decreased immune function.

The client has a fractured tibia. After the cast is applied, he is at high risk for compartment syndrome caused by:

Inflammation

When a client with a kidney transplant develops graft versus host disease, a suicide gene transfer can be accomplished by:

Infusion of donor lymphocytes

A client is brought to the emergency department semicomatose and a blood glucose reading of 673. He is diagnosed with diabetic ketoacidosis (DKA). Blood gas results are as follows: serum pH 7.29 and HCO3- level 19 mEq/dL; PCO2 level 32 mm Hg. The nurse should anticipate that which of the following orders may correct this diabetic ketosis?

Initiating an insulin IV infusion along with fluid replacement

Allostasis is characterized by:

Interactive physiologic changes in numerous systems

Despite the low levels of radiation used in contemporary radiologic imaging, a radiology technician is aware of the need to minimize her exposure to ionizing radiation. What is the primary rationale for the technician's precautions? Radiation:

Interferes with DNA synthesis and mitosis

Despite the low levels of radiation used in contemporary radiologic imaging, a radiology technician is aware of the need to minimize her exposure to ionizing radiation. What is the primary rationale for the technician's precautions? Radiation: a) Stimulates pathologic cell hypertrophy and hyperplasia b) Interferes with DNA synthesis and mitosis c) Results in the accumulation of endogenous waste products in the cytoplasm d) Decreases the action potential of rapidly dividing cells

Interferes with DNA synthesis and mitosis Explanation: Radiation has a damaging effect on DNA synthesis and mitosis, a process that is especially harmful to rapidly dividing cells. Radiation does not directly influence the action potential of cells or the accumulation of endogenous waste products. Cell changes such as hypertrophy or hyperplasia may result from radiation exposure, but such changes are secondary to interference with DNA synthesis and mitosis

A client has sought care because of recent malaise and high fever. Upon assessment, the client states that his current fever began 2 days earlier, although he states that for the last 2 weeks he is in a cycle of high fever for a couple of days followed by a day or two of normal temperature. Which of the following fever patterns is this client experiencing?

Intermittent fever

A patient has sought care of recent malaise and high fever. Upon assessment, the patient states that his current fever began two days earlier, although he state that for the last 2 weeks he is in a cycle of high fever for a couple of days followed by a day or two of normal temp. Which fever pattern is this patient experiencing?

Intermittent fever

A female client with a history of chronic renal failure has a total serum calcium level of 7.9 mg/dL. While performing an assessment, the nurse should focus on which of the following clinical manifestations associated with this calcium level?

Intermittent muscle spasms and complaints of numbness around her mouth

A client with a history of several previous abdominal surgeries has been admitted to the hospital with several abdominal pains. Knowing that fibrous strands of collagen can form following abdominal surgery resulting in adhesions, the nurse should assess the client for:

Intestinal obstruction causing constipation

Diabetic and hypertensive retinopathy are both characterized by the appearance of:

Intraretinal hemorrhages

A child has been hospitalized for the treatment of hematogenous osteomyelitis. The defining characteristic of this type of osteomyelitis is:

Introduction of microorganisms from the bloodstream

Client asks the nurse how malignant tumors differ from benign tumors. The best response would be that malignant tumors..

Invade surrounding tissues

Smooth muscle contractions are typically characterized as:

Involuntary

A 72-year-old woman with complaints of increasing fatigue has completed a series of fecal occult blood tests that indicate the presence of blood in her stool. Which of the following health problems is likely to accompany this client's gastrointestinal bleed?

Iron deficiency anemia

A 72 year old woman with complaints of increasing fatigue has completed a series of fecal occult blood tests that indicate the presence of blood in her stool. Which of the following health problems is likely to accompany this patinet's gastrointestinal bleed?

Iron-deficiency anemia

Which client has a high risk of chronic inflammation. One who:

Is morbidly obese and who has a sedentary lifestyle

Which of the following individuals most likely has the highest risk of experiencing chronic inflammation? A client who:

Is morbidly obese and who has a sedentary lifestyle

Which of the following pregnant women likely faces the greatest risk of developing gestational diabetes? A client who:

Is morbidly obese defined as greater than 100 pounds over ideal weight

A client's emergency magnetic resonance imaging (MRI) has been examined by the physician and a clot busting medication called tissue plasminogen activator (tPA) has been administered to the client. What was this client's most likely diagnosis?

Ischemic stroke

Which describes the function of the nucleus?

It contains the genetic code for the individual

The nurse is caring for a client with an accumulation of 2.5cm of darkened tissue scar over the area of a 3-mm injury. How does the nurse correctly document this finding the medical record?

Keloid

While the nurse is performing a skin assessment on a dark-skin client, the nurse notes that the client has a healed wound on the leg but that the wound has an excess of scar tissue. The nurse documents this as

Keloid

Which of the following practitioners is most likely to be of assistance in the early care of an infant with a cleft lip?

Lactation consultant

Nurse evaluating the bloodwork results of a client with an infected leg ulcer. The white blood cell count is 18,000 cells/uL. The nures inteprets this as:

Leukocytosis

What chemical mediator attracts neutrophils to the sire of injured site?

Leukotrienes

The living part of the cell found both inside and outside of the nucleus is called protoplasm. In addition to proteins, carbohydrates, electrolytes, and water, what other substance is found in protoplasm?

Lipids

Which endogenous pigment gives cells a yellow color due to accumulation of an indigestible residue leftover from normal turnover of cell structures?

Lipofuscin

Weakened immune system; immunodeficiency or immunocompromised. Poor hygiene. Malnutrition. Inadequate physical barriers. Emotional stressors. Physical stressors. Chronic diseases. Medical and surgical treatments. Inadequate immunizations. Newborns and old age.

List risk factors for infection

A mother rushes her 4-year-old child to the emergency department after she found an empty Tylenol (acetaminophen) bottle beside her child. The nurse is trying to explain why it is so important to give the child Ipecac to induce vomiting in order to prevent:

Liver failure

A mother rushes her 4-year-old child to the emergency department after she found an empty Tylenol (acetaminophen) bottle beside her child. The nurse is trying to explain why it is so important to give the child Ipecac to induce vomiting in order to prevent: a) Liver failure b) Renal failure c) Hemorrhage d) Seizures

Liver failure Explanation: Acetaminophen, a commonly used over-the-counter analgesic drug, is detoxified in the liver, where small amounts of the drug are converted to a highly toxic metabolite. This metabolite is detoxified by a metabolic pathway that uses a substance normally present in the liver. When large amounts of the drug are ingested, this pathway becomes overwhelmed and toxic metabolites accumulate, causing massive liver necrosis

When an infant is born with gene mutations in his cells, the nurse explains to the parents that accidental errors may be a result of: Select all that apply.

Loss of one or more base pairs Substitution of one base pair for another D. Rearrangement of the base pairs

Sue is fatigued, and some blood tests are done. Her results include Hct 40%; Hgb 8g/dL; WBC 8000; and platelets 175,000. The nurse should interpret Sue's blood work as indicative of:

Low hemoglobin/anemia

A frail, 87-year-old female client has been admitted to a hospital after a fall and has been diagnosed with failure to thrive. Which of the following laboratory values would suggest that the client may be experiencing malnutrition?

Low prealbumin

A frail, 87-year-old female patient has been admitted to a hospital after a fall and has been diagnosed with failure to thrive. Which of the following laboratory values would suggest that the patient may be experiencing malnutrition?

Low prealbumin

The syndrome of inappropriate antidiuretic hormone (SIADH) is characterized by:

Low serum sodium level of 122 mEq/L

The nurse caring for a client with diverticulitis should assess for which of the following clinical manifestations?

Lower left quadrant pain

In addition to facilitating bowel movements, a diet that is high in fiber confers which of the following benefits?

Lowering cholesterol and blood glucose

A client has been diagnosed with non-Hodgkin lymphoma (NHL), a form of malignancy that most likely originated in which of the following sites?

Lymph nodes

In contrast to acute inflammation, chronic inflammation is characterized by what phenomena?

Lymphocytosis and activated macrophages

In contrast to acute inflammation, chronic inflammation is characterized by which of the following phenomena?

Lymphocytosis and activated macrophages

Breakdown and removal of foreign substances and worn out cell parts are performed by which of the following organelles?

Lysosomes

Select the organelle that is responsible for the breakdown of excess and worn-out cell parts as well as foreign substances that are taken into the cell.

Lysosomes

Which of the following cell structures is most likely associated with the accumulation of gangliosides?

Lysosomes

A patient is about to go into surgery. Why is it important to get family and personal history before the surgery?

Malignant hyperthermia could be dangerous from the anesthesia, the patient's muscles lose stored calcium, causing increase temperature and rigidity.

Crohn disease not only affects adults but also can occur in children. The nurse assesses for which major manifestation in children with Crohn disease?

Malnutrition

Antigen

Markers on the outside of bacteria and viruses, which allow antibodies to recognize foreign invaders.

The mediators involved in type I hypersensitivity allergic responses are released from which of the following?

Mast cells

A client has an abscess in the mouth with a profuse amount of thick creamy white exudate. The nurse knows that this wound with necrotic cells is classified as:

Membranous

Genetic disorders that involve a single gene trait are characterized by:

Mendelian patterns of transmission

Arterial blood gases of a client with a diagnosis of acute renal failure reveal a pH of 7.25, HCO3- level of 21 mEq/L, and decreased PCO2 level accompanied by a respiratory rate of 32. This client is most likely experiencing which disorder of acid-base balance?

Metabolic acidosis

The most recent assessment of a client with a diagnosis of type 1 diabetes indicates a heightened risk of diabetic nephropathy. Which of the following assessment findings is most suggestive of this increased risk?

Microalbuminuria

Semen analysis indicates that the client's sperm have decreased motility. Which of the following cellular components may be defective within the client's sperm?

Microtubules

Which statement accurately explains function of cellular components?

Microtubules aid intracellular transport

Protein contains nitrogen. A negative nitrogen balance represents:

More nitrogen excreted than consumed

A 2-week-old infant (full-term at birth) is admitted to the pediatrics unit with "spitting up large amounts of formula" and diarrhea. The infant has developed a weak suck reflex. Which of the following statements about total body water (TBW) is accurate in this situation?

Most full-term infants have a TBW of approximately 75% due to their high metabolic rate.

Which of the following statements is true of genetic mutations?

Mutations may result from environmental agents.

Which of the following statements is true of genetic mutations?

Mutations may result from extrinsic factors or from spontaneous error

Which hypersensitivity disorder is associated with IgG or IgM antibodies binding to normal body cells and causing cellular injury?

Myasthenia graves

One of the principal mechanisms by which the heart compensates for increased workload is:

Myocardial hypertrophy

Although stress exposure initiates integrated responses by multiple systems, the functional changes are first manifested by which system?

Neuroendocrine

Which of the following cells will likely stay in a permanent resting state known as G0?

Neurons

During a lecture on inflammation, the physiology instructor discusses the major cellular components involved in the inflammation response. The instructor asks, "Which of the following cells arrives early in great numbers?" The student with the correct response is:

Neutrophils

During a lecture on inflammation, the professor describes the major cellular components of the inflammation response. Which cells arrive early in great numbers?

Neutrophils

A school nurse is teaching a class on immunity. Which statement contains an accurate explanation about cellular defenses?

Neutrophils engulf invading organisms where lysosomes break them down.

During lecture on wound care, the instructor mentions the final stage of the cellular response of acute inflammation. Of the following statements, which describes what physiologically occurs in the final stage?

Neutrophils, monocytes, and macrophages engulf and degrade the bacteria/cellular debris

During lecture on wound care, the instructor mentions the final stage of the cellular response of acute inflammation. Of the following statements, which describes what physiologically occurs in the final stage?

Neutrophils, monocytes, and macrophages engulf and degrade the bacteria/cellular debris.

During lecture on wound care, the instructor mentions the final stage of the cellular response of acute inflammation. What physiologically occurs in the final stage?

Neutrophils, monocytes, and macrophages engulf and degrade the bacteria/cellular debris.

A client is experiencing stress as a nurse prepares to insert a peripheral intravenous catheter into his forearm. The client's locus ceruleus (LC) is consequently producing which of the following hormones?

Norepinephrine (NE)

A client asks why his temperature is always below 98.6°F. The nurse responds:

Normal core temperature varies between individuals within the range of 97.0°F to 99.5°F.

A client asks why his temperature is always below 98.6°F. What is the normal range for body temperature?

Normal core temperature varies between individuals within the range of 97.0°F to 99.5°F.

Autoimmune reaction

Normal defense mechanisms become self - destructive recognizing self - antigens as foreign.

A client had a positive Pap smear. The surgeon diagnosed "cancer in situ of the cervix." The client asks, "What does this mean?" From the following statements, which is most appropriate in response to this question? The tumor has:

Not crossed the basement membrane, so it can be surgically removed with little chance of growing back

Nociceptors are sensory receptors that are activated by:

Noxious stimuli

cell death by necrosis is:

Often a response to inflammation

A client with trigeminal neuralgia typically complains of excruciating pain. Which of the following actitivies may trigger an acute pain attack? Select all that apply.

One's significant other lightly stroking their face; Working in the office that has an air duct located directly overhead; Walking outside on a windy day

The most common cause of thyrotoxicosis is Graves disease. When assessing this client, the nurse should put priority on which of the following signs/symptoms?

Ophthalmopathy

Opsonization

Opsonin binds to bacteria and renders it more attractive for phagocytosis

While reviewing the basic information related to hemodialysis, the professor explains that water molecules move through adjacent phospholipid molecules in the cell membrane by:

Osmosis

An older adult client has just sheared the skin on her elbow while attempting to boost herself up in bed, an event that has precipitated acute inflammation in the region surrounding the wound. What will occur during the vascular stage of the client's inflammation?

Outpouring of exudate into interstitial spaces

An older adult client has just sheared the skin on her elbow while attempting to boost herself up in bed, an event that has precipitated acute inflammation in the region surrounding the wound. Which of the following events will occur during the vascular stage of the client's inflammation?

Outpouring of exudate into interstitial spaces

COPD clients live with persistently elevated PCO2 levels. Therefore, which assessment finding would likely initiate a stimulus for ventilation in this client population?

PO2 level of 50 mm Hg

When lecturing about heart attacks (myocardial infarctions), the instructor will emphasize the client may present with: Select all that apply

Pain that radiates to the left arm • Substernal chest pain • Neck pain

A client is being seen in the dermatology clinic for a follow-up appointment for a second-degree full- thickness burn. What are characteristics of second-degree burns? Select all that apply

Painful Mottled, pink red Blistered

A 16 year old girl has been brought to her primary health care provider by her mother due to the daughter's recent malaise and lethargy. Which of the following assessments should the clinician perform in an effort to confirm or rule out infectious mononucleosis?

Palpating the patient's lymph nodes.

A teenaged cystic fibrosis client presents to the clinic. The health care provider (HCP) knows that cystic fibrosis (CF) causes severe chronic respiratory disease in children. In addition, the HCP should also focus his or her assessment on which of the other body systems affected by CF?

Pancreatic

A newly diagnosed paraplegic client who suffered an automobile accident appears to have control of bladder emptying. The health care provider explains this process to the client/family stating, "This function is allowing the motor component of the neural reflex to assist with bladder emptying and is primarily controlled by the:

Parasympathetic division of the ANS

Which of the following best describes how vacuoles differ between plant cells and animal cells?

Plant cells typically have a single vacuole while animal cells have multiple vacuoles.

Multiple myeloma is a malignancy of what type of cell?

Plasma cells

Which of the following cell types is responsible for producing antibodies against specific pathogens?

Plasma cells

A client is undergoing a diagnostic workup to rule out multiple myeloma. Which of the following diagnostic findings would confirm the diagnosis of multiple myeloma?

Plasma cells greater than 10% on bone marrow biopsy

Which of the following patients is most likely to have impairments to the wound healing process? A patient with:

Poorly controlled blood sugars with small blood vessel disease.

More complex patterns of movements, such as throwing a ball or picking up a fork, are controlled by the ______ cortex in the frontal lobe.

Premotor cortex

A 41-year-old woman has made the recent decision to start a family and is eager to undergo testing to mitigate the possibility of having a child with Down syndrome. Which of the following tests is most likely to provide the data the woman seeks?

Prenatal blood tests

To which of the following does the term apoptosis refer?

Preprogrammed cell self-destruction

Which of the following vision deficits is a clinician justified in attributing to the normal aging process?

Presbyopia

Metaplasia

Preserves tissue function in harsh environment by changing type of normal cells. (replacement of 1 mature cell type that belongs there with another mature cell type that belong in different location)

A 69-year-old client who is obese and has a diagnosis of angina pectoris has been prescribed clopidogrel (Plavix) by his primary care provider. The client asks, "Why do I need this medication? It won't help my chest pain." The best response would be this medication:

Prevents the blood cells from forming a clot in your heart vessels

What organisms are responsible for causing the neurodegenerative mad cow disease?

Prions

Which one of the following pathogens contains no DNA or RNA?

Prions

Exogenous pyrogens (interleukin-1) and the presence of bacteria in the blood lead to the release of endogenous pyrogens that:

Produce leukocytosis and shivering

Which of the following statements best conveys the endocrine function of adipose tissue? Adipose tissue:

Produces leptin, which mediates body weight

When educating students about the differences between brown fat versus white fat, the instructor will share that brown fat has iron in its mitochondria that will facilitate:

Production of a protein that releases the energy generated from metabolism as heat

Which of the following is an application of recombinant DNA technology?

Production of human insulin

A client with a history of emphysema from long-term cigarette smoking has loss of many alveoli. When comparing the type I alveolar cell physiologic function with the primary role of type II alveoli, the nurse would be aware that the type II alveoli are responsible for:

Production of surfactant

When an Rh-negative mother has been sensitized and is pregnant with an Rh-positive fetus, what happens to the fetus?

Profound red cell hemolysis

Chronic leukemia are malignancies involving what type of abnormal blood cells in the marrow?

Proliferation of well-differentiated cells

A client sustained an injury 3 days ago. The nurse is assessing the status of the wound and anticipates the wound to be in which phase of healing?

Proliferative

A hospital client with a diagnosis of type 1 diabetes has been administered a scheduled dose of regular insulin. Which of the following effects will result from the action of insulin?

Promotion of glucose uptake by target cells

In isotonic fluid volume deficit, changes in total body water are accompanied by:

Proportionate losses of sodium

What chemical mediator(s) from injured cells initiate the fever response?

Prostaglandins and interleukins

Which of the following assessment findings would lead the nurse to suspect the client has nephrotic syndrome?

Proteinuria and generalized edema

Which of the following aspects of the function of the nucleus is performed by ribosomal RNA (rRNA)?

Providing the site where protein synthesis occurs

Which of the following aspects for the function of the nucleus is performed by ribosomal RNA?

Providing the site where protein synthesis occurs.

A patient has pus containing foul smelling fluid leaking from a site of inflammation. The nurse would document this type of exudate as?

Purulent

When explaining the function of glycolysis as it relates to anaerobic metabolism, the faculty will mention which of the following key points? Select all that apply.

Pyruvic acid is an end result from a series of reactions that converts glucose. Glycolysis occurs in mitochondrion-lacking cells.

A young adult is preparing to begin treatment for non-Hodgkin lymphoma (NHL), a disease that has disseminated widely. What is the most likely treatment regimen for this client?

Radiation and chemotherapy

An elderly client is dressed only in a hospital gown and complains of a draft in her room. Consequently, she has requested a warm blanket while she sits in her wheelchair. Which of the following mechanisms of heat loss is most likely the primary cause of her request?

Radiation and convection

Despite the low levels of radiation used in contemporary radiologic imaging, a radiology technician is aware of the need to minimize her exposure to ionizing radiation. What is the primary rationale for the technician's precautions?

Radiation interferes with DNA synthesis and mitosis

Alloimmune reaction

Reactions directed at antigens of tissue of other

The Golgi complex, or Golgi bodies, consists of stacks of thin, flattened vesicles or sacs within the cell. These Golgi bodies are found near the nucleus and function in association with the endoplasmic reticulum (ER). What is one purpose of the Golgi complex?

Receive proteins and other substances from the cell surface by a retrograde transport mechanism

An elderly female client has been hospitalized for the treatment of acute pyelonephritis. Which of the following characteristics of the client is most likely implicated in the etiology of her current health problem? The client:

Recently had a urinary tract infection

A client sought care bc of recent malaise and high fever. He said his fever started 2 days earlier, lasted 2 weeks, with a cycle of high fever for a couple days followed by a day or two of normal temperature. What pattern is he experiencing?

Recurrent fever

The parents of a 4-year-old girl have sought care because their daughter has admitted to chewing and swallowing imported toy figurines that have been determined to be made of lead. Which of the following blood tests should the care team prioritize?

Red blood cell levels and morphology

A client is admitted to the hospital with first-and second degree burns. Which assessment findings are associated with first-degree burns? Select all that apply.

Red or pink Painful

A diabetic client has injured his foot walking barefoot. What are local cardinal signs of acute inflammation?

Redness and edema at injury site.

A diabetic client has injured his foot while walking barefoot on the lawn. On admission, what assessment findings would be considered a localized cardinal sign of acute inflammation (other than heat)?

Redness and edema at the injured site

A diabetic client has injured his foot while walking barefoot on the lawn. On admission, which assessment finding would be considered a localized cardinal sign acute inflammation?

Redness and edema at the injured site

A diabetic client has injured his foot while walking barefoot on the lawn. On admission, which of the following assessment findings would be considered a localized cardinal sign of acute inflammation?

Redness and edema at the injured site

Diabetic client has injured his foot while walking barefoot on the lawn. On admission, which of the following assessment findings would be considered a localized cardinal sign of acute inflammation?

Redness and edema at the injured site

Older adult client with a history of chronic obstructive pulmonary disease (COPD) develops a fever of 38.3ºC (101ºF). What is the primary reason for the nurse to implement temperature lowering measures?

Reduce oxygen demand

Atrophy

Reduction of workload or resources causes reduction in normal cell components. (Decrease cell size due to loss of cellular component)

The client is undergoing diagnostic workup for possible Hodgkin type of lymphoma. Which of the following laboratory results would confirm the diagnosis of Hodgkin lymphoma?

Reed-Sternberg cells

The patient is diagnosed with Hodgkin type of lymphoma based on the result of laboratory tests and study of the tumor cells. A distinct characteristic of Hodgkin lymphoma is the presence of;

Reed-Sternberg cells

Ischemia and other toxic injuries increase the accumulation of intracellular calcium as a result of:

Release of stored calcium from the mitochondria

To effectively relay signals, cell-to-cell communication utilizes a chemical messenger system to:

Release secretions into extracellular fluid

Knowing that both binge-eating and bulimia nervosa clients consume excessive amounts of foods secretively, which assessment data lead the high school nurse to suspect the student is a binge eater?

Remain overweight

A client who had an implantable cardioverter-defibrillator (ICD) returns the next week with a fever, chills, and elevated WBC. The physician suspects the wound is infected. If this wound does not respond to antibiotic therapy, the nurse can anticipate the client will undergo:

Removal of device

Exocytosis allows granular content to be released into extracellular fluid by:

Removing cellular debris & releasing synthesized substances like hormones

Exocytosis allows granular content to be released into extracellular fluid by:

Removing cellular debris and releasing synthesized substances like hormones

The client recently returned from a year of military battle duty and has posttraumatic stress disorder (PTSD). This disorder includes an "intrusion" state that is experienced as:

Repeated relived memories as nightmares

Immunity

Resistance to a disease causing organism.

A client is devastated to receive a diagnosis of amyotrophic lateral sclerosis (ALS). The symptomatology of this disease is a result of its effects on upper and lower motor neurons. The health care provider caring for this client will focus on which priority intervention for this client?

Respiratory ventilation assessment and prevention of aspiration pneumonia

Which of the following statements is true for both prokaryotic and eukaryotic cells?

Ribosomes synthesize proteins using mRNA sequences.

Assessment of an elderly female client reveals the presence of bilateral pitting edema of the client's feet and ankles and pedal pulses that are difficult to palpate. Auscultation of the client's lungs reveals clear air entry to bases, and the client's oxygen saturation level is 93%, and vital signs are within reference ranges. What is this client's most likely health problem?

Right-sided heart failure

Which of the following signs and symptoms is most indicative of Ménière disease?

Rotary vertigo and tinnitus

There are two forms of endoplasmic reticulum (ER) found in a cell. They are the rough and the smooth ER. What does the rough ER do in a cell?

Rough ER does protein synthesis

While teaching a class of nursing students about spinal cord injury, the instructor mentions that male SCI clients will be able to have a sexual response if their injury is at which level on the spinal column?

S4

A recently injured (3 months ago) client with a spinal cord injury at T4 to T5 is experiencing a complication. He looks extremely ill. The nurse recognizes this as autonomic dysreflexia (autonomic hyperreflexia). His BP is 210/108; skin very pale; gooseflesh noted on arms. The priority nursing intervention would be to:

Scan his bladder to make sure it is empty

Which disease is considered to be caused by a lack of a specific vitamin or mineral?

Scurvy

Which of the following diseases would be considered to be caused by a lack of a specific vitamin or mineral?

Scurvy

Establishment of a stable airway and intravenous access, as well as the administration of epinephrine

Select the most important intervention in the management of anaphylaxis.

Which of the following assessments should be the priority when caring for a client with anorexia nervosa?

Serum electrolyte levels

Systemic immune complex type III hypersensitivity disorder that is caused by insoluble antigen-antibody complexes being deposited in blood vessels, the joints, the heart, or kidney tissue is called?

Serum sickness

An 86-year-old female client has been admitted to the hospital for the treatment of dehydration and hyponatremia after she curtailed her fluid intake to minimize urinary incontinence. The client's admitting laboratory results are suggestive of prerenal failure. The nurse should be assessing this client for which of the following early signs of prerenal injury?

Sharp decrease in urine output

"Your HAART medications are working to slow the progression of the disease."

Shortly after being diagnosed with HIV, a client has begun highly active antiretroviral therapy (HAART). The client asks, "My doctor tells me that my viral load is going down. What does that mean?" The nurse's best response is:

Fungi

Simple organisms that grow as single cells or thread like filaments

Bacteria

Single celled organisms without nucleus

If the parasympathetic neurotransmitter releases acetylcholine, the nurse should anticipate observing what changes in the ECG pattern?

Slowing of heart rate to below 60 beats/minute

A nurse caring for a client with a diagnosis of diabetes insipidus (DI) should prioritize the close monitoring of which of the following electrolyte levels?

Sodium

A nurse who is providing care for a patient with a diagnosis of diabetes insipidus should prioritize the close monitoring of serum levels of which of the following electrolytes?

Sodium

While explaining cellular membrane potentials, the instructor states that during an action potential, the cell membrane becomes more permeable to which mineral/electrolyte?

Sodium

Anaphylactic reaction

Some members of the population are so sensitive to certain antigens that they react within minutes by developing itching, hives, and skin erythema, followed shortly thereafter by bronchospasm and respiratory distress. What is this near-immediate reaction commonly known as?

Which of the following meals is most likely to exacerbate an individuals celiac disease?

Spaghetti with meatballs and garlic bread

A client has arrived in the emergency department with massive blood loss from a lacerated liver. Knowing the physiological principles behind the capacity for cells to regenerate, the client's body will likely:

Stimulate rapid proliferation of RBCs by the blood-forming progenitor cells of the bone marrow

Gastrin production, a task that is performed by the stomach results in which of the following effects?

Stimulation of HCl secretions by parietal cells

The ulcerative colitis patient should be assessed by the health care provider for which of the following clinical manifestations? Select all that apply.

Stool containing blood, persistent diarrhea

While sponging a client who has a high temperature, the nurse observes the client begins to shiver. At this point, the priority nursing intervention would be to:

Stop sponging the client and retake a set of vital signs

A client who has been taking 80 mg of prednisone, a glucocorticoid, each day has been warned by his primary care provider to carefully follow a plan for the gradual reduction of the dose rather than stopping the drug suddenly. What is the rationale for this directive?

Stopping the drug suddenly may cause acute adrenal insufficiency

A 70-year-old client admitted to a hospital for a prostatectomy is surprised to learn that his physician has prescribed insulin on a sliding scale, despite the fact that the client successfully manages his type 2 diabetes using diet and oral antihyperglycemics when at home. Which of the following facts may underlie the physician's action?

Stress and illness can increase glycogenolysis and insulin resistance.

Antigens

Substances foreign to the host that can stimulate an immune response are known as:

When caring for a client with a wound that is healing by primary intention, the nurse recognizes which characterization best describes this type of wound?

Surgical incision

Which of the following clients is most likely to be susceptible to developing a neurogenic fever? A client who has:

Sustained a head injury in a bicycle crash

In the chemical phase of the inflammatory response, increased capillary permeability causes fluid to move into tissue. what characteristic cardinal signs are associated with increased capillary permeability?

Swelling

While educating a class about adipocytes in adipose tissue, the faculty will emphasize these cells not only serve as storage sites but also are able to:

Synthesize triglycerides

Which immune cells bind with cells that have been infiltrated by pathogens?

T-cells

A Jewish couple that is trying to conceive ask the nurse if they should receive genetic counseling. The nurse is aware that an autosomal recessive disorder that causes a deficiency in hexosaminidase A may be present. What disorder should the couple be tested for?

Tay-Sachs

A client's primary care provider has recommended biofeedback in an effort to address her chronic stress and reduce the potential for complications. What will be the goal of this intervention?

Teaching the client to consciously control her own body functioning

Concept of replicative senescence

Telomeres become progressively shorter in successive generations of a cell

Which of the following facts underlies the concept of replicative senescence?

Telomeres become progressively shorter in successive generations of a cell

Which of the following facts underlies the concept of replicative senescence?

Telomeres become progressively shorter in successive generations of a cell.

Patient is experiencing anorexia, myalgia, arthralgia, headache, and fatigue. The nurse should assess for which of the following?

Temperature

Which of the following most accurately explains why the Golgi complex is often near the rough ER?

The Golgi complex modifies and packages proteins that are produced in the rough ER.

When lecturing to a group of students about the pathophysiological principles behind heart failure, the instructor explains that cardiac output represents:

The amount of blood the heart pumps each minute

While explaining wound healing to an older adult client with a diabetic ulcer on the foot, which statement best explains the early process

The basement membrane must restored before epithelia cells can form a covering over the wound.

Antibodies

The cells that mediate humoral immunity do so because they are capable of producing:

A client has experienced a myocardial infarction with necrosis of cardiac muscle. What ramifications are there since that muscle is permanent tissue?

The cells will not proliferate and will be replaced by scar tissue

A client has experienced a myocardial infarction with accompanying necrosis of cardiac muscle, a permanent tissue. What are the ramifications of the fact that cardiac muscle is a permanent tissue?

The cells will not proliferate and will be replaced with scar tissue

A client has experienced a myocardial infarction with accompanying necrosis of cardiac muscle, a permanent tissue. What are the ramifications of the fact that cardiac muscle is a permanent tissue?

The cells will not proliferate and will be replaced with scar tissue.

A patient has experienced a myocardial infarction with accompanying necrosis of cardiac muscle, a permanent tissue. What are the ramifications of the fact that cardiac muscle is a permanent tissue?

The cells will not proliferate and will be replaced with scar tissue.

A client has presented to the emergency department after twisted his ankle while playing soccer. Which assessment findings are cardinal signs that the client is experiencing

The client is experiencing pain The ankle appears to be swollen Th ankle is warmer than the unaffected ankle The client's ankle is visibly red

As people get older, their immune system does not respond as well as it did when they were younger.

The daughter of a 79-year-old woman asks the nurse why her mother gets so many infections. The daughter states, "My mother has always been healthy, but now she has pneumonia. Last month she got cellulitis from a bug bite she scratched. The month before that was some other infection. How come she seems to get sick so often now?" What is the nurse's best response?

During physiology class, the instructor asks students to explain the pathology behind development of multiple sclerosis. Which student gave the most accurate description?

The demyelination and subsequent degeneration of nerve fibers and decreased oligodendrocytes, which interfere with nerve conduction

Lysis

The destruction of bacteria, usually by disruption of the cell membrane

Which of the following is true regarding apoptosis? a) The endotoxin found in the outer cell membrane of gram-negative bacteria may activate the extrinsic pathway. b) The execution of only the intrinsic pathway is carried out by proteolytic enzymes called caspases. c) The intrinsic pathway involves the activation of receptors such as tumor necrosis factor (TNF). d) The extrinsic pathway, or mitochondrion-induced pathway of apoptosis, is activated by conditions such as DNA damage.

The endotoxin found in the outer cell membrane of gram-negative bacteria may activate the extrinsic pathway. Explanation: Endotoxins found in the outer cell membrane of gram-negative bacteria activate the extrinsic pathway. The intrinsic pathway, or mitochondrion-induced pathway of apoptosis, is activated by conditions such as DNA damage. The execution phase of both pathways is carried out by proteolytic enzymes called caspases. The extrinsic pathway involves the activation of receptors such as tumor necrosis factor (TNF)

coats a microbe to activate phagocytosis recognition.

The entrance of a microbe into an individual's vascular space has initiated opsonization. How will the health care provider explain this process that is critical in stopping the infiltration of the microbe through opsonization? Opsonization:

Skin and mucous membranes

The first physical line of defense in innate immunity is:

A cytologist uses a microscope to compare a human hepatocyte and a human epithelial cell. She observes that the smooth endoplasmic reticulum (ER) of the hepatocyte is larger and contains more folds than the smooth ER of the epithelial cell. Which of the following is best supported by this observation?

The hepatocyte is able to synthesize lipids more efficiently than the epithelial cell.

Client has developed an infection from an endogenous source. The nurse recognizes this as acquired from:

The host's own microbial flora, as would be the case in an opportunistic infection

CD4+ T lymphocytes

The human immunodeficiency virus (HIV) acts on which type of cells?

Which of the following best describes how the internal environment of a lysosome compares to the environment of the cytosol?

The internal environment of a lysosome is more acidic than the cytosol.

Which of the following factors differentiates chronic leukemias from acute leukemias?

The leukemic cells are more fully differentiated than in acute leukemias

Which of the following best explains why excess lipids accumulate in individuals with Wolman disease?

The lysosomes contain an insufficient amount of lipid-digesting enzymes.

Which of the following is the most likely outcome of Leigh syndrome on affected mitochondria?

The mitochondria cannot produce sufficient ATP to power normal cellular processes.

When discussing digestion with a group of nursing students, the instructor asks, "where does the digestion of our food begin?" the most accurate student response would be?

The mouth with ptyalin and amylase breaking down starches

A student uses a microscope to compare a human skeletal muscle cell and a human epithelial cell. The student observes that the muscle cell has more mitochondria than the epithelial cell. Which of the following conclusions is best supported by this observation?

The muscle cell requires more ATP than the epithelial cell.

"Immunization is an important means of inhibiting the spread of infection by decreasing your child's susceptibility to the infection."

The nurse is administering a childhood vaccine to a pediatric client. The mother asks the nurse why the child needs so many vaccinations. How should the nurse respond?

Hypotension Bronchospasm and respiratory distress Laryngeal edema and obstruction Vomiting and abdominal cramps

The nurse is caring for a client experiencing a systemic anaphylactic reaction. Which assessment findings correlate with this diagnosis? Select all that apply. (4)

Histamine

The nurse is caring for a client who has developed facial swelling related to a hypersensitivity reaction to a medication. Which of these substances does the nurse recognize is implicated in causing this type of reaction?

Tissue hypoxia

The nurse is caring for a client who has experienced hypovolemic shock secondary to penetrating multiple trauma. When caring for the client postoperatively, which of these factors does the nurse recognize places the client at risk for poor wound healing?

Heart rate

The nurse is caring for a client whose temperature is increasing. Which other vital sign/body response will the nurse monitor for an increase?

Thrombocytes

The nurse is caring for a client with a stab wound. The nurse recognizes that deficiency of which of these elements of the blood will interfere with hemostasis at the site of injury?

Elevated C-reactive protein Fever Decreased cardiac output Intravascular fluid deficit

The nurse is caring for a client with systemic inflammatory response syndrome (SIRS) and septic shock. Which of these assessment findings is consistent with SIRS? Select all that apply. (4)

EpiPen

The nurse is educating a client who is being discharged from the hospital with severe allergies and who has experienced an anaphylactic reaction. What should the client be advised to have on hand at all times?

Leukocytosis

The nurse is evaluating the bloodwork results of a client with an infected leg ulcer. The white blood cell count is 18,000 cells/uL. The nurse interprets this as:

They are substances foreign to the host that can stimulate an immune response

The nurse is explaining to a colleague the basis of a client's allergy to dust. What statement by the nurse most accurately describes antigens in this situation?

Family members should not come in contact with your blood.

The nurse is teaching a male client who has been diagnosed as HIV positive. The client asks what precautions he should take to prevent his family members from contracting HIV. Which statement will the nurse include in teaching this client?

"I will breast-feed my baby since there is no risk of my baby getting the AIDS virus that way." "I just know my baby will be born with AIDS." "If I have a cesarean birth, my baby will not become HIV positive."

The nurse knows that further teaching is needed when a client who is pregnant and also HIV positive states which of the following? Select all that apply.

Redness

The nurse reads in the medical record that a client's surgical wound is ruborous. When assessing the client the nurse anticipates the characteristics of the wound will reflect which of these?

A client has been diagnosed with a gram-negative bacillus in his blood cultures. The health care providers know these bacteria may produce clinical manifestations such as high temperature, high respiratory rate, and low blood pressure. These manifestations are primarily caused by:

The outer layer of the bacterial membrane acting as an endotoxin

A female patient with a history of chronic renal failure has developed hypocalcemia. Which of the following assessment findings would provide potential confirmation of this diagnosis?

The patient has muscle spasms and complains of numbness around her mouth

Following peripheral blood testing and a bone marrow biopsy, a client has been diagnosed with chronic myelogenous leukemia. Which of the following is most likely to have preceded the client's diagnosis?

The presence of a Philadelphia chromosome

Which of the following best describes a difference between the rough ER and the smooth ER?

The rough ER has ribosomes on its surface, but the smooth ER does not.

Which body tissue exhibits the highest rate of turnover and renewal?

The squamous epithelial cells of the skin

A couple has chosen to pay for the harvesting and storage of umbilical cord blood after the delivery of their child to secure a future source of embryonic stem cells. What is the most likely rationale for the couple's decision?

The stem cells may be able to produce a wide range of body cells

A couple has chosen to pay for the harvesting and storage of umbilical cord blood after the delivery of their child to secure a future source of embryonic stem cells. What is the most likely rationale for the couple's decision?

The stem cells may be able to produce a wide range of body cells.

A couple have chosen to pay for the harvesting and storage of umbilical cord blood after the delivery of their child to secure a future source of embryonic stem cells. What is the most likely rationale for the couple's decision?

The stem cells may be able to produce a wide range of body cells.

A couple chose to store umbilical cord blood for their baby for future stem cell source. Why?

The stem cells may be able to produce wide range of body cells.

A student is participating in an extended fast as part of a charitable fundraising effort. Which of the following is an example of the physiologic reserve that will facilitate the student's adaptation to the stress of this sudden change in diet?

The student's adipose tissue contains large and accessible stores of energy.

Each skeletal muscle is a discrete organ made up of hundreds or thousands of muscle fibers. Although muscle fibers predominate, substantial amounts of connective tissue, blood vessels, and nerve fibers are also present. What happens during muscle contraction?

The thick myosin and thin actin filaments slide over each other, causing shortening of the muscle fiber.

A lung biopsy and magnetic resonance imaging have confirmed the presence of a benign lung tumor in a client. Which of the following characteristics is associated with this client's neoplasm?

The tumor will grow by expansion and is likely encapsulated

A lung biopsy and magnetic resonance imaging have confirmed the presence of a benign lung tumor in a client. Which of the following characteristics is associated with this client's neoplasm?

The tumor will grow by expansion and is likely encapsulated.

A lung biopsy and magnetic resonance imaging have confirmed the presence of a benign lung tumor in a patient. Which of the following characteristics are associated with this patient's neoplasm?

The tumor will grow by expansion and is likely encapsulated.

Which of the following statements about the unknown cell is most likely true?

The unknown cell can perform photosynthesis.

Following a biopsy, a 54 year-old man has been diagnosed as having a benign neoplastic tumor. Which of the following characteristics most likely applies to his tumor?

The well-differentiated cells are clustered together in a single capsulated mass

Malnutrition Infections Hyperglycemia

The wound care nurse is teaching a group of nurses about wound healing and, specifically, delays in wound healing. Which situations that interfere with wound healing, and could cause a delay in healing, should the nurse include in the discussion? Select all that apply.

T lymphocytes B lymphocytes

There are many cells that make up the passive and adaptive immune systems. Which cells are responsible for the specificity and memory of adaptive immunity? Select all that apply.

T lymphocytes B lymphocytes

There are many cells that make up the passive and adaptive immune systems. Which cells are responsible for the specificity and memory of adaptive immunity? Select all that apply. (2)

Patients are commonly administered antipyretics when their oral temp exceeds 37.5 (99.5). Which of the following statements related to the rationale for this action is most accurate?

There is little empirical evidence for theis treatment modality

A client arrived in the emergency department following 2 days of general malaise. The temperature is 103.8°F. The nurse administers the prescribed aspirin, an antipyretic. Which of the following statements relates to the rationale for this action?

These medications protect vulnerable organs, such as the brain, from extreme temperature elevation.

Why is aspirin given to combat pain and inflammation?

They deactivate the enzyme in cyclooxygenase, preventing synthesis of prostaglandin, a potent inflammatory mediator.

Which identifies correctly how the G protein-linked receptors are similar?

They have a ligand-binding extracellular receptor component, which causes changes that activate the G protein on the cytoplasmic side of the cell membrane.

Which of the following best describes the function of the structures labeled X?

They translate mRNA into amino acid sequences.

A young child develops type 1A diabetes. The parents ask, "They tell us this is genetic. Does that mean our other children will get diabetes?" The best response by the healthcare provider is:

This autoimmune disorder causes destruction of the beta cells, placing your children at risk risk of developing diabetes

Virus

Thread-like particles that reproduce by taking over living cells. Totally dependent upon living host for survival and materials to replicate.

A patient was started on a protocol for the prevention of deep vein thrombosis shortly after admission, and has been receiving 5000 units of heparin twice daily for the last 5 days. An immune response to this treatment may increase the patient's chance of developing which health problem?

Thrombocytopenia

A 48-year-old male client, who normally enjoys good health, has been admitted to the hospital for the treatment of polycythemia vera. The nurse who is providing care for the client should prioritize assessments aimed at the early identification of which of the following health problems?

Thromboembolism

a 48 year old patient, who normally enjoys good health, has bee admitted to the hospital for the treatment of polycythemia vera. The nurse who is providing care for the patient should prioritize assessments aimed at the early identification of which of the following health problems?

Thromboembolism

The most important complication of atherosclerosis that may cause occlusion of small heart vessels is

Thrombosis

Dysplasia

Tissue loses organization in harsh environment. (disorder growth)

A man sustained a puncture injury to his chest that caused a tension pneumothorax to form. This is a life-threatening condition because:

Trapped, inspired air collapses the lung

Diabetes mellitus and hypertension are two major causes of glomerular injury. True False

True

Esophageal varices result from portal hypertension. True False

True

In the adult, the kidneys are perfused with 20% to 25% of the cardiac output. True False

True

T or F: Cells are able to adapt to changes in work demands or threats to survival by changing their size, number, and type.

True

T or F: Cells with a similar embryonic origin or function are often organized into larger functional units called tissues.

True

T or F: Chronic inflammation is self-perpetuating and may last for weeks, months, or even years.

True

T or F: Prolonged exposure to cold increases blood viscosity and induces vasoconstriction.

True

T or F: The glycolytic pathway does not require oxygen to produce cellular energy.

True

Which of the following dietary guidelines should a nurse provide to a group of older adults to possibly decrease their risks of developing colon cancer?

Try to minimize fat and maximize fiber when you're planning your meals.

A client had excessive blood loss and prolonged hypotension during surgery. His postoperative urine output is sharply decreased, and his blood urea nitrogen (BUN) is elevated. The most likely cause for the change is acute:

Tubular necrosis

A breast cancer client has just learned that her tumor clinical stage is T3, N2, M0. After the physician leaves, the client asks the nurse to explain this to her again. The nurse will use which of the following statements in his or her answer? Your:

Tumor is large and at least two lymph nodes are positive for cancer cells

A breast cancer client has just learned that her tumor clinical stage is T3, N2, M0. After the physician leaves, the client asks the nurse to explain this to her again. The nurse will use which of the following statements in his or her answer? Your:

Tumor is large and at least two lymph nodes are positive for cancer cells.

A pregnant client in a nutrition class learns that she should avoid what food that contains methyl mercury?

Tuna

A pregnant client is attending a nutrition class for first-time moms. During the class, the instructor stressed that they should avoid consumption of which food that may cause brain damage from methyl mercury exposure?

Tuna

Knowing that excess adipose tissue can result in chronic inflammation, the high school nurse should be assessing obese students for which of the following health problems?

Type 2 diabetes

When discussing adolescent health with a group of high school teachers, the school nurse shares that each year their student population is becoming increasingly obese. Therefore, the school nurse took the opportunity to educate the teachers about signs and symptoms of which of the following high-risk disease processes?

Type 2 diabetes mellitus

The nurse is assessing a client with diabetes and notes an area on the client's right foot as inflamed, necrotic, and eroded. The client states he accidentally slammed his foot in a door 2 weeks ago. The nurse would document this finding as

Ulceration

A client with hepatitis B asks the nurse, "How did I get this hepatitis?". The nurse responds that the mode of transmission is predominantly by:

Unprotected sexual intercourse; intravenous drug use

A 5-year-old girl's diagnosis of bone cancer required an aggressive treatment regimen. Which of the following considerations forms the most significant threat to her future health?

Unwanted effects of chemotherapy and radiation therapy

In an ICU setting, one assessment that would lead the nurse to suspect shock has resulted in decrease blood flow to vital organs is:

Urine output less than 20 mL/hour

Many factors contribute to the incontinence that is common among the elderly. A major factor is increased:

Use of multiple medications

A client is undergoing a bioelectrical impedance test to estimate body fat. The nurse will explain to the client that this test involves:

Using electrodes to send harmless current through the body to measure resistance to estimate body fat

A client with a long history of cigarette smoking and poorly controlled hypertension has experienced recent psychomotor deficits as a result of hemorrhagic brain damage. The client's psychomotor deficits are likely the result of:

Vascular dementia

Client who has developed a fever is now complaining of a headache. The nurse would recognize this manifestation as a result from the:

Vasodilatation of cerebral vessels

A client cuts herself with a sharp knife while cooking dinner. The client describes how the wound started bleeding and had a red appearance almost immediately. The nurse knows that in the vascular stage of acute inflammation, the vessels:

Vasodilate causing the area to become congested causing the red color and warmth

In the vascular, hemodynamic stage of acute inflammation, initial vasoconstriction is followed by which of the following assessment findings?

Vasodilation and swelling

While on a cruise to the Caribbean, a person develops "motion sickness" with associated malaise, and nausea/vomiting. The nurse notes the client's BP is 88/52; pulse is 110; and skin moist with perspiration. The client diagnosis related to the clinical manifestations would most likely be:

Vertigo

A client with a long-standing diagnosis of human immunodeficiency virus (HIV) has recently developed neutropenia and been admitted to a hospital. Which of the following measures should be prioritized by the nurses who are providing his care?

Vigilant handwashing to protect against severe bacterial infections

A patient with a long standing diagnosis of human immunodeficiency virus (HIV) has recently developed neutropenia and been admitted to a hospital. Which of the following measures should be prioritized by the nurses who are providing his care?

Vigilant infection control and handwashing

The nurse is assessing a client with acute sinusitis. What is the primary cause of this condition?

Viral infection transmitted via respiratory droplets.

Which of the following characteristics differentiates a migraine with aura from a migraine without aura?

Visual symptoms that precede the headache

A patient with a diagnosis of schizophrenia has been admitted to the emergency department after ingesting more than 2 gallons of water. Which of the following pathophysiologic processes may result from the sudden water gain?

Water movement from the extracelluar to intracelluar compartment

A client diagnosed with schizophrenia has been admitted to the emergency department (ED) after ingesting more than 2 gallons of water in one sitting. Which of the following pathophysiologic processes may result from the sudden water gain?

Water movement from the extracellular to the intracellular compartment

A client has been diagnosed with a cerebral aneurysm and placed under close observation before treatment commences. Which of the following pathophysiologic conditions has contributed to this client's diagnosis?

Weakness in the muscular wall of an artery

While teaching a client with new-onset right-sided heart failure, the nurse should educate the client to monitor for fluid accumulation by:

Weighing every day at the same time with same type of clothing

A patient with a vascular disease complains of weakness and discomfort in their left foot. The foot is cold, swollen and clammy. No pulse and has an odor. What is most likely the diagnosis?

Wet gangrene

Cellular bags of granules located in the loose connective tissue close to blood cells. Skin, digestive lining, and respiratory tract. It secretes histamine to dilate arteries.

What are mast cells and what is their role in the inflammatory process?

Flora is a nonpathogenic microorganism on the body surface. They compete with pathogens for nutrients and space and they produce toxins that are harmful to some pathogenic microorganisms. Serves as both as a defense mechanism against infection, and is also a source of potential pathogens.

What are normal bacterial flora and how do they protect the body against infection?

Western Blot: detects antibodies to 4 HIV antigens. If 2 antigens show up then its positive. Viral loading testing: Measures presence of HIV viral material. Monitor progression of disease.

What are the 2 tests that are used to monitor the progression of the HIV disease process and what are the lab values for a patient with HIV?

1. Prevent and limit injury and further damage. 2. Initiate adaptive immune response. 3. Initiate healing

What are the 3 goals of the inflammatory response?

Redness: due to local vessel dilation Heat: Due to local vessel dilation Swelling: Due to influx of plasma proteins and phagocytic cells into the tissue spaces Pain: Due to local release of enzymes and increased tissue pressure Loss of function: Pain can inhibit mobility or from severe swelling that prevents movement.

What are the 5 s/s of the inflammatory response and what is taking place to cause each?

Opportunistic infections, lymphadenopathy, SOB, cough, confusion, dementia complex, personality changes, rashes, dry itchy skin, kaposi's lesions, diarrhea, weight, n/v, pneumocystis pneumonia, toxoplasmosis, candidas albicans, herpes simplex virus, mycobacterium tuberculosis, cryptosporidiosis.

What are the chronic assessment findings in the patient with AIDS?

Fever, night sweats, chills, headache, muscle aches, rash, sore throat

What are the initial assessment findings in the first month after being infected with HIV?

Unprotected sex with an infected partner. Sharing needles with infected person. Transmission from infected mother to fetus. Infection from blood products.

What are the methods of transmission of HIV?

4,500 - 10,000 WBC/mcL

What are the normal results of the WBC test?

AB

What blood type is the universal recipient?

O

What blood type that is the universal donor?

Identifies the relative count of each 5 types of WBC's

What is a WBC differential test?

A chemical that starts the process that hustle those allergens out of your body or off your skin. Dilate capillaries. Release pyrogens that stimulate hypothalamus to increase temp. Activate pain receptors. Draes WBCs to site. Platelet activation.

What is histamine and what response in the body do they stimulate?

Macrophages

What is the WBC that responds later in the inflammatory response?

Erythrocyte sedimentation rate

What is the definitive test for diagnosis an infection?

Helper recognizes self cells versus non self cells. If non self cells secrete cytokines to increase WBC activity. Suppressors prevent hypersensitivity and releases cytokines to inhibit response.

What is the difference between T-cells and suppressor T-cells?

Neutrophils

What is the first WBC to respond in the inflammatory response?

Physical barriers

What is the first line of defense in the immune system?

Vascular stage. Mast cells secrete histamine that dilates arteries. (vasodilation. Increased capillary permeability.

What is the first stage of the inflammatory response and what is taking place?

Detecting and quantifying soluble substances such as peptides, proteins, antibodies, and hormones.

What is the first test used to diagnose for HIV and what is it testing for?

To prolong immune function.

What is the goal of drug therapy used for HIV?

WBC

What is the name of the first test completed to see f if the body has responded to a pathogen?

An abnormal response to a foreign antigen

What is the pathogen that causes AIDS?

Identify microbe

What is the purpose of a culture test?

Identifies which antibiotics the microbe is sensitive to in order to guide treatment.

What is the purpose of a sensitivity test?

Preparation of killed microorganisms, living attenuated organisms, or living fully virulent organisms that is administered to produce or artificially increase immunity to a particular disease

What is the purpose of a vaccine?

Purpose is to neutralize, eliminate or destroy foreign organisms.

What is the purpose of the immune system?

Inflammatory response

What is the second line of defense in the immune system?

Exudate (cellular stage). WBC's and platelets move toward damaged cells (hours after invasion). WBC's remove pus, platelets control excess bleeding, prevent spread of infective agent, heparin released to maintain blood flow to area.

What is the second stage of the inflammatory response and what is taking place?

Western Blot Assay

What is the second test used to confirm a diagnosis of HIV?

Adaptive immunity

What is the third line of defense in the immune system?

Tissue repair and replacement. Phagocytes consume bacteria and cell debris; tissue heals

What is the third stage of the inflammatory response and what is taking place?

Call RRT, apply oxygen, DC allergen (IV drug), Give Benadryl and epinephrine IV

What is the treatment for anaphylaxis?

Antimicrobials.

What is the treatment for bacterial infections?

Chronic multisystem inflammatory disease of connective tissue.

What takes place in the tissues to cause an autoimmune response?

Defense against parasites and regulation of vascular system. Elevated levels in allergic reactions.

What type of situation do eosinophils predominatly respond to?

Infection

When a pathogenic microorganism invades the body and causes disease.

a second test known as the Western blot assay will be ordered to confirm positive HIV status.

When counseling a male client with suspected HIV, the nurse informs him that if the enzyme-linked immunosorbent assay (ELISA) comes back positive, then:

A female client asks, "Why do I leak urine every time I cough or sneeze?" The health care worker's response is based on which physiologic principle?

When intravesical pressure exceeds maximal urethral closure pressure

Hypertrophy may occur as the result of normal physiologic or abnormal pathologic conditions. The increase in muscle mass associated with exercise is an example of physiologic hypertrophy. Pathologic hypertrophy occurs as the result of disease conditions and may be adaptive or compensatory. Examples of adaptive hypertrophy are the thickening of the urinary bladder from long-continued obstruction of urinary outflow and the myocardial hypertrophy that results from valvular heart disease or hypertension. What is compensatory hypertrophy? a) When the body stimulates gene expression to begin a progressive decrease in left ventricular muscle mass b) When one kidney is removed, the remaining kidney enlarges to compensate for the loss. c) When the body controls myocardial growth by stimulating actin expression to enlarge the heart d) When the body increases its major organs during times of malnutrition

When one kidney is removed, the remaining kidney enlarges to compensate for the loss. Explanation: Compensatory hypertrophy is the enlargement of a remaining organ or tissue after a portion has been surgically removed or rendered inactive. The body does not enlarge its major organs during times of malnutrition. Gene expression, not actin expression, stimulates the body to increase the muscle mass of the heart. Hypertrophy is not a progressive decrease in the size of anything

Agglutination in a blood transfusion reaction

When people are given blood transfusions of the wrong blood group, the anybodies react with the incorrectly transfused blood group and as a result, the erythrocytes clump up and stick together

Which of the following best describes how the central vacuole helps support the structure of a plant cell?

When the central vacuole is full of water, it applies pressure to the cell wall and maintains the shape of the cell.

Production of enzymes that inactivate antibiotics Changes in the filtration qualities of the bacterial cell Genetic mutations Creation of alternative metabolic pathways that bypass antibiotic activity

When treating a client with antibiotics, a nurse recognizes that which of these can cause bacterial resistance? Select all that apply. (4)

They do not cause harm to the body. They live on nonsterile areas of the body. They provide a type of natural immunity.

Which are characteristics of resident flora? Select all that apply. (3)

Fever and tachycardia

Which body response to an acute inflammation will the nurse assess if the client is experiencing a systemic response?

Mast cells

Which cellular mediator is involved in the development of type I hypersensitivity reactions?

A client with AIDS who has a decreased CD4+ TH1 count

Which client is at the greatest risk for developing an intracellular pathogen infection?

An HIV-positive client with a CD4+ count of 100 cells/μL with pulmonary tuberculosis An HIV-positive client with a CD4+ count of 300 cells/μL and wasting syndrome

Which clients would have a confirmed diagnosis of AIDS? Select all that apply.

CD4+ count less than 200 cells/μL

Which of these is an AIDS-defining condition?

Similar immune response for each exposure Immediate response to antigen invasion

Which physiologic characteristics correspond with innate immunity? Select all that apply.

A 6-week-old infant receives antibodies from his mother's breast milk.

Which situation can best be characterized as an example of passive immunity?

Serum sickness

Which term best describes a systemic immune complex disorder that is caused by insoluble antigen-antibody complexes being deposited in blood vessels, the joints, the heart, or kidney tissue?

Which of the following clients is most likely to be in positive nitrogen balance? A client:

Who has been admitted to the hospital in early labor

Prior to undergoing diagnostic testing with contrast, it is recommended that older adult clients have their creatinine level checked. The rationale for this is to ensure the client:

Will not undergo an acute kidney injury by decreasing renal blood flow

A patient has a genetic disorder that results in an absence of the enzyme needed to repair sunlight-induced DNA damage, 2000 x higher risk of skin cancer, suffers from:

Xeroderma pigmentosum

A newborn is admitted to the hospital with a high bilirubin level of 13mg/dL. The assessment data related to this lab value includes: Select all that apply.

Yellowing of the sclera of the eye; Dark-colored urine; Yellowish discoloration of the skin

In contrast to malignancies, benign tumors are characterized by

a fibrous capsule

Which of the following surgical patients is most likely to experience enhanced wound healing as a result of his or her diet?

a patient who eats a high calorie diet and large amounts of red meat

Select the main function of nervous tissue. a) Communication b) Contractility c) Support d) Connecting body structure

a) Communication Explanation: Nervous tissue is designed for communication purposes and includes the neurons, the supporting neural structures, and the ependymal cells that line the ventricles of the brain and the spinal canal. Muscle tissue is a specialized tissue designed for contractility. Connective tissue supports and connects body structures.

Select the organelle that is responsible for the breakdown of excess and worn-out cell parts as well as foreign substances that are taken into the cell. a) Lysosomes b) Peroxisomes c) Ribosomes d) Proteasomes

a) Lysosomes Explanation: Lysosomes break down excess and worn-out cell parts as well as foreign substances that are taken into the cell. Peroxisomes contain enzymes that are used in oxidative reactions. Proteasomes are responsible for proteolysis of malformed and misfolded proteins. Ribosomes serve as sites for protein synthesis in the cell.

What are the three main parts of a typical cell? (Select all that apply.) a) Nucleus b) Cell membrane c) Vacuole d) Cell wall e) Cytoplasm

a) Nucleus b) Cell membrane e) Cytoplasm Explanation: When seen under a light microscope, three major components of the cell become evident: the nucleus, the cytoplasm, and the cell membrane. Cell walls and vacuoles are found in plants and bacteria

During a discussion on cellular components and their function, a student asked the instructor the purpose of messenger RNA (mRNA). Of the following, which is the most accurate answer? a) Performs an active role of protein synthesis, where mRNA molecules direct the assembly of proteins on ribosomes to the cytoplasm. b) Assists cells in forming neoplastic progression by altering the response of chromatin in the nuclear matrix. c) Acts as an inner nuclear support membranes for a rigid network of protein filaments that binds DNA to the nucleus. d) To transport amino acids to the site of protein synthesis.

a) Performs an active role of protein synthesis, where mRNA molecules direct the assembly of proteins on ribosomes to the cytoplasm. Explanation: The nucleus is the site for the synthesis of 3 types of RNA that move to the cytoplasm and carry out the actual synthesis of proteins. Messenger RNA copies and carries the DNA instructions for protein synthesis to the cytoplasm. Ribosomal RNA is the site of actual protein synthesis; transfer RNA transports amino acids to the site of protein synthesis.

The functional component of the cell that separates intracellular and extracellular environments is known as the: a) Plasma membrane b) Cytoskeleton c) Mitochondria d) Lysosomes

a) Plasma membrane Explanation: The plasma membrane is a thin membrane that separates the intracellular contents from the extracellular. A system of tubules and filaments known as the cytoskeleton maintains the shapes of cells and their ability to move. The lysosomes, which can be viewed as the digestive organelles of the cell, are small, membrane-enclosed sacs filled with hydrolytic enzymes. The mitochondria are literally the "power plants" of the cell because they transform organic compounds into energy that is easily accessible to the cell.

The nurse is aware of the importance of potassium and sodium for normal physiologic functions of a client. The rapid movement of potassium and sodium in actions is known as: a) Primary active transport b) Endocytosis c) Exocytosis d) Vesicular transport

a) Primary active transport Explanation: Substances transported by primary active transport are sodium, potassium, calcium, and hydrogen ions. The active transport system studied in the greatest detail is the sodium/potassium (Na+/K+)-adenosine triphosphatase (ATPase) membrane pump. Exocytosis is the mechanism for the secretion of intracellular substances into the extracellular spaces. Vesicular transport is a mechanism in which materials are transported in membrane-bound vesicles. Endocytosis occurs when materials are moved into a cell in a vesicle formed from the cell membrane.

The student is examining a tissue sample with a microscope. The student decides that it is squamous keratinized tissue. Which type of epithelial tissue is the student viewing? a) Stratified epithelium b) Simple epithelium c) Neuroepithelium d) Glandular

a) Stratified epithelium Explanation: Stratified, squamous keratinized epithelium makes up the epidermis of the skin. Simple epithelium is found in the lining of blood vessels, lymph nodes, and alveoli of the lungs. Glandular epithelium is located in exocrine and endocrine glands. Neuroepithelium is located in in olfactory mucosa, retina, and the tongue.

While reviewing the basic information related to hemodialysis, the professor explains that water molecules move through adjacent phospholipid molecules in the cell membrane by: a) osmosis. b) active transport. c) secondary active transport. d) diffusion. e) vesicular transport.

a) osmosis Explanation: Water molecules move through adjacent phospholipid molecules in the cell membrane by osmosis without actually dissolving in the region occupied by the fatty acid side of the chains. Osmosis is regulated by the concentration of nondiffusible particles on either side of the membrane, with water moving from the side with the lower concentration of particles to the side with the higher concentration.

A mother of a 3-year-old asks a clinic nurse about appropriate and safe toys for the child. The nurse should tell the mother that the most appropriate toy for 3-year-old is which? a. A wagon. b. A golf set. c. A farm set. d. A jack set with marbles.

a. A wagon.

A nurse has given home care instructions to the parent of a child with head lice. Which of the following statements indicates that the instructions are correctly understood? a. All family members must be treated. b. I must discard all of my child's personal linens. c. I must wash my child's hair with an antibacterial shampoo. d. I must keep my child at home for at least one week after treatment.

a. All family members must be treated. - Rationale: Pediculosis capitis is head lice. They are spread by direct contact and can live up to 3 days between hosts.

While in a skilled nursing facility, a male client contracted scabies, which is diagnosed the day after discharge. The client is living at her daughter's home, where six other persons are living. During their visit to the clinic, she asks a staff nurse, "What should my family do?" The most accurate response from the nurse is: a. All family members will need to be treated. b. If someone develops symptoms, tell him to see a physician right away. c. Just be careful not to share linens and towels with family members. d. After you're treated, family members won't be at risk for contracting scabies.

a. All family members will need to be treated.

A clinic nurse is reviewing her assessment on a pediatric patient. What would be assessment findings with the intestinal parasite giardiasis? Select all that apply. a. Anorexia. b. Abdominal cramps with intermittent loose stools and constipation. c. Intense perianal itching. d. Bed wetting/poor sleeping. e. Failure to thrive. f. Diarrhea and vomiting.

a. Anorexia. b. Abdominal cramps with intermittent loose stools and constipation. e. Failure to thrive. f. Diarrhea and vomiting.. - Rationale: Clinical manifestations for giardiasis are as follows: anorexia, diarrhea, vomiting, failure to thrive, abdominal cramps with intermittent loose stools and constipation, steatorrhea. Clinical manifestations for pinworms include intense perianal itching, bed wetting/poor sleep, and irritability or restlessness.

The development of a 2-year-old child is characterized by: a. Engaging in parallel play. b. Dressing self with supervision. c. Having a vocabulary of at least 500 words. d. Having attained one third of adult height.

a. Engaging in parallel play. - Rationale: Two-year-olds typically play alongside each other.

A female child, age 2, is brought to the emergency department after ingesting an unknown number of aspirin tablets about 30 minutes earlier. On entering the examination room, the child is crying and clinging to the mother. Which data should the nurse obtain first? a. Heart rate, respiratory rate and blood pressure. b. Recent exposure to communicable diseases. c. Number of immunizations received. d. Height and weight.

a. Heart rate, respiratory rate and blood pressure. - Rationale: The most important data to obtain on a child's arrival in the emergency department are vital sign measurements. The nurse should gather the other data later.

The preschooler's interest in gender sexuality is characterized by an interest in: a. His or her genitalia. b. Learning how and why his or her anatomy differs from that of other children. c. Playing with and developing friendships with children of the opposite sex. d. Spending most of his or her time with the parent of the opposite sex.

a. His or her genitalia.

Which medication used in the management of acne vulgaris has a side effect of elevated triglyceride levels? a. Isotretinoin (Accutane). b. Tretinoin (Retina-A). c. Erythromycin. d. Topical benzoyl peroxide.

a. Isotretinoin (Accutane).

An adolescent with a depressive disorder is more likely than an adult with the same disorder to exhibit: a. Negativism and acting. b. Sadness and crying. c. Suicidal thoughts. d. Weight gain.

a. Negativism and acting.

School phobia is usually treated by: a. Returning the child to school immediately with family support. b. Calmly explaining why attendance in school is necessary. c. Allowing the child to enter the school before the other children. d. Allowing the parent to accompany the child in the classroom.

a. Returning the child to school immediately with family support. - Rationale: Exposure to the feared situation can help in overcoming anxiety.

The nurse counsels the parent of an infant diagnosed with nonorganic failure to thrive. The nurse notes that the mother appears depressed and is expressing feelings of inadequacy and resentment toward her infant. Which approach by the nurse is MOST appropriate? a. Structure environment so that the mom feels accepted. b. Refer her to a family therapist. c. Tell her she is an unfit mother. d. Recommend that she attend child care classes.

a. Structure environment so that the mom feels accepted. - Rationale: Help parents increase self-esteem by making them feel accepted; nurse should empathize with parents about the difficulty of childrearing.

To form a platelet plug, platelets are attracted to the damaged vessel, and the platelet ___ occurs.

adhesion

According to Hans Selye, the first stage of the general adaptation syndrome (GAS) is:

alarm

Injured cells have impaired flow of substances through the cell membrane as a result of;

altered permeability

Immune thrombocytopenia purpura (ITP) is a/an ___ disorder that destroys ___.

autoimmune; platelets

While sponging a client who has a high temperature, the nurse observes the client begins to shiver. At this point, the priority nursing intervention would be to: a. administer an extra does of aspirin b. stop sponging the client and retake a set of vitals c. increase the room temperature by turning off the air conditioner and continue sponging the client with warmer water d. place a heated electric blanket on the client's bed

b

When working with a client who has end-stage renal disease (ESRD) and is receiving peritoneal dialysis, the concept of diffusion can be explained by which of the following statements? a) "Your potassium molecules are lipid soluble and will dissolve in the lipid matrix of your cell membranes." b) "If your potassium level is high, then K+ particles will move from your peritoneal cavity into the dialysis solution, where the concentration of K+ is lower." c) "You will need to give yourself a potent diuretic so that you can pull the potassium into your blood stream and filter the potassium out in your kidneys." d) "If you can get very warm in a sauna, you will heat up your K+ particles, and the kinetic movement of the particles will increase and pass through the cell membranes faster."

b) "If your potassium level is high, then K+ particles will move from your peritoneal cavity into the dialysis solution, where the concentration of K+ is lower." Explanation: Diffusion refers to the passive process by which molecules and other particles in a solution become widely dispersed and reach a uniform concentration because of energy created by their spontaneous kinetic movements. In the process of reaching a uniform concentration, these molecules and particles move "downhill" from an area of higher to an area of lower concentration. Lipid-soluble molecules, such as oxygen, carbon dioxide, alcohol, and fatty acids (not K+), become dissolved in the lipid matrix of the cell membrane and diffuse through the membrane in the same manner that diffusion occurs in water. Diuretics are not very effective if a person has ESRD. The rate of diffusion depends on how many particles are available for diffusion, the kinetic movement of the particles, and the number and size of the openings in the membrane through which the molecules or ions can move. The environmental temperature does not play a role in this.

The nurse monitors the client's oxygen levels and respiratory status. The nurse is aware that oxygen is necessary for: a) Lactic acid formation b) Aerobic metabolism c) Anaerobic metabolism d) Glycolytic pathway

b) Aerobic metabolism Explanation: Aerobic metabolism, which supplies 90% of the body's energy needs, occurs in the cell's mitochondria and requires oxygen. The other options do not require oxygen.

When exercising outside on an extremely warm day, the client can feel his heart pounding very rapidly. Thinking in terms of the ability of the aorta to stretch to accommodate more blood circulating during exercise and the warm environment, this is an example of the function of which fibrous protein? a) Ligaments b) Elastin c) Reticular d) Collagen

b) Elastin Explanation: Three types of fibers are found in the extracellular space: collagen, elastin, and reticular fibers. Elastin acts like a rubber band; it can be stretched and then returns to its original form. Elastin fibers are abundant in structures subjected to frequent stretching, such as the aorta and some ligaments. Collagen is the most common protein in the body. It is a tough, nonliving, white fiber that serves as the structural framework for skin, ligaments, tendons, and many other structures. Reticular fibers are extremely thin fibers that create a flexible network in organs subjected to changes in form or volume, such as the spleen, liver, uterus, or intestinal muscle layer.

Which of the following statements regarding cell metabolism is most accurate? a) Energy from foodstuffs is used to convert ATP to ADP. b) Glycolysis also provides energy in situations when delivery of oxygen to the cells is delayed. c) Anabolism consists of breaking down stored nutrients and body tissues to produce energy. d) The aerobic pathway occurs in the cytoplasm.

b) Glycolysis also provides energy in situations when delivery of oxygen to the cells is delayed. Explanation: Glycolysis provides energy in situations when delivery of oxygen to the cells is delayed or impaired. Anabolism is a constructive process, in which more complex molecules are formed from simpler ones. Energy from foodstuffs is used to convert ADP to ATP. The aerobic pathway occurs in the mitochondria

Which of the following best describes the consumption of a foreign substance, such as bacteria, in a cell? a) Anaerobic metabolism b) Phagocytosis c) Meiosis d) Hemolysis

b) Phagocytosis Explanation: Phagocytosis refers to the consumption of a foreign substance.. Energy transformation takes place without oxygen is referred to as anaerobic metabolism. Meiosis a type of cell division that results in two daughter cells each with half the chromosome number of the parent cell. Hemolysis refers to the destruction of red blood cells which leads to the release of hemoglobin from within the red blood cells into the blood plasma.

While caring for a client who has just been diagnosed with leukocyte adhesion deficiency (LAD), the client asks why he is always sick with an infection. The nurse responds, "LAD is a rare autosomal recessive disorder that results in recurrent infections because of the lack of transmigration. This means: a) I don't really understand this, but it sounds like a good question to ask your physician." b) Your white blood cells are not able to leave the blood vessels and move into the area of infection." c) Your bone marrow is damaged and can't put out enough white blood cells to fight off your infections." d) Your body doesn't make enough white blood cells."

b) Your white blood cells are not able to leave the blood vessels and move into the area of infection." Explanation: Leukocyte adhesion deficiency (LAD) is a rare autosomal recessive disorder characterized by immunodeficiency, resulting in recurrent infections. A WBC differential will reveal extremely elevated levels of neutrophils (on the order of 6-10 times normal) because they are unable to leave the blood vessels. Certain integrins play an important role in allowing white blood cells to pass through the vessel wall, a process called transmigration.

Which type of receptor binds with insulin? a) G-protein-linked b) Enzyme-linked c) Ion-channel-linked d) Intracellular

b) enzyme- linked Explanation: G-protein-linked receptors often convert an inactive precursor molecule into a second messenger. Enzyme-linked receptors either associate directly with an enzyme, or are intrinsically enzymatic. Insulin binds with enzyme-linked receptors. Ion-channel-linked receptors participate in rapid synaptic signaling in nerve and muscle cells. Intracellular receptors are located in the cytoplasm and combine directly with certain molecules such as thyroid hormone and steroids

A child with Rubeola (Measles) is being admitted to the hospital. In preparing for the admission of the child, the nurse should plan to place the child on which precautions? a. Enteric. b. Airborne. c. Protective. d. Neutropenic.

b. Airborne. - Rationale: Rubeola is transmitted via airborne particles or direct contact with infectious droplets. Airborne droplet precautions are required, and persons in contact with the child should wear masks.

Which of the following statements about acne is true? a. It is caused by dirt left on the face. b. Keeping hair clean and off the forehead tends to lessen the severity of the lesions. c. Foods high in caffeine have been demonstrated to increase the occurrence of acne. d. Aggressive washing of the face has been shown to be the best treatment for acne.

b. Keeping hair clean and off the forehead tends to lessen the severity of the lesions.

A nurse in the pediatric clinic is advising the mother of a toddler who has pinworm infestation. What should the nurse teach the mother about caring for her child during and after treatment? Select all that apply. a. How to identify pinworm eggs. b. Strategies to avoid reinfestation. c. Need for medication for the entire family. d. Importance of handwashing before eating. e. Reason for obtaining stool specimen from the child.

b. Strategies to avoid reinfestation. c. Need for medication for the entire family. d. Importance of handwashing before eating. - Rationale: Reinfestation is common because pinworms easily spread to family members who are in close contact with the child. All family members must take the medication to help prevent reinfestation. Toddlers touch dirt and then put their hands in their mouths. Handwashing for all the family members before eating can help prevent reinfestation.

Mara and Cassie are both 2-1/2 years old. They both want to play with the same toy and are fighting, grabbing the toy back and forth. Mara's mother is very upset and asks the nurse why they can't get along. You would most likely suggest that she do which of the following based on your understanding of development? a. Suggest to the mothers that they reprimanded the children and tell them to share the toy. b. Suggest to the mothers that they give each child a toy to play with. Have two toys on hand. c. Suggest that when the children begin to fight over a toy that they have the toy removed and then place the children in separate parts of the room. d. Suggest that they tell the children it is not necessary to fight, there are "plenty of toys," and provide another toy for one of the children.

b. Suggest to the mother that they give each child a toy to play with. Have two toys on hand.

The clinic nurse is assessing a child who is scheduled to receive a live virus vaccine (immunization). What are the general contraindications associated with receiving a live virus vaccine? Select all that apply. a. The child has symptoms of a cold. b. The child had a previous anaphylactic reaction to the vaccine. c. The mother reports that the child is having intermittent episodes of diarrhea. d. The mother reports that the child has not had an appetite and has been fussy. e. The child has a disorder that caused a severely deficient immune system. f. The mother reports that the child has recently been exposed to an infectious disease.

b. The child had a previous anaphylactic reaction to the vaccine. e. The child has a disorder that caused a severely deficient immune system. Rationale: The general contraindications for receiving live virus vaccines include a previous anaphylactic reaction to a vaccine or a component of a vaccine. In addition, live virus vaccines generally are not administered to individuals with a severely deficient immune system, individuals with a severe sensitivity to gelatin, or pregnant women. A vaccine is administered with caution to an individual with a moderate or severe acute illness, with or without fever.

Which is descriptive of the nutritional requirements of preschool children? a. Average daily intake of preschoolers should be about 3000 calories. b. The quality of the food consumed is more important than the quantity. c. Nutritional requirements for preschoolers are very different from requirements for toddlers. d. The requirement for calories per unit of body weight increases slightly during the preschool period.

b. The quality of the food consumed is more important than the quantity. - Rationale: It is essential that the child eat a balanced diet with essential nutrients.

A mother arrives at the emergency department with her child, stating that she just found the child sitting on the floor next to an empty bottle of aspirin. On assessment, the nurse notes that the child is drowsy but conscious. The nurse anticipates that the physician will prescribe which of the following? a. ipecac syrup. b. activated charcoal. c. magnesium citrate. d. magnesium sulfate.

b. activated charcoal. - Rationale: Where as ipecac is administered to induce vomiting in certain poisoning situations, it is not recommended as the initial treatment in the hospital setting for ingestion of salicylates. This is because ipecac does not totally remove the poison from the child's system. In this situation, the child is conscious and the ingested substance (aspirin) would not damage the esophagus or lungs from vomiting. However, activated charcoal would be prescribed as an antidote in this poisoning situation, because its action is to absorb ingested toxic substances and thus decrease absorption. Options C and D are unrelated to treatment for this occurrence.

As other mechanisms prepare to respond to a pH imbalance, immediate buffering is a result of increased

bicarbonate/carbonic acid regulation

Hypoparathyroidism causes hypocalemia by

blocking bone release of calcium

The most reliable method of measuring body water or fluid volume increase is by assessing:

body weight change

Although the basic structure of the cell plasma membrane is formed by a lipid bilayer, most of the specific membrane functions are carried out by:

bound and transmembrane proteins.

Which of the following mediators of inflammation causes increased capillary permeability and pain?

bradykinin

The Krebs cycle provides a common pathway for the metabolism of nutrients by the body. The Krebs cycle forms two pyruvate molecules. Each of the two pyruvate molecules formed in the cytoplasm from one molecule of glucose yields another molecule of what? a) H2O b) NADH + H+ c) ATP d) FAD

c) ATP Explanation: Each of the two pyruvate molecules formed in the cytoplasm from one molecule of glucose yields another molecule of ATP, which is a special carrier for cellular energy. FAD, or flavin adenine dinucleotide, is a coenzyme of protein metabolism that accepts electrons and is reduced. NADH + H+ is an end product of glycolysis. The electron transport chain oxidizes NADH + H+ and FADH2 and donates the electrons to oxygen, which is reduced to water.

A couple is scheduled to obtain familial lineage studies. The nurse is aware that an analysis will occur identifying maternally inherited: a) Extrachromosomal DNA b) G-protein-linked receptors c) Mitochondrial DNA d) Mitochondrial RNA

c) Mitochondrial DNA Explanation: Mitochondrial DNA is inherited maternally (i.e., from the mother) and provides a basis for familial lineage studies. Extrachromosomal DNA is used in the synthesis of mitochondrial RNAs and proteins used in oxidative metabolism. G-protein-linked receptors mediate cellular responses for numerous types of first messengers, including proteins, small peptides, amino acids, and fatty acid derivatives such as the prostaglandins.

A school nurse is teaching a class on immunity. Which statement contains an accurate explanation about cellular defenses? a) Ion channels allow rapid polarity changes, which inactivate invading organisms. b) Cells release enzymes into the extracellular fluid that degrade invading organisms. c) Neutrophils engulf invading organisms where lysosomes break them down. d) Active transport draws invading organisms through cell channels to be destroyed.

c) Neutrophils engulf invading organisms where lysosomes break them down. Explanation: Phagocytosis is when a cell, such as macrophages or neutrophils, engulfs an invading organism or damaged cells. Once inside the cell, the encapsulated particle is broken down by lysosomal enzymes.

A nurse is providing care for a client with a diagnosis of Crohn's disease. The nurse recognizes the fact that the disease involves the inflammation and irritation of the intestinal lining. Which of the following types of tissue is most likely involved in the client's pathology? a) Stratified epithelium b) Glandular epithelium c) Simple columnar epithelium d) Simple cuboidal epthelium

c) Simple columnar epithelium Explanation: Simple columnar epithelium lines the intestines, and is cilia and mucus-secreting goblet cells. The intestinal tract does not consist of glandular epithelium, simple cuboidal epithelium, or stratified epithelium.

A mother tells the nurse that she is very worried because her 2-year-old child does not finish his meals. What should the nurse advise the mother? a. Make the child sit with the family in the dining room until he finishes his meal. b. Provide quiet environment for the child before meals. c. Do not give snacks to the child before meals. d. Put the child on a chair and feed him.

c. Do not give snacks to the child before meals. - Rationale: If the child is hungry, he/she more likely would finish his meals. Therefore, the mother should be advised not to give snacks to the child. The child is a "busy toddler." He/she will not be able to keep still for a long time.

A mother brings her 8-month-old daughter to the clinic for well-child care. The mother says to a nurse, "My daughter loves to suck her thumb, and I'm worried this habit will cause permanent damage to her teeth." Which of these responses should the nurse give to the mother? a. You should wrap your daughter's thumb in adhesive tape. b. Offer her favorite toy to distract her from sucking her thumb. c. Infants need sucking; it's okay to let your daughter suck her thumb. d. Remove her thumb from her mouth when you see her thumbsucking.

c. Infants need sucking; it's okay to let your daughter suck her thumb.

A mother of a 3-year-old tells a clinic nurse that the child is rebelling constantly and having temper tantrums. The nurse most appropriately tells the mother to: a. Punish the child every time the child says "no", to change the behavior. b. Allow the behavior because this is normal at this age period. c. Set limits on the child's behavior. d. Ignore the child when this behavior occurs.

c. Set limits on the child's behavior.

A mother of a 30-month-old toddler who was treated for pinworm infestation is taught how to prevent a reoccurrence. Which statement by the mother indicates that the teaching is effective? Select all that apply. a. I will keep the cat off my child's bed. b. I will disinfect my child's room every 2 days. c. The family's sheets will be washed every day. d. The school nurse must be told to have the toilets cleaned. e. I will remind family members to take the medication again in 2 weeks.

c. The family's sheets will be washed every day. e. I will remind family members to take the medication again in 2 weeks. - Rationale: Washing clothing and bed linens daily will help limit transmission. Medications are effective, but must be repeated in 2 weeks to prevent reinfestations. Cats do not transmit pinworms. Disinfecting surfaces does not help prevent transmission. Toilets are not the usual mode of transmission; the rectal-oral cycle must be completed for an infestation to occur.

Which of the following actions should be included in the teaching plan for the parents of a preschool-aged child with pediculosis capitis? a. Apply calamine lotion or Burow's solution. b. Comb the child's hair each week. c. Wash all of the family's clothing and linens. d. Administer topical anti-itch medication twice a day.

c. Wash all of the family's clothing and linens.

An emergency department nurse prepares to treat a child with acetaminophen (Tylenol) overdose. The nurse reviews the physicians orders, expecting that which of the following will be prescribed? a. protamine sulfate. b. succimer (chemet). c. acetylcysteine (mucomyst). d. vitamin K (aquaMEPHYTON)

c. acetylcysteine (mucomyst) - Rationale: Acetylcysteine is the antidote for acetaminophen overdose. It is administered orally or via nasogastric tube in a diluted form with water, juice or soda. It can also be administered intravenously. Vitamin K is the antidote for warfarin; protamine sulfate is the antidote for heparin; succimer is used in the treatment of lead poisoning.

A patient with upper body obesity also has central fat distribution. This body fat configuration places the patient at greater risk for ________ than a patient with lower body obesity

cardiometabolic disorders

A diabetic patient has impaired sensation, circulation, and oxygenation of his feet. He steps on a piece of glass, the wound does not heal, and the area tissue becomes necrotic. The necrotic cell death is characterized by;

cellular rupture

A 20 yo college student has presented to her campus medical clinic for a scheduled Pap smear. The clinician who will interpret the smear will examine cell samples for evidence of:

changes in cell shape, size, and organization

Polycythemia develops in patients with lung disease as a result of:

chronic hypoxia

A person eating peanuts starts choking and collapses. His airway obstruction is partially cleared, but he remains hypoxic until he reaches the hospital. The prolonged cell hypoxia caused a cerebral infarction and resulting ___ in the brain.

coagulation necrosis

A woman had a cholecystectomy 4 days ago, her incision is in the proliferative phase of healing. What is the dominant cellular process that characterizes this phase?

collagen secretion by fibroblasts

An injured patient develops interstitial edema as a result of decreased:

collodial osmotic pressure

Aerobic metabolism, also know as oxidative metabolism, provides energy by:

combining hydrogen and oxygen to form water.

The patient is found to have liver disease, resulting in the removal of a lobe of his liver. Adaptation to the reduced size of the liver leads to ____ of the remaining liver cells?

compensatory hyperplasia

The vascular, hemodynamic stage of acute inflammation is initiated by momentary vasoconstriction followed by vasodilation that causes localized:

congestion

Postsurgical client who is recovering in the post anesthesia recovery unit (PACU) states that she is "freezing cold." Which of the following measures is likely to be initiated in the clients hypothalamus in an effort to reduce heat loss?

contraction of pilomotor muscles

A post surgical patient who is recovering in the postanesthetic recovery unit states that she is "freezing cold" Which of the following measuers is likely to be initiated in the hypothalamus in an effort to reduce heat loss?

contraction of the pilomotor muscles

A client has experienced significant decreases in mobility and stamina during a 3-week hospital stay for the treatment of a femoral head fracture. Which of the following phenomena most likely accounts for the client's decrease in muscle function? a. dysplasia as a consequence of inflammation during bone remodeling b. impaired muscle cell metabolism resulting from metaplasia c. ischemic atrophy resulting from vascular changes while on bedrest d. disuse atrophy of muscle cells during a prolonged period of immobility

d

A client has sought care because of recent malaise and high fever. Upon assessment, the client states that his current fever began 2 days earlier , although he states that for the last 2 weeks he is in a cycle of high fever for a couple of days followed by a day or two of normal temperature. Which of the following fever patterns is this client experiencing? A) Recurrent fever B) Remittent fever C) Sustained fever D) Intermittent fever

d

The cells of the client's epithelial tissue are tightly bound together by specialized junctions. Select the junctions primarily responsible for preventing cell separation. a) Occluding junctions b) Continuous tight junctions c) Nexus junctions d) Adhering junctions

d) Adhering junctions Explanation: The primary role of adhering junctions is preventing cell separation. Nexus junctions play a role in cell-to-cell communications. Continuous tight or occluding junctions seal the surface membranes of adjacent cells together.

A patient has been diagnosed with a neurodegenerative disease called multiple sclerosis (MS). The physician explains to the patient that this disease may be caused by dysregulated apoptosis. Later that day, the patient asks the nurse what this means. The nurse should reply: a) The cells around your nerves don't know how to die correctly. b) There is an inappropriately low rate of apoptosis occurring within the cells. c) The cytoplasm should neutralize the various apoptoic inhibitors but isn't working correctly. d) Dysregulated apoptosis has caused an excessive rate of programmed cell death along the neuro-pathways.

d) Dysregulated apoptosis has caused an excessive rate of programmed cell death along the neuro-pathways. Explanation: Dysregulated apoptosis can mean too little or too much and has been implicated in neurodengererative disease, in which there is an increased or excessive rate of apoptosis.

Which statement accurately explains function of cellular components? a) Microfilaments help conduct electrical signals between cells. b) Mitochondria make energy for the cells to use. c) Plasma membranes are impermeable. d) Microtubules aid intracellular transport.

d) Microtubules aid intracellular transport. Explanation: Microtubules assist in maintaining the cell shape and structure. They also aid in transporting components within cells. Microfilaments aid in cell wall contractions, and in maintaining cell shape. Mitochondria do not make energy, but break down organic compounds to extract energy. Plasma membranes are semipermeable.

Which of the following cellular organelles transforms organic compounds into energy that is easily accessible to the cell? a) Golgi complex b) Ribosomes c) Lysosomes d) Mitochondria

d) Mitochondria Explanation: The mitochondria are literally the "power plants" of the cell because they transform organic compounds into energy that is easily accessible to the cell. They do not make energy, but extract it from organic compounds.

Which identifies correctly how the G protein-linked receptors are similar? a) These linked receptors are involved in rapid synaptic signaling between cardiac electrical cells. b) Their cytosolic domain has intrinsic enzyme activity. c) Insulin is an example of the second messenger cAMP, which binds to an enzyme-linked receptor. d) They have a ligand-binding extracellular receptor component, which causes changes that activate the G protein on the cytoplasmic side of the cell membrane.

d) They have a ligand-binding extracellular receptor component, which causes changes that activate the G protein on the cytoplasmic side of the cell membrane. Explanation: Although there are differences among the G protein-linked receptors, all share a number of features. They all have a ligand-binding extracellular receptor component, which recognizes a specific ligand or first messenger. Upon ligand binding, they all undergo conformational changes that activate the G protein found on the cytoplasmic side of the cell membrane. Instead of having a cytosolic domain that associates with a G protein, enzyme-linked receptors have cytosolic domain either that has intrinsic enzyme activity or that associates directly with an enzyme. The binding of the hormone to a special transmembrane receptor results in activation of the enzyme adenylyl cyclase at the intracellular portion of the receptor. This enzyme then catalyzes the formation of the second messenger cAMP, which has multiple effects on cell function. Insulin, for example, acts by binding to an enzyme-linked receptor. Ion channel-linked receptors are involved in the rapid synaptic signaling between electrically excitable cells.

How does facilitated diffusion enable molecules to cross cell membranes? a) Substances use energy to move against the concentration gradient. b) Ion channels are triggered to open and molecules cross the membrane. c) Molecules are pulled in the direction of less concentration. d) Transport proteins carry molecules to areas of lower concentration.

d) Transport proteins carry molecules to areas of lower concentration. Explanation: Diffusion is movement of a substance across cell membranes in the direction of a lower concentration. Facilitated diffusion also moves substances to areas of lower concentration, but with the assistance of a carrier molecule. Ion channels are proteins that allow certain electrolyte ions to pass rapidly through the membrane. Active transport occurs when a transport protein carries substances across the cell membrane toward an area of higher concentration

A mother is concerned because her child was exposed to varicella in daycare. What is the nurse's best response? a. Initially, the rash appears as fluid filled blisters. b. The treatment of choice is aspirin. c. Varicella usually has an incubation period of 5 to 10 days. d. A child is no longer contagious once the rash has crusted over.

d. A child is no longer contagious once the rash has crusted over. - Rationale: Once every varicella lesion has crusted over, the child is no longer considered contagious. The rash is typically a maculopapular vesicular rash. The rash first appears as pink or red bumps which occur over several days. Following the rash, fluid filled vesicles form over the next 24 hours before breaking and leaking. THe use of aspirin has been associated with Reye's syndrome and is contraindicated in varicella. The incubation period for varicella is 10-20 days.

A child is receiving a series of the hepatitis B vaccine and arrives at the clinic with his parent for the second dose. Before administering the vaccine, the nurse should ask the child and parent about a history of severe allergy to which substance? a. Eggs. b. Penicillin. c. Sulfonamides. d. A previous dose of hepatitis B vaccine or component.

d. A previous dose of the hepatitis B vaccine or component. Rationale: A contraindication to receiving the hepatitis B vaccine is a previous anaphylactic reaction to a previous dose of hepatitis B vaccine or to a component (aluminum hydroxide or yeast protein) of the vaccine. An allergy to eggs, penicillin, and sulfonamides is unrelated to the contraindication to receiving this vaccine.

In which child population would the nurse be alert for giardiasis, the most common parasitic infection in the United States? a. Children riding a school bus. b. Children playing on a playground. c. Children attending sporting event. d. Children attending group day care or nursery school.

d. Children attending group day care or nursery school. - Rationale: Giardiasis is caused by protozoa and is prevalent among children in crowded environments, such as classrooms or day care centers.

Which statement by a 14-year-old girl receiving isotretinoin (Accutane) indicates that more teaching is needed? a. If I were to become pregnant, the medication could have severe effects on my baby. b. Use of this medication is reserved for people with severe inflammatory acne who have not responded to standard therapies. c. I will need to have my blood checked to monitor for side effects of this medication. d. If this medication does not work within two weeks, I will not be able to ever receive it again.

d. If this medication does not work within two weeks, I will not be able to ever receive it again.

A 7-year-old is seen in a clinic, and the primary health care provider documents a diagnosis of primary nocturnal enuresis. When the mother asks a nurse about the diagnosis, the nurse plans to respond knowing that: a. Primary nocturnal enuresis does not respond to treatment. b. Primary nocturnal enuresis is caused by a psychiatric problem. c. Primary nocturnal enuresis requires surgical intervention to improve the problem. d. Most children outgrow the bed-wetting problem without therapeutic interventions.

d. Most children outgrow the bed-wetting problem without therapeutic interventions. - Rationale: Primary nocturnal enuresis occurs in a child who has never been dry at night for extended periods. The condition is common in children, and most children eventually outgrow bed-wetting without therapeutic intervention. The child is unable to sense a full bladder and does not awaken to void. The child may have delayed maturation of the CNS. The condition is not caused by a psychiatric problem.

A mother asks the nurse how to handle her 5-year-old child, who recently started wetting the pants after being completely toilet trained. The child just started attending nursery school 2 days a week. Which principle should guide the nurse's response? a. The child forgets previously learned skills. b. The child experiences growth while regressing, regrouping, and then progressing. c. The parents may prefer less mature behaviors. d. The child returns to a level of behavior that increases the sense of security.

d. The child returns to a level of behavior that increases the sense of security.

The nurse provides home care instructions to the parents of a child hospitalized with pertussis who is in the convalescent stage and is being prepared for discharge. Which statement by a parent indicates a need for further instruction? a. We need to encourage our child to drink fluids. b. Coughing spells may be triggered by dust or smoke. c. Vomiting may occur when our child has coughing episodes. d. We need to maintain droplet precautions and a quiet environment for at least 2 weeks.

d. We need to maintain droplet precautions and a quiet environment for at least 2 weeks. - Rationale: Pertussis is transmitted by direct contact or respiratory droplets from coughing. The communicable period occurs primarily during the catarrhal stage. Respiratory precautions are not required during the convalescent phase. Options A, B, and C are accurate components of home care instructions.

Cellular atrophy results in:

decrease in tissue volume due to decrease in cell size

Protein-calorie malnutrition with loss of lean tissues and muscle mass results in

diarrhea

The syndrome of inappropriate ADH is characterized by

dilutional hyponatremia

An 84 yo patient's blood cultures have come back positive, despite the fact that his oral temp has remained within normal range. Which phenomena underlies the alterations in fever responses that occur in the elderly?

disturbance in the function ing of the thermoregulatory center

A change in a tissue marked by cells that vary in size and shape and show increased mitotic figures would be called:

dysplasia

Which of the following pathophysiologic phenomena may result in a diagnosis of Cushing's disease?

excess ACTH production by a pituitary tumor

Fetal alcohol syndrome is unlike other teratogens in that the harmful effects on the fetus:

extend throughout the pregnancy

Although both grading and staging are methods for classifying cancer and selecting a treatment plan, staging is used to determine the:

extent of disease spread

Although enegery is not made in mitochondria, they are known as the "power plants: of the cell because they:

extract energy from organic compounds.

Which pathway do extracellular signaling molecules, Fas ligand, bind to "death receptors" on the cell's surface?

extrinsic

Manifestations of anemia that are directly due to the diminished oxygen-carrying capacity of hemoglobin include:

fatigue

The acute phase systemic response usually begins within hours of the onset of inflammation and includes:

fever and lethargy

Splicing of mRNA during processing permits a cell to

form different proteins

A nursing student is cleaning and changing the dressing on a patient's sacral ulcer. The student has vigorously cleansed the wound bed to remove all traces of beefy, red tissue that existed in the wound bed. The student has most likely removed:

granulation tissue

Epithelialization, the first component of the proliferative phase of wound healing, is delayed in open wounds until after___ has formed.

granulation tissue

The angiogenesis process, which allows tumors to develop new blood vessels, is triggered and regulated by tumor-secreted:

growth factors

Disseminated intravascular coagulation (DIC) is characterized by:

hemorrhage

Rigorous weight lifting/body building regimens may result in the skeletal muscle cells undergoing

hypertrophy

A patient with severe peripheral vascular disease has developed signs of dry gangrene on the great toe of one foot. Which of the following pathphysiologic processes most likely contributed to this diagnosis?

impaired arterial blood supply

The pathologic effects of the thalassemias are primarily due to which of the following pathophysiologic processes?

impaired hemoglobin synthesis

Hemolytic anemia is characterized by excessive red blood cell destruction and compensatory:

increased erythropoiesis

Allostasis is characterized by

interactive physiologic changes in numerous systems

Leukemias are classified according to the predominant cell type. The myelogenous cell type of leukemia can:

interfere with thromobcyte cell maturation

Despite the low levels of radiation used in contemporary radiologic imaging, a radiology technician is aware of the need to minimize her exposure to ionizing radiation. What is the primary rationale for the technician's precautions?

interferes with DNA synthesis and mitosis

The client asks the health care provider to explain phagocytosis. The provider will respond, "Phagocytosis:

is a process where microorganisms are engulfed & subsequently degraded or killed"

A dominant genetic trait:

is expressed in either a homozygous or heretozygous pairing.

An elevated level of unconjugated bilirubin, due to hemolysis of RBC's, results in high level of iron released and:

jaundice

Natural appetite suppression mechanisms, necessary for food intake control, include

leptin receptor stimulation

A patient with a genetic condition that prevents arachidonic acid from being released from cell membranes would prevent formation of which inflammatory mediators?

leukotrienes

Because of their rapid growth, malignant tumors affect area tissues by:

liberating enzymes and toxins

Sue is fatigued and some blood tests are done. He results include Hct 40%, Hgb 8g/dL; WBC 8000, platelets 175,000. The nurse should interpret Sue's blood work as indicative of:

low hemoglobin/anemia

A patient has been diagnosed with non-Hodgkin lymphoma (NHL), a form of malignancy that most likely originated in which of the following sites?

lymph nodes

Adaptive immune responses, also called acquired or specific immunity, are composed of which of the following and their products?

lymphocytes

Which of the following statements is true of messenger RNA?

mRNA provides the template for protein synthesis

A client has an abscess in the mouth with a profuse amount of thick creamy white exudate. The nurse knows that this wound with necrotic cells is classified as:

membranous

A patient who has a 2 pack per day cigarette smoking history presents at a facility with a respiratory disturbance. Examination of his airway demonstrates that the stratified epithelial cells have replaced the normal columnar ciliated cells. This type of adaptation is called

metaplasia

Semen analysis indicates that the client's sperm have decreased motility. Which of the following cellular components may be defective within the client's sperm?

microtubules

Although the majority of cellular DNA exists in the cell nucleus, part of the cell DNA is located in the:

mitochondria

Multifacotrial inheritance is similar to polygenic inheritance because both involve:

multiple alleles at different loci

Cancerous transformation of a cell requires the activation of:

multiple mutations

The cellular state of acute inflammation is marked by the movement of leukocytes into the area. Which of the following cells arrives early in great numbers?

neutrophils

During lecture on wound care, the final stage of cellular response of acute inflammation is described by

neutrophils, monocytes, and macrophages engulf and degrade the bacteria/ cellular debris.

Which gene is a mutated gene?

oncogene

An older adult patient has just sheared the skin on her elbow while attempted to boost herself up in bed, an event that has percipitated acute inflammation in the region surrounding the wound. Which of the following events will occur during the vascular stage of the patient's inflammation?

outpouring of exudate into interstitial spaces

An older adult patient has just sheared the skin on her elbow while attempting to boost herself out of bed, causing acute inflammation around the wound. Which event will occur during the vascular stage?

outpouring of exudate into interstitial spaces

A mother has brought her 2 week old infant to the emergency department due to the baby's persistent and increasing jaundice. Blood testing reveals that the infant's unconjugated bilirubin level is 28 mg/dL and assessment does not reveal neurologic deficits. The infant's weight is normal and the mother claims to have had no significant difficulty feeding the infant. The most likely treatment for this infant will be:

phototherapy

Multiple myeloma is a malignancy of:

plasma cells

During the transcription process, RNA

polymerase attaches to DNA

Inflammation is ultimately needed to:

prepare the site for healing

a 69 year old patient who is obese and has a diagnosis of angina pectoris has been prescribed clopidogrel (Plavix) by his primary care provider. The patient's medication achieves its therapeutic effect in which of the following ways?

prevention of platelet aggregation

Exogenous pyrogens and the presence of bacteria in the blood lead to the release of endogenous pyrogens that:

produce leukocytosis and anorexia

Triplet codes of three bases are the genetic codes used in transmitting genetic information necessary for:

protein synthesis

An elderly client is dressed only in a hospital gown and complains of a draft in her room. Consequently, she has requested a warm blanket while she sits in her wheelchair. Which of the following mechanisms of heat loss is most likely the primary cause of her request?

radiation and convection

Despite the low levels of radiation used in contemporary radiologic imaging, a radiology tech is aware of the need to minimize her exposure to ionizing radiation. What is the primary rationale for the tech's precautions?

radiation interferes with dan synthesis and mitosis

The phagocytosis process involves three distance steps. What is the initial step in the process?

recognition and adherence

Cytokines that affect hematopoiesis in bone marrow are called colony stimulating factors (CSF's) based on their ability to:

regulate blood cells

Stress-induced cortisol hormone secretion is associated with

regulation of the stress response

Ischemia and other toxic injuries increase the accumulation of intracellular calcium as a result of:

release of stored calcium from the mitochondria.

To effectively relay signals, cell to cell communication utilizes chemical messenger systems that:

release secretions into extracellular fluid.

One of the major causes of hyperkalemia is __________, which alters potassium elimination

renal dysfunction

The patient recently returned from a year of military battle duty and has posttraumatic stress disorder (PTSD). This disorder includes an "intrusion" state that is experienced as

repeated relived memories as nightmares

A teratogenic environmental agent can cause birth defects when:

retained during early pregnancy

Depth of injury is important to determine with burns. You are in the sun too long without sunscreen and develop redness and blistering on your face, chest, and back. What depth of burn did you experience?

second degree

Client has a watery fluid leaking from a site of inflammation. The nurse would document this types of exudate as:

serous

Early diagnosis of childhood cancers is often difficult because the signs and symptoms are:

similar to those of other childhood diseases

Prenatal diagnosis methods include the use of ultrasonography from identifying ____ abnormalities

skeletal

Although clinical manifestations vary with the type of cancer and organs involved, abnormal tumor growth causes general manifestations that include:

sleep disturbances

Some tissues, such as the mucosal surface of the gastrointestinal tract, have cells that are capable of cell division and serve as a reserve source for specialized cells. What term describes these specialized cells?

stem cells

connective tissue contains fibroblasts that are responsible for:

synthesis of collagen, elastin, and reticular fibers.

The adipocytes in an adipose tissue not only serve as a storage site, they also

synthesize triglycerides

A pregnant client is attending a nutrition class for first-time moms. During the class, the instructor stressed that they should avoid consumption of which food that may cause brain damage from methyl mercury exposure?

tuna

The newborn has the distinctive physical features of trisomy 21, Down syndrome, which includes:

upward slanting of eyes

In addition to having a 50% chance of inheriting an autosomal dominant disorder from an affected parent, such a disorder is characterized by:

varied gene penetration and expression

Increased platelet function, and consequent hyper coagulability, can be caused by:

vascular wall damage

Which of the following membrane transport mechanisms requires the greatest amount of energy?

vesicular transport.

The first step of hemostasis occurs as a:

vessel spasm

The majority of the cell's protoplasm is:

water

Which client most likely faces the highest risk of neuroleptic malignant syndrome

young adult client who is taking Risperdal, an antipsychotic medications to block dopamine receptors, for the treatment of schizophrenia

A client presents to a health clinic complaining of several vague symptoms. As the history/physical continues, the health care provider clearly thinks the client may have myasthenia gravis. Which statements by the client would correlate with this diagnosis? Select all that apply.

• "Sometimes I have double vision." • "I have more energy in the morning but get worse as the day goes by." • "I feel like I don't have enough energy to chew my food sometimes.

Which of the following interventions would be considered a nonpharmacologic method of pain control? Select all that apply.

• Distraction by knitting • Guided imagery • Biofeedback

Which of the following should a nurse stress when teaching patients to avoid exposure to lead in the environment? Select all that apply. a) Not everyone has a reaction to lead. b) Repeated exposure to small amounts of lead is not a problem. c) Root vegetables can contain more lead than other vegetables. d) Avoid flaking paint. e) Lead can contaminate soil

• Lead can contaminate soil. • Root vegetables can contain more lead than other vegetables. • Avoid flaking paint. Explanation: Lead is a very toxic metal and small amounts can accumulate to reach toxic levels. The nurse should teach patients to avoid flaking paint, especially in older homes, as older paint contains lead. Lead can contaminate soil, and root vegetables tend to contain more lead.

During a discussion of different cell types in the body, the instructor mentions that which types of cells renew themselves continuously throughout life?

• Neurons • Cells lining the GI tract


Related study sets

Economics // Chapter 8 Content // Aggregate Expenditures

View Set

C 207 Data Driven Decision Making

View Set

World Geography A - Unit 1 (Primavera - Online)

View Set

Issues Related to Pediatric Health

View Set

Med-Surg I: ATI Practice Cardiovascular

View Set

Disease Conditions Ch 7 Poss ? may be on test.

View Set

PASS 610 - Exam 3 - Pelvis & Pelvic Viscera

View Set

Direct Variation, Constant of Proportionality, and Slope

View Set